Материалы для подготовки к ЕГЭ

Евдокимова Людмила Алексеевна

Предварительный просмотр:

Ep1+Ek1= Ep2+Ek2

E= Ek+Ep

E1=E2

Полная механическая энергия

Дж

Дж

Дж

Закон Бернулли

м2, м/с

Колебания и волны

Механические колебания

   

Гармонические колебания: координата тела, скорость и ускорение в момент времени

м (м; с)

м/с (м/с, с, м/с, м)

м/с2 (м/с2, с, м/с2, м)

T==2π =2π

Период свободных колебаний математического маятника и тела на пружине

с (с; м, м/с2; кг, Н/м)

Частота колебаний и циклическая частота

Гц (с); рад/с (с; Гц)

Ускорение при колебаниях тела на пружине и математического маятника

м/с2 (Н/м, м, кг; м/с2, м, м)

Электромагнитные колебания

Колебательный контур: заряд, сила тока в момент времени

Кл  (Кл, с)

А (А, с)

Период колебаний в колебательном контуре

с (Гн, Ф)

 

Частота и циклическая частота в колебательном контуре

рад/с (Гн, Ф); Гц (с, рад/с)

Колебательный контур: магнитный поток, ЭДС и напряжение в момент времени

Вб (Тл, м2)

В (Вб, В);  В (Тл, м2, рад/с)

В (В, рад/с)

Действующие значения напряжения и силы тока при переменном токе

В (В);  А (А)

Ёмкостное сопротивление и закон Ома.  

Опережение колебаний I от U на π/2.

Ом (с-1, Ф);  А (В, Ом)

А (А, рад/с, с; В, Ф, рад/с, рад/с, с)

Индуктивное сопротивление и закон Ома для катушки. Отставание колебаний I от U на π/2.

Ом (Гн, с-1); А (В, Ом)

В (Гн, рад/с, А, рад/с, с; В, рад/с, с)

Полное сопротивление

Ом (Ом, Ом, Ом)

Коэффициент трансформации

Механические волны

υ==

Скорость волны

м/с (м, с; Гц)

Расстояние от ист. звука до отраж. звука

м (м/с; с)

Уравнение бегущей волны

м (м, рад/с, с, м, м/с)

Электромагнитные волны

Длина электромагнитной волны

м (м/с, Гн, Ф; м/с, Гц)

Интенсивность электромагнитной волны

Вт/м2 (Дж, м2, с; Вт, м2; Дж/м3, м/с)

Плотность энергии электромагнитной волны

Дж/м3 (Дж, м3)

Молекулярная физика. Тепловые явления

Основы молекулярно-кинетической теории

Кол-во вещества через молярную массу, объём и число Авогадро

моль (кг, кг/моль, моль-1, м3)

Концентрация частиц

м-3 (м3)

Средняя скорость молекул идеального газа

м/с (м/с; Дж/К, К, кг)

Давление идеального газа

Н/м (кг, м-3, м/с, Дж, кг/м3, К)

Относительная влажность воздуха

% (Па, Па; кг/м3, кг/м3)

Энергия теплового движения молекул

Средняя кинетическая энергия поступательного движения частиц

Дж (Дж/К, К)

Уравнение сост. идеального газа. Уравнение Клайперона (m=const)

Па, м3  (кг, кг/моль, Дж/к×моль, К)

 

Уравнение термодинамического равновесия

Па, м3 (Дж/К, К)

Основы термодинамики

, где i = 3; 5; 6

Внутренняя энергия идеального газа: 1-, 2- и 3-атомного

Дж (кг, кг/моль, Дж/К×моль, К; Па, м3)

Изменение внутренней энергии и количествава теплоты

Дж (Дж, Дж); Дж (Дж, Дж)

Работа идеального газа в термодинамике

Дж (Па, м3; моль, Дж/К×моль, К)

Кол-во теплоты при (p, T, V)=const. Адиабатный процесс.

Дж (Дж, Дж); Дж (Дж); Дж (Дж); Дж (Дж)

Механика

Кинематика

Кинематика точки

S= υt;

x=x0+ υt

Равномерное прямолинейное движение

м (м/с, с);

м (м, м/с, с)

Ускорение при равноускоренном прямолинейном движении

м/с2 (м/с, м/с, с)

Перемещение  при равноускоренном прямолинейном движении  

м (м/с, м/с, с; м/с, с, м/с2, с; м/с, м/с, м/с2)

x=x+S=x0 +

Координата точки  при равноускоренном прямолинейном движении

м (м, м; м, м/с, с м/с2, с)

Средняя скорость

м/с (м, с)

Кинематика твёрдого тела

a= =4π2r ν2

Центростремительное и линейное ускорение при движении по окружности

м/с2 (м/с, м; м, Гц)

Частота обращения  при движении по окружности

Гц (с)

υ=2πr ν=ωr

Линейная скорость при движении по окружности

м/с (м, Гц; рад/с, м)

Угловая скорость  при движении по окружности

рад/с (рад, с; Гц)

Динамика

Законы механики Ньютона

F=0, то a=0        

I закон Ньютона

Н

F=ma

II закон Ньютона

Н (кг, м/с2)

III закон Ньютона

Н, Н

Силы в механике

Fупр=kx (=N;P)

Сила упругости

Н (Н/м, м)

Fтр=μN (N= -P)

Сила трения

Н (Н)

Fгр=G

Гравитационная сила

Н (Н×м2/кг2, кг, кг, м)

Fтяж=mq

Сила тяжести

Н (кг, м/с2)

Сила притяжения к Земле  и первая космическая скорость

Н (Н×м2/кг2, кг, кг, м; кг, м/с2; кг, м/с, м)

м/с (Н×м2/кг2, кг, м)

Ускорение свободного падения на высоте h планеты и от поверхности Земли

м/с2 (Н×м2/кг2, кг, м; м, м)

Ускорение в зависимости от массы и плеча

м/с2, м/с2; кг, кг; м, м

y=y0+ υ0 t –

υ y= υ0 - qt

Скорость и координата тела при движении ↑

м (м, м/с, с, м/с2, с)

м/с (м/с, м/с2, с)

y=y0   υ0 t –

υ y= - υ0 – qt

Скорость и координата тела при движении ↓

м (м, м/с, с, м/с2, с)

м/с (м/с, м/с2, с)

x= υ0 t,  υx= υ0 ;

y=y0 –, υy=-qt 

Скорость и координата тела при движении →

м (м/с, с); м/с (м/с)

м (м/с, м/с2, с); м/с

υ0x= υ0 cosα, υx=  υ0x

x= υ0 cosα t;

υ0y= υ0 sinα, υy= υ0 sinα - qt

.

Скорость и координата тела при движении под углом к горизонту

м/с; м/с

м

м/с; м/с

м

;

Время подъёма тела и время полёта тела

с (м/с, °, м/с2); с (м/с, °, м/с2)

Максимальная высота подъёма тела. Дальность полёта тела

м (м/с, °, м/с2); м (м/с, °, м/с2)

Абсолютное и относительное удлинения

м (м, м)

Жесткость

Н/м (Н/м2, м2, м)

Закон Гука

Статика

F1+F2+…+Fn=0

Геометрическая сумма сил, приложенных к телу

Н

M=Fd

M1+M2+…+Mn=0

Момент силы

Н×м (Н, м)

Рычаг

Н, Н; м, м

Законы сохранения в механике

Закон сохранения импульса

Ft=mυ-mυ0

Равенство импульса силы и тела

Нс; кгм/с

I=Ft

Импульс силы

Н×с (Н, с)

P=mυ

Импульс тела

кг×м/с (кг, м/с)

m1υ1+ m2υ2=m1υ'1+ m2υ'2

Закон сохранения импульса

кг×м/с (кг, м/с)

υоб = 

Реактивное движение

м/с (кг, м/с, кг)

Закон сохранения энергии

A=Fs cosα

Работа

Дж (Н, м)

N=

Мощность

Вт (Дж, с; Н, м/с)

Ek=

Кинетическая энергия тела

Дж (кг, м/с)

Ep=

Потенциальная энергия деформированного тела

Дж (Н/м, м)

Ep=mgh

Потенциальная энергия поднятого тела

Дж (кг, м/с2, м)

A=∆ Ek= -∆ Ep=mgh

Работа

Дж (Дж; Дж, кг, м/с2, м)



Предварительный просмотр:

Мощность электрического тока

Вт (Дж, с; В, А)

Q=I2Rt

Количество теплоты, выделяемое проводником

Дж (А, Ом, с)

Магнитное поле

Сила Ампера

Н (Тл, А, м)

Сила Лоренца и радиус описанной окружности

Н (Кл, м/с, Тл); м (кг, м/с, Кл, Тл)

Электромагнитная индукция

Магнитный поток

Вб (Тл, м2)

Электромагнитная индукция

В (Вб, с)

ЭДС самоиндукции и магнитный поток при ней

В (Гн, А, с); Вб (Гн, А)

Энергия магнитного поля тока и конденсатора

Дж (Гн, А); Дж (Кл, Ф)

Оптика

Световые волны

Закон преломления света. Предельный угол отражения.

(°, м/с); °

Формула тонкой линзы

Дптр (м, м, м); м (м, м)

Увеличение линзы

(м, м)

Условие максимумов и минимумов

м (м); м (м)

Максимумы в дифракционной решётке

м , °, м

Элементы теории относительности

Относительность промежутков массы, времени, расстояний

кг (кг, м/с, м/с); с (с, м/с, м/с); м (м, м/с, м/с)

Релятивистский закон сложения скоростей

м/с (м/с, м/с, м/с, м/с)

Формула Эйнштейна. Энергия покоя. Полная энергия.

Дж (кг, м/с; кг, м/с, м/с, м/с); Дж (кг, м/с); Дж (Дж, Дж)

Квантовая физика

Световые кванты

Энергия кванта

Дж (Джс, с-1)

Длина волны де Бройля (излучаемая движущимися частицами)

м (Дж с, кгм/с)

Теория фотоэффекта

Джс, с-1; Дж, кг, м/с

Энергия и импульс фотона

Дж (Джс, с-1), кгм/с (Джс, с-1)

Давление света

Па (Вт/м2, м/с)

Атомная физика

Энергия излучённого или поглощенного фотона

Джс, Гц (Дж, Дж)

Частота света при переходе из стационарного состояния n в k

Гц (Гц)

Физика атомного ядра

Энергия связи нуклонов в атомных ядрах

МэВ (а.е.м., МэВ/а.е.м.; а.е.м., МэВ/а.е.м.)

Удельная энергия связи нуклона в атомных ядрах

МэВ

Закон радиоактивного распада

(с, с)

Правила смещения для α, β, γ и +β распадов

Доза излучения

Гр (Дж, кг)

Теплоемкость тела и молярная теплоемкость

Дж/К (Дж, К), Дж/моль×К (кг/моль, Дж/К×моль)

Удельная и молярная теплоем-ти при  изобаре

Дж/К; Дж/моль×К

Удельная и молярная теплоем-ти при  изохоре

Дж/К (Дж/К×моль, моль); Дж/моль×К (Дж/К×моль)

Тепловой баланс в замкнутой системе

Дж

КПД и максимальный КПД теплового двигателя

% (Дж, К)

Твердые тела

Q=cm∆t

Количество теплоты

Дж (Дж/кг, кг, К)

Взаимные превращения жидкостей и газов

Q=qm

Теплота сгорания

Дж (Дж/кг, кг)

Q=±λm

Теплота плавления

Дж (Дж/кг, кг)

Q=±Lm

Теплота парообразования

Дж (Дж/кг, кг)

Гидростатика

Давление

Н/м2 (Н, м2)

FA=жVт

Сила Архимеда

Н (Н/кг, кг/м3, м3)

Гидравлический пресс

Н; м2

Давление в жидкостях

Н/м2 (Н, м2)

Плотность

кг/м3 (кг, м3)

Электродинамика

Электростатика

Алгебр, сумма зарядов

Кл

Сила взаимодействия 2-х точечных зарядов

Н (Н×м2/Кл2, Кл, м)

Напряженность поля

В/м (Н, Кл; Кл Н×м2/Кл2, м; В, м)

Геометрическая сумма напряжений

В/м

;

Диэлектрическая проницаемость среды и сила, действ, на заряд

;

Потенциал. энергия заряда в однородном поле и двух точечных зарядов

Дж (Кл, В/м, м)

Работа электрического поля

Дж (Дж)

Поверхностная плотность заряда

Кл/м2 (Кл, м2)

Потенциал, разность потенциалов и потенциал точечного заряда

В (Дж, Кл); В (В, Дж, Кл)

Электроемкость и электроёмкость плоского конденсатора

Ф (Кл, В); Ф (м2, м)

Потенциальная энергия плоского конденсатора

Дж (Кл, В/м, м, Кл, В, Ф, В, Кл, Ф)

Электродвижущая сила

В (Дж, Кл)

Отношения напряжения и силы тока к сопротивлению

В; А; Ом

Закон Ома для полной цепи и ЭДС источника

А (В, Ом); В (В, В)

КПД источника

% (Ом; В)

Зависимость сопротивления проводника от температуры

1, К-1, К; Ом×м,  К-1, К

m=kIt;

Закон электролиза и электролитический эквивалент

кг (кг/Кл, А, с); кг/Кл (Кл, моль-1, моль, м-3)

Законы постоянного тока

Сила тока и производная от силы тока, концентрации и др. ед.

А (Кл, с); А (Кл, м-3, м/с, м2)

Электрическое напряжение

В (Дж, Кл)

Электрическое сопротивление для участка цепи

Ом (В, А)

Электрическое сопротивление проводника

Ом (Оммм2/м, мм2,м)

I=I1=I2

R=R1+R2

U=U1+U2

Последовательное соединение проводников

А

Ом

В

I=I1+I2

U=U1=U2

Параллельное соединение проводников

А

В

Ом

U=U1+U2

q=q1=q2

Последовательное соединение конденсаторов

Ф

В

Кл

С=С12

U=U1=U2

q=q1+q2

Параллельное соединение конденсаторов

Ф

В

Кл

A=Uq=UIt

Работа электрического тока

Дж (В, Кл; В, А, с)

G

Гравитационная постоянная

6,672×10-11 Н×м2/кг2

NA

Постоянная Авогадро

6,022×1023 моль-1

Vп

Моль-объём газов

22,4 л/моль

k

Постоянная Больцмана

1,3807×10-23 Дж/К

k

Коэффициент Кулона

9×109 Н×м2/Кл2

R

Универсальная газовая постоянная

8,31 Дж/К×моль

R

Постоянная Ридберга

3,20×1015 Гц

e

Элементарный заряд

1,60219×10-19 Кл

ε0

Электрическая постоянная

8,854×10-12 Ф/м

c

Скорость света в вакууме

2,9979×108 м/с

h

Постоянная Планка

6,626×10-34 Дж×с=4,136×10-15 эВ×с

T

Соотношение тем-ры по Кельвину и по Цельсию

0 К = -273,15° С

а.е.м

Соотношение между а.е.м. и кг

1 а.е.м. = 1,66×10-27 кг

с2

1 единица массы эквивалента

931 МэВ/а.е.м.

эВ

Энергия, приобретённая эл. пройдя U в 1В

1 эВ =1,6×10-19 Дж

me

Масса электрона

9,1095×10-31кг=5,486×10-4 а.е.м.

mp

Масса протона

1,6726×10-27кг=1,00728 а.е.м.

mn

Масса нейтрона

1,6749×10-27кг=1,00867 а.е.м.



Предварительный просмотр:

МЕХАНИКА

  • Кинематика.

Перечень элементов содержания раздела «Кинематика материальной точки», проверяемых на едином государственном экзамене по физике таков:

Код

раздела

Код

контролируемого

элемента

Элементы содержания,

проверяемые заданиями КИМ

1

МЕХАНИКА

1.1

КИНЕМАТИКА

1.1.1

Механическое движение и его виды

1.1.2

Относительность механического движения

1.1.3

Скорость

1.1.4

Ускорение

1.1.5

Равномерное движение

1.1.6

Прямолинейное равноускоренное движение

1.1.7

Свободное падение (ускорение свободного падения)

1.1.8

Движение по окружности с постоянной

Обобщенный план экзаменационной работы ЕГЭ 2011 г. по физике

Обозначение

задания в

работе

Проверяемые элементы

содержания

Коды

элементов

содержания

по кодификатору

элементов

содержания

Уровень

сложности

задания

Макс. балл

за

выполнение

задания

А1

Кинематика

1.1.1.-1.1.8.

Б

1

А2

Кинематика,

законы Ньютона

1.1.5.-1.1.8.

1.2.1.- 1.2.8.

Б

1

А7

Механика

1.1. - 1.5

П

1

А24-25

Механика –

квантовая физика

 (методы познания)

1.1. – 5.3

Б,П

1

В1-В4

Механика –

квантовая физика

 (задачи на соответствие)

1.1. – 5.3

Б,П

2

С1-С2

Механика –

квантовая физика

 (расчетные задачи)

1.1. – 5.3

Б,П

3

Особое внимание при обобщении материала по данной теме необходимо обратить внимание на следующие понятия и определения:

  • механическое движение;
  • материальная точка;
  • поступательное движение;
  • система отсчета;
  • тело отсчета;
  • система координат;
  • траектория;
  • путь;
  • перемещение;
  • скорость (мгновенная, средняя);
  • ускорение;
  • свободное падение;
  • баллистика;
  • периодическое движение;
  • линейная скорость;
  • угловая скорость;
  • период;
  • частота;
  • работа с векторами.

При повторении теории можно составить следующие схемы:

или таблицы:

Вид механического движения

Характеристика движения

Уравнение

График

Примечание

конечная координата

перемещение

Путь, траектория, перемещение.

Равномерное прямолинейное движение

скорость

             

               

Перемещение численно равно площади фигуры, ограниченной графиком скорости.

перемещение

Равнопеременное  прямолинейное движение

ускорение

скорость

перемещение

Баллистика

скорость

υоx = υ0cos β,                  υоу = υosin β - gt,

перемещение

,              

уравнение траектории

                             

дальность полета

время полета

Движение по окружности

центростремительное ускорение

R                                         

       

                               

линейная скорость

угловая скорость

                                                      ω 

                                                               ω=                                           

период

частота

Рассмотрим некоторые задачи базового уровня. В первую очередь предлагаю разобрать примеры равноускоренного прямолинейного движения, причем  движение в них описывается словесно, графически и с помощью таблиц.

Часть А.

А1

t, c

0

1

2

3

4

5

x1, м

0

2

4

6

8

10

x2, м

0

0

0

0

0

0

x3, м

0

1

4

9

16

25

x4, м

0

2

0

-2

0

2

Четыре тела двигались по оси Ох. В таблице представлена зависимость их координат от времени.

       

У какого из тел скорость могла быть постоянна и отлична от нуля?

1) 1                         2) 2                         3) 3                         4) 4

А2

На рисунке приведен график зависимости проекции скорости тела от времени. Проекция ускорения тела в интервале времени от 12 до 16 с представлена графиком

1)

2)

3)

4)

А3        

На рисунках изображены графики зависимости модуля ускорения от времени движения. Какой из графиков соответствует равномерному прямолинейному движению?

1)        а        2) а        3)  а        4) а

0        t        0         t        0        t        0        t

А4.

На графике показана зависимость скорости тела от времени. Каков путь, пройденный телом к моменту времени t = 4 c?

                1)        7 м               2)        6 м              3)        5 м                 4)        4 м

А5.

На рисунке изображен график зависимости координаты бусинки, движущейся по горизонтальной спице, от времени. На основании графика можно утверждать, что

1)

на участке 1 бусинка покоится, а на участке 2 – движется равномерно

2)

на участке 1 движение является равномерным, а на участке 2 –равноускоренным

3)

проекция ускорения бусинки всюду увеличивается

4)

на участке 2 проекция ускорения бусинки положительна

 

        В части А также встречаются задачи на сравнение физически величин. При решении подобных задач рекомендуется проводить расчетные действия.

А6

Материальная точка движется по окружности с постоянной по модулю скоростью. Как изменится модуль ее центростремительного ускорения, если скорость точки увеличить втрое?

1)        увеличится в 3 раза                        3)        уменьшится в 3 раза

2)        увеличится в 9 раза                        4)        уменьшится в 9 раза

А7

Две  материальные точки движутся по окружностям радиусами R и R= 2R1 с одинаковыми по модулю скоростями. Их периоды обращения по окружностям связаны соотношением

1)

Т1 = 2Т2

2)

Т1 = Т2

3)

Т1 = 4Т2

4)

Т1 = Т2

А8.

Две материальные точки движутся по окружностям радиусами R1 и R2, причем R2 = 2R1. При условии равенства линейных скоростей точек их центростремительные ускорения связаны соотношением

1)

a1 = 2a2

2)

a1 = a2

3)

a1 = a2

4)

a1 = 4a2

А9.

Мотоциклист и велосипедист одновременно начинают равноускоренное движение.  Ускорение мотоциклиста в 3 раза больше, чем у велосипедиста. В один и тот же момент времени скорость мотоциклиста больше скорости велосипедиста

1)

в 1,5 раза

2)

в  раза

3)

в 3 раза

4)

в 9 раз

В части А включены задачи повышенной сложности. Это как правило расчетные задачи

А10.

На последнем километре тормозного пути скорость поезда уменьшилась на  

10 м/с. Определите скорость в начале торможения, если общий тормозной путь поезда составил 4 км, а торможение было равнозамедленным.

1) 20 м/с                 2) 25 м/с                 3) 40 м/с                 4) 42 м/с

А11.

Велосипедист съезжает с горки, двигаясь прямолинейно и  равноускоренно. За время спуска скорость велосипедиста увеличилась на 10 м/с. Ускорение велосипедиста 0,5 м/с2. Сколько времени длится спуск?

1)

0,05 с

2)

2 с

3)

5 с

4)

20 с

Часть В.

Задачи на соответствие условно можно разделить на следующие группы:

физическая величина

- изменение величины

график

- физическая величина

физическая величина

- приборы

физическая величина

- формулы

физическая величина

- определение

физическая величина

- единицы измерения

ученые

- открытия

В1.

Материальная точка движется по окружности радиуса R. Что произойдет с периодом, частотой обращения и центростремительным (нормальным) ускорением точки при увеличении линейной скорости движения в 2 раза?

К каждому элементу первого столбца подберите соответствующий элемент из второго и внесите в строку ответов выбранные цифры под соответствующими буквами.

ФИЗИЧЕСКИЕ

ВЕЛИЧИНЫ

A. Период обращения материальной точки

Б. Частота обращения

материальной точки

В. Центростремительное (нормальное) ускорение материальной точки

ИЗМЕНЕНИЕ

ВЕЛИЧИНЫ

1) увеличится

2) уменьшится

3) не изменится

А

Б

В

        

В2.    

Тело начинает двигаться из состояния покоя. На рисунке изображен график зависимости ускорения тела от времени движения.

Установите соответствие между графиками и физическими величинами, зависимости которых от времени эти графики могут представлять. К каждой позиции первого столбца подберите соответствующую позицию второго и запишите в таблицу выбранные цифры под соответствующими буквами. Цифры в ответе могут повторяться.

ГРАФИКИ

А

Б

ФИЗИЧЕСКИЕ ВЕЛИЧИНЫ

1)  проекция силы тяжести, действующая на тело

2)  скорость тела

3)  путь, пройденный телом

4)  проекция импульса тела

В3.

Установите соответствие между физическими величинами и приборами,

с помощью которых эти величины определяются.

К каждой позиции первого столбца подберите нужную позицию из второго и запишите в таблицу выбранные цифры под соответствующими буквами.

ФИЗИЧЕСКИЕ

ВЕЛИЧИНЫ

А) мгновенная скорость

Б) время

В) перемещение

 ПРИБОРЫ

1) часы

2) спидометр

3) гигрометр

4) измерительная лента

5) акселерометр

А

Б

В

В4.

Установите соответствие между физическими величинами и формулами, по которым эти величины определяются.

К каждой позиции первого столбца подберите нужную позицию из второго и запишите в таблицу выбранные цифры под соответствующими буквами.

ФИЗИЧЕСКИЕ

ВЕЛИЧИНЫ

А) ускорение

Б) перемещение

В) период

ФОРМУЛЫ

А

Б

В


В5

Установите соответствие между физическими величинами и их определениями.

К каждой позиции первого столбца подберите нужную позицию из второго и запишите в таблицу выбранные цифры под соответствующими буквами

ФИЗИЧЕСКИЕ

ВЕЛИЧИНЫ

ОПРЕДЕЛЕНИЯ

А)  Перемещение

Б) Средняя скорость

В)   Угловая скорость

1)  физическая величина, равная отношению угла   поворота радиуса-вектора к промежутку времени , за который этот поворот произошел.

2)  скалярная величина, равная отношению пути к промежутку времени, затраченному на его прохождение.

3)  физическая величина, равная длине траектории

4)  отношение изменения скорости  к промежутку времени, в течение которого это изменение произошло

5)   вектор, проведенный из начального положения материальной точки в конечное.

А

Б

В

В 6

Установите соответствие между физическими величинами и их единицами измерения.

К каждой позиции первого столбца подберите нужную позицию из второго и запишите в таблицу выбранные цифры под соответствующими буквами

ФИЗИЧЕСКИЕ  ВЕЛИЧИНЫ

ЕДИНИЦЫ ИЗМЕРЕНИЯ

А) Угловая скорость

Б) Линейная скорость

В) Частота

  1. с

А

Б

В

В 7

Установите соответствие между физическими  открытиями и  именами ученых.

К каждой позиции первого столбца подберите нужную позицию из второго и запишите в таблицу выбранные цифры под соответствующими буквами

ИМЕНА УЧЕНЫХ

ОТКРЫТИЯ, ЗАКОНЫ

А) Исаак Ньютон

  1. Закон инерции

Б) Галилео Галилей

  1. Движение планет и светил в геоцентрической системе

  1. Законы классической механики

  1. Основоположник теории космических полетов.

  1. Движение планет и светил в гелиоцентрической системе

А

Б

Часть С.

С1

Наклонная плоскость пересекается с горизонтальной плоскостью по прямой AB. Угол между плоскостями α = 30°. Маленькая шайба начинает движение вверх по наклонной плоскости из точки A с начальной скоростью v0 = 2 м/с под углом β = 60° к прямой AB. В ходе движения шайба съезжает на прямую AB в точке B. Пренебрегая трением между шайбой и наклонной плоскостью, найдите расстояние AB.

Образец возможного решения

Выбор системы координат: ось x направлена по прямой АВ, ось y – вверх по наклонной плоскости перпендикулярно линии АВ (см. рис.).

Проекции вектора ускорения свободного падения g:

gx = 0,    gy = – g sin α

Кинематика движения по наклонной плоскости эквивалентна кинематике движения тела, брошенного под углом β к горизонту, в поле тяжести с ускорением  g sinα.

Выписывание уравнений движения вдоль осей x и y (в известных уравнениях для тела, брошенного под углом β к горизонту, делается замена g → g sin α):

Ответ на вопрос задачи находится из этих уравнений при наложении дополнительных условий.

Условие y = 0 позволяет найти расстояние АВ, исключая время t из выписанных уравнений для x и y:  .

Критерии оценки выполнения задания

Баллы

Приведено полное правильное решение, включающее следующие элементы:

— верно записаны формулы, выражающие физические законы, применение которых необходимо для решения задачи выбранным способом (в данном решении — формулы кинематики для равноускоренного движения, тригонометрические соотношения для проекций величин);

— проведены необходимые математические преобразования и расчеты, приводящие к правильному числовому ответу, и представлен ответ. При этом допускается решение "по частям" (с промежуточными вычислениями).

3

— Представлено правильное решение только в общем виде, без каких-либо числовых расчетов.

ИЛИ

— Правильно записаны необходимые формулы, записан правильный ответ, но не представлены преобразования, приводящие к ответу.

ИЛИ

— В математических преобразованиях или вычислениях допущена ошибка, которая привела к неверному ответу.

2

– В решении содержится ошибка в необходимых математических преобразованиях и отсутствуют какие-либо числовые расчеты.

ИЛИ

– Записаны все исходные формулы, необходимые для решения задачи, но в ОДНОЙ из них допущена ошибка.

ИЛИ

– Отсутствует одна из формул, необходимых для решения задачи.

1

Все случаи решения, которые не соответствуют вышеуказанным  критериям  выставления оценок в 1, 2, 3 балла (использование неприменимого закона, отсутствие более одного исходного уравнения, разрозненные записи и т.п.).

0

С2

По пересекающимся под углом α дорогам движутся две автомашины со скоростями υ1 и υ2.  Определите величину и направление скорости одного автомобиля относительно другого. Через какое время после встречи на перекрестке расстояние между машинами будет равно S?

Дано:         

α

S

t - ?

                                                        Рис.1.                                                           Рис.2.

Решение:

Возможен случай движения автомашин, показанный на рисунке 1.

В системе отсчета, связанной с первым автомобилем, первый автомобиль покоится, а второй автомобиль имеет скорость

             (рис.2)

угол между векторами (-υ1) и υ2.  равен

1800 – α                        (рис.2)

Из треугольника скоростей  ,.  и (- ) определим модуль скорости  (рис.2).

– модуль скорости первого автомобиля.

Рассмотрим еще один случай движения двух автомобилей по пересекающимся дорогам (рис.3)

                      Рис3.                                                                          Рис.4.

В этом случае модуль скорости  υотн2  равен (рис.4):

Объединим оба решения в одно:

искомое время t составляет:          .

С3

Два тела брошены вертикально вверх с поверхности Земли из одной точки вслед друг за другом с интервалом времени img34, с одинаковыми начальными скоростями v0. Определить, через какое время тела встретятся.

Решение: Начало отсчета поместим в точку бросания. Ось OY направим вертикально вверх. Отсчет времени начнем с момента бросания первого тела. На рис.7 изображена ось OY и вектора начальной скорости и ускорения свободного падения. http://festival.1september.ru/articles/103842/img41.gif

Рис. 5

В выбранной СО y10 = y20 = 0, v01y = v02y = v0, a1y = a2y = -g. Уравнения кинематики для первого тела имеют вид

img42

Основные уравнения кинематики для второго тела с учетом того, что оно начала свое движение на img34секунд позже первого, запишутся в виде

img43

Тела “встретятся”, когда y1 (t) = y2 (t), т.е. получаем уравнение для нахождения искомого времени “встречи”

img44

откуда

img45

С4

Тело брошено под углом α к горизонту с начальной скоростью  ύ0. Через какие промежутки времени после бросания скорость тела будет составлять с горизонтом углы  β1 = 45°   и β2 =3150

Дано:

α

ύ0

β1 = 45°

β2 =3150

t =?

Рис.6

Разложим скорость  тела  на две составляющие: горизонтальную   и вертикальную (см. рис.6).

Горизонтальная составляющая начальной скорости тела остается при движении тела неизменной по модулю и направлению:

.

Вертикальная составляющая скорости тела изменяется со временем  закону :

Определим угол наклона вектора к горизонту:

Итак, вектор скорости   тела составляет с горизонтом   β1 = 45°   и     β2 =3150

 в моменты времени:    ;      

                                       

С5

Определить радиус R маховика, если при вращении скорость точек на его ободе = 7 м/с, а скорость точек, находящихся на l = 10 см ближе к оси, υ2 = 6,5 м/с. Сколько оборотов делает маховик в минуту?

 

Решение:

Угловая скорость со всех точек, лежащих вдоль радиуса R равномерно вращающегося маховика, является величиной постоянной и равна:

,   (1)

где υ — линейная скорость любой точки, лежащей на радиусе R. Следовательно, для точек А и В справедливо равенство:

;  (2)

, , /

Выражение для R радиуса маховика имеет вид:

.

Связь угловой и линейной скоростей выражается формулой (1). С другой стороны, угловую скорость  можно выразить через число оборотов в единицу времени:

    (3)

Приравняв выражения для  (1) и (3), получим:

.

Ответ:  R = 1,8 м; п = 37 об/м.



Предварительный просмотр:

  • Динамика.

Перечень элементов содержания раздела «Кинематика материальной точки», проверяемых на едином государственном экзамене по физике таков:

Код

раздела

Код

контролируемого

элемента

Элементы содержания,

проверяемые заданиями КИМ

1

МЕХАНИКА

1.2

Динамика

1.2.1

Инерциальные системы отсчета . Первый закон Ньютона

1.2.2

Принцип относительности Галилея

1.2.3

Масса тела

1.2.4

Плотность вещества

1.2.5

Сила

1.2.6

Принцип суперпозиции сил

1.2.7

Второй закон Ньютона

1.2.8

Третий закон Ньютона

1.2.9

Закон всемирного тяготения. Искусственные спутники Земли

1.2.10

Сила тяжести

1.2.11

Вес тела и невесомость

1.2.12

Сила упругости

1.2.13

Сила трения

1.2.14

Давление

Обобщенный план экзаменационной работы ЕГЭ 2011 г. по физике

Обозначение

задания в

работе

Проверяемые элементы

содержания

Коды

элементов

содержания

по кодификатору

элементов

содержания

Уровень

сложности

задания

Макс. балл

за

выполнение

задания

А2

Кинематика,

законы Ньютона

1.1.5.-1.1.8.

1.2.1.- 1.2.8.

Б

1

А3

Силы в природе

1.2.9 - 1.2.13

Б

1

А4

Силы в природе, импульс, закон сохранения импульса

1.2.9 - 1.2.13.

Б

1

А24-25

Механика –

квантовая физика

 (методы познания)

1.1. – 5.3

Б,П

1

В1-В4

Механика –

квантовая физика

 (задачи на соответствие)

1.1. – 5.3

Б,П

2

6

С1-С2

Механика –

квантовая физика

 (расчетные задачи)

1.1. – 5.3

Б,П

3

Особое внимание при обобщении материала по данной теме необходимо обратить внимание на следующие понятия и определения:

1.1  Инерциальные системы отсчета

         1.2  Неинерциальные системы отсчета

         1.3  Скалярные и векторные величины

         1.4  Инерция

         1.5  Инертность

         1.6  Масса

         1.7  Сила

         1.8  Сила всемирного тяготения

         1.9  Сила тяжести

         1.10 Сила трения

                   а) покоя

                   б) скольжения

                   в) качения

                   г) сухое

                   д) жидкое

          1.11  Деформация

                   а) упругая

                   б) неупругая

                   в) сжатие

                   г) растяжение

                   д) сдвиг

                   е) кручение

                   ж) изгиб

           1.12  Сила упругости

           1.13  Ускорение

           1.14 Ускорение свободного падения

           1.15  Первая космическая скорость

           1.16  Сила реакции опоры

           1.17  Равнодействующая всех сил, приложенных к телу                        

           1.18  Вес тела

           1.19  Коэффициент трения

           1.20  Коэффициент жесткости

           1.21  Гравитационная постоянная

           1.22  Центр тяжести

           1.23  Невесомость

           1.24  Перегрузки

           1.25  Материальная точка

           1.26 Проекции вектора на оси координат

           1.27 Плотность

           1.28 Давление

           1.29 Преобразования Галилея

.2.      Формулировки законов и границы их применения

           2.1   Первый закон Ньютона

           2.2   Второй закон Ньютона

           2.3  Третий закон Ньютона

           2.4   Закон всемирного тяготения

           2.5   Закон Гука

3.         Формулы  

            3.1   Второй закон Ньютона

            3.2   Третий закон Ньютона

            3.3   Закон всемирного тяготения

            3.4   Закон Гука

            3.5  Сила трения

            3.6  Сила тяжести

            3.7  Вес тела, движущегося с ускорением

            3.8  Первая космическая скорость

            3.9  Ускорения свободного падения на разных планетах

            3.10 Плотность вещества

            3.11 Давление

Систематизации теоретического материала

Законы динамики

     

На какой вопрос отвечает?

Формула

 Следствия из законов динамики

 Первый закон динамики – закон инерции

 Когда скорость тела не изменяется?    

Если ∑ F= 0, то

α = 0

Пространство и время однородно в инерциальных системах отсчета

Второй закон динамики        

От чего зависит  величина изменение скорости?

F = ∆(mυ)/t

F = ∆p/t

α = F/m

F = mα

F ↑↑ α

Третий закон динамики  

Взаимодействуют друг с другом тела?

 F1 = - F2

Силы возникают парами, приложены к разным телам и  компенсировать друг друга не могут

                   


Границы применимости законов динамики:

  1. Выполняются только в инерциальных системах отсчета
  2. Для тел, размерами которых можно пренебречь, т. е. материальных точек
  3. Массы тел много больше массы элементарных частиц
  4. Скорости материальных точек много меньше скорости света в вакууме

                                            Масса и вес тела

Обозначение

m

P

Единицы измерения в СИ

кг

Н

Скалярная/векторная

величина

скалярная

векторная

Зависимость от скорости

движения тела

Если υ ≤ с, то

m – const

Если υ ≈ с, то

m = m о /√ 1 - (υ∕с)²

Если тело движется по окружности в вертикальной

плоскости  P = m(g ±υ²∕R)

масса

Вес тела

Зависимость от ускорения

движения тела

нет

Если опора движется вместе с телом вертикально P = m(g ±α)

                                       Сила тяжести и вес тела

Сила тяжести

Вес тела

обозначения

Fтяж

P

Единицы измерения в СИ

Н

Н

Природа возникновения

гравитационная

электромагнитная

Скалярная/векторная величина

векторная

векторная

Направление вектора

Вертикально вниз к центру Земли

Перпендикулярно поверхности вниз

От чего зависит значение силы

От расстояния до центра Земли

От характера движения тела

                                  Силы в механике

п/п

Название силы

Закон или формула определения силы

Рисунок

1

Сила всемирного тяготения

F = G · m1· m2 /R²

стр4-1

2    

Сила тяжести

 F = mg

 F = G · M· m /R²

g = G · M  /R²

стр4-2

3

Сила упругости

Fупр =  - kx      

стр4-3

4

Сила трения

 Fтр  = µN      

стр5-1

5

Вес тела

 Р = mg, если опора горизонтальна неподвижна или движется с постоянной скоростью

стр5-3

P = m(g +α), если опора движется с ускорением, направленным вверх.

стр5-4

P = m(g - α), если опора движется с ускорением, направленным вниз.

стр5-5

Р = 0, если опора движется с ускорением направленным вниз и равным α =g

стр5-6

Р = mg cos α, если опора наклонена к горизонту

стр5-7

6

Сила реакции опоры

 N = P

См. рисунок выше

Законы динамики

     

На какой вопрос отвечает?

Формула

 Следствия из законов динамики

 Первый закон динамики – закон инерции

 Когда скорость тела не изменяется?    

Если ∑ F= 0, то

α = 0

Пространство и время однородно в инерциальных системах отсчета

Второй закон динамики        

От чего зависит  величина изменение скорости?

F = ∆(mυ)/t

F = ∆p/t

α = F/m

F = mα

F ↑↑ α

Третий закон динамики  

Взаимодействуют друг с другом тела?

 F1 = - F2

Силы возникают парами, приложены к разным телам и  компенсировать друг друга не могут

                    Границы применимости законов динамики:

  1. Выполняются только в инерциальных системах отсчета
  2. Для тел, размерами которых можно пренебречь, т. е. материальных точек
  3. Массы тел много больше массы элементарных частиц
  4. Скорости материальных точек много меньше скорости света в вакууме

                                            Масса и вес тела

Обозначение

m

P

Единицы измерения в СИ

кг

Н

Скалярная/векторная

величина

скалярная

векторная

Зависимость от скорости

движения тела

Если υ ≤ с, то

m – const

Если υ ≈ с, то

m = m о /√ 1 - (υ∕с)²

Если тело движется по окружности в вертикальной

плоскости  P = m(g ±υ²∕R)

масса

Вес тела

Зависимость от ускорения

движения тела

нет

Если опора движется вместе с телом вертикально P = m(g ±α)

                                       Сила тяжести и вес тела

Сила тяжести

Вес тела

обозначения

Fтяж

P

Единицы измерения в СИ

Н

Н

Природа возникновения

гравитационная

электромагнитная

Скалярная/векторная величина

векторная

векторная

Направление вектора

Вертикально вниз к центру Земли

Перпендикулярно поверхности вниз

От чего зависит значение силы

От расстояния до центра Земли

От характера движения тела

                                  Силы в механике

п/п

Название силы

Закон или формула определения силы

Рисунок

1

Сила всемирного тяготения

F = G · m1· m2 /R²

стр4-1

2    

Сила тяжести

 F = mg

 F = G · M· m /R²

g = G · M  /R²

стр4-2

3

Сила упругости

Fупр =  - kx      

стр4-3

4

Сила трения

 Fтр  = µN      

стр5-1

5

Вес тела

 Р = mg, если опора горизонтальна неподвижна или движется с постоянной скоростью

стр5-3

P = m(g +α), если опора движется с ускорением, направленным вверх.

стр5-4

P = m(g - α), если опора движется с ускорением, направленным вниз.

стр5-5

Р = 0, если опора движется с ускорением направленным вниз и равным α =g

стр5-6

Р = mg cos α, если опора наклонена к горизонту

стр5-7

6

Сила реакции опоры

 N = P

См. рисунок выше

                         

  Эти таблицы могут служить опорой для выполнения заданий базового уровня.

                 Разберем несколько примеров заданий каждого типа

стр7-рис2Примеры заданий  типа А2стр7-рис1

1.   На рис.1 представлены вектор скорости и вектор            

      равнодействующей всех сил, действующих на тело.

      Какой из четырех векторов на рис.2 указывает

     направление вектора ускорения тела в инерциальной

системе отсчета?                                                        

РРис.1                          Рис.2

        1) 1         2) 2       3) 3            4) 4

2. Самолет летит по прямой с постоянной скоростью на высоте 9000 м. Систему отсчета,

   связанную с Землей, считать инерциальной. В этом случае

  1. на самолет не действует сила тяжести
  2. сумма всех сил, действующих на самолет, равна нулю
  3. на самолет не действуют ни какие силы
  4. сила тяжести равна силе Архимеда, действующей на самолет.

3.В инерциальной системе отсчета сила F сообщает телу массой m ускорение α..Как

    надо изменить массу тела, чтобы вдвое меньшая сила сообщила ему в 4 раза  большее  

    ускорение?

          1) оставить неизменной          2) уменьшить в 8 раз   

стр7-рис3          3) уменьшить в 2 раза             4) увеличить в 2 раза                

4.Скорость автомобиля массой 1000 кг, движущегося вдоль оси оХ , изменяется со временем в соответствии с графиком на рис.3 . Систему отсчета считать инерциальной.

    Равнодействующая всех сил, действующих на автомобиль, равна…                                                                                                                    

Рис.3

 1) 500 Н        2) 1000 Н        3) 10 000 Н         4) 20 000 Н                          

стр8-рис4

5.На тело, находящееся на горизонтальной поверхности,

    действует три силы ( рис.4)

     Каков модуль равнодействующей этих сил, если F1 = 1 Н.                

                                                                                                                                                                                            Рис.4

 

1) √10 Н           2) 6 Н           3) 4 Н           4) √13 Н                                        

6.  Земля притягивает к себе подброшенный мяч с силой 5 Н. С какой силой этот мяч притягивает к себе Землю?

     

 1) 50 Н          2) 5 Н          3) 0,5 Н             4) 0,05 Н

стр8-рис57. Брусок лежит на шероховатой наклонной опоре  (рис.5). На него действуют три силы: сила тяжести  mg, сила трения Fтр, сила реакции опоры N. Если брусок покоится, то модуль равнодействующей сил трения Fтр и тяжести mg равен…      

Рис.5

                 1)  N          2) N cosα         3) N sinα          4) mg  + Fтр

                         

                                        Примеры заданий  типа А3

1.Две пружины растягиваются одинаковыми силами.F. . Жесткость первой пружины равна

  k1 в 1,5 раза больше жесткости второй пружины k2. Удлинение второй пружины равно ∆l2, а удлинение первой ∆l1 равно…

      1) 0.5∆l2         2) 0,67∆l2         3) 1.5∆l2          4) 3∆l2

2.  Расстояние между центрами двух шаров равно 1 м, масса каждого 1 кг. Сила

          всемирного тяготения между ними…

     

       1) 1 Н           2) 0,001 Н        3) ≈7·10-5 Н         4) ≈7·10-11 Н

3. Космонавт на Земле притягивается к ней с силой 700 Н. С какой приблизительно силой он будет притягиваться к Марсу, находясь на его поверхности, если радиус Марса в 2 раза, а масса – в 10 раз меньше , чем у Земли?

     1) 70 Н             2) 140 Н         3) 210 Н           4)  280 Н

4.Два маленьких шарика массой m каждый находятся на расстоянии  r друг от друга и притягивают с силой  F .  Какова сила гравитационного притяжения двух других шариков, если масса каждого m ∕3, а расстояние между ними  r∕3?

     

 1) 3 F          2) 1/3 F           3) F             4)  1/27 F

5.Под действием силы 3 Н пружина удлинилась на 4 см. Чему равен модуль силы, под действием которой удлинение  этой пружины составит 6 см?

   

 1) 3.5 Н         2) 4 Н         3)  4,5 Н          4) 5 Н                                                                  

6.Деревянный брусок массой m , площадь граней которого связаны соотношением

    S1 : S2 : S3 = 1 : 2 : 3,  скользит равномерно по горизонтальной шероховатой опоре,    

    прикасаясь  с ней гранью площадью S1,  под действием горизонтальной силы.. Какова  

   величина этой силы , если коэффициент трения скольжения бруска об опору равен μ ?

 

   1)   μmg        2)1/2 μmg        3) 3 μmg        4) 1/6 μmg

7.На какой высоте от поверхности Земли сила притяжения тела к ней будет в 9 раз

   меньше, чем на ее поверхности?

     

 1)   Rз                    2) 2 Rз                 3)  3 Rз             4) 9 Rз

         

                            Примеры заданий  типа А4

     

1.Тело движется по прямой в одном направлении . Под действием постоянной силы за 3 с  

   импульс тела изменился на 6 кг м /с. Каков модуль силы?

1) 0,5 Н             2) 2 Н            3) 9 Н             4) 18 Н

2.Тело движется по прямой. Под действием постоянной силы 4 Н за 2 с импульс тела

   увеличился и стал равным  20 кг м /с. Первоначальный импульс тела был равен…

1) 4 кг м /с           2) 8 кг м /с           3)  12 кг м /с          4) 28 кг м /

3.Тело движется по прямой. Под действием постоянной силы 5 Н. Импульс тела

   уменьшается от 25 кг м/с до 15 кг м /с. Для чего потребовалось время равное …

 

     1) 1с            2) 2 с           3) 3 с          4) 4 с

4.С какой силой давит воздух на поверхность письменного стола , длина которого

   120 см, ширина – 60 см, если атмосферное давление равно  105 Па?

   

 1) 72 ·10-3 Н       2) 105 Н       3) 72 · 103 Н      4) 72 · 107 Н

Примеры заданий  типа А7

     1.На горизонтальном полу стоит ящик массой 10 кг. Коэффициент трения между полом и

   ящиком равен 0,25. К ящику прикладывают в горизонтальном направлении силу 16 Н.

    Какова сила трения между ящиком и полом?

 

      1) 0 Н           2) 2,5 Н        3) 4 Н            4) 16 Н

2.Одинаковые бруски, связанные нитью, движутся под действием силы по гладкой горизонтальной поверхности (см. рис.1). Как изменится сила натяжения нити Т, если

стр9-рис1   третий брусок переложить с первого на второй?                                  

    1) увеличится в 2 раза          2) увеличится в 3 раза

    3) уменьшится в 1,5 раза       4) уменьшится в 2 раза                                                                                              

 

                                                                                                Рис.1

стр9-рис23.К системе из кубика массой 1 кг  и двух пружин приложена горизонтальная сила  F  (рис.2). Между кубиком и опорой трения нет. Система покоится. Жесткость первой пружины 300 Н/м, жесткость второй пружины 600 Н/м. Удлинение  первой пружины равно 2 см. Модуль силы F равен…                                                                                                            

 

1) 6 Н         2) 9 Н        3) 12 Н         4) 18 Н                                    

Рис.2

4.Средняя плотность планеты Плюк равна средне плотности Земли, а радиус планеты

   Плюка в 2 раза больше радиуса Земли. Во сколько раз первая космическая скорость    

   для Плюка больше, чем для Земли ?

             

1) 1            2) 2            3) 1.41           4) 4

5.По горизонтальному столу из состояния покоя движется брусок массой 0,8 кг, соединенный с грузом массой 0,2 кг невесомой нерастяжимой нитью, перекинутой через

стр9-рис3   невесомый блок (см. рис.3).Груз движется с ускорением 1,2 м/с2. Коэффициент трения бруска о поверхность стола равен…                                

     

 1) 0,10            2) 0,13           3) 0,22             4) 0,88          Рис.3

Примеры заданий  типа В1 – В4

1.Установите соответствие между физическими величинами и единицами их измерение в системе СИ..

 

ФИЗИЧЕСКАЯ ВЕЛИЧИНА                                           ЕДИНИЦЫ ИЗМЕРЕНИЯ

            А. масса                                                                           1. секунда  ( с)

            Б. сила                                                                              2. килограмм   ( кг)

            В. вес тела                                                                       3. метр         ( м )

                                                                                                     4. ньютон      ( Н)

                                                                                                     5. грамм           ( г )

         Ответ     2 4 4

2.Установите соответствие между физическими величинами и формулами,  по  которым эти величины определяются.

 

    ФИЗИЧЕСКАЯ ВЕЛИЧИНА                                            ФОРМУЛА                

             А. сила упругости                                                           1. mg

             Б. сила тяжести                                                               2. mα

             В. сила трения скольжения                                           3. μN

                                                                                                      4. G Mm/ R2

                                                                                                      5. k∆l

    Ответ      5 1 3

3.Установите соответствие между физическими величинами и их возможными    

     изменениями, анализируя следующую ситуацию:

                           

а) Тело свободно падает с высоты  Н

       

ФИЗИЧЕСКАЯ ВЕЛИЧИНА                    ИЗМЕНЕНИЕ ФИЗИЧЕСКОЙ ВЕЛИЧИНЫ

           А. сила тяжести                                         1. увеличивается

           Б.  ускорение движения                           2. уменьшается

           В. скорость тела                                       3. не изменяется

         Ответ        3 3 1

                         

б) Тело доставили с поверхности Земли на Луну

       

ФИЗИЧЕСКАЯ ВЕЛИЧИНА                    ИЗМЕНЕНИЕ ФИЗИЧЕСКОЙ ВЕЛИЧИНЫ

           А. сила тяжести                                        1. увеличивается

           Б. масса тела                                             2. уменьшается

           В. вес тела                                                 3. не изменяется

        Ответ      2 3 2

в) На тело массой m, покоящееся на горизонтальной поверхности, стала действовать сила, направленная параллельно поверхности и изменяющаяся cо временем  по закону  F =  Ct, где С – коэффициент пропорциональности

     

 ФИЗИЧЕСКАЯ ВЕЛИЧИНА                    ИЗМЕНЕНИЕ ФИЗИЧЕСКОЙ ВЕЛИЧИНЫ

             А. ускорение                                                         1. увеличивается

             Б.  сила трения скольжения                                2. уменьшается

             В. скорость тела                                                   3. не изменяется

           Ответ    1 3 1

4.Установите соответствие между научными открытия и именами ученых, которым    

      эти  открытия принадлежат

   

       НАУЧНЫЕ ОТКРЫТИЯ                                            ИМЕНА УЧЕНЫХ

            А. закон инерции                                             1.  И. Ньютон

            Б. закон всемирного тяготения                      2. Р. Гук

            В. закон упругой деформации                       3. Архимед

                                                                                      4. Галилей

                                                                                      5. Б. Паскаль

            Ответ  4 1 2

5.Установите соответствие между приборами  и физическими величинами, которые эти

   приборы измеряются .

               

  ПРИБОР                                                             ФИЗИЧЕСКАЯ ВЕЛИЧИНА

               А. динамометр                                                          1. давление

               Б. барометр                                                               2. скорость    

               В. спидометр                                                            3. сила  

                                                                                                  4. путь

                                                                                                  5. ускорение

                 Ответ    3 1 2

6.Установите соответствие между техническими устройствами (приборами) и

    физическими закономерностями, лежащими в основе принципа их действия.

 ТЕХНИЧЕСКОЕ УСТРОЙСТВО                      ФИЗИЧЕСКАЯ ЗАКОНОМЕРНОСТЬ

 А. пружинный динамометр                           1. условие равновесия рычага

 Б, рычажные весы                                          2. зависимость силы упругости от величины

 В.ртутный барометр                                           деформации    

                                                                           3. объемное расширение жидкости при      

                                                                              нагревании

                                                                    4. зависимость гидростатического давления от

                                                                         высоты столба жидкости  

                                                                 5. изменение атмосферного давления с высотой

  Ответ     2 1 4

7.По графику зависимости модуля равнодействующей всех сил, действующих на тело, движущегося прямолинейно, от времени установите соответствие между  характером движения тела и промежутком времени действия силы (рис.1)

стр11                   

                                                           

                                                                                        Рис.1  

     

   ХАРАКТЕР ДВИЖЕНИЯ ТЕЛА                                       ПРОМЕЖУТОК ВРЕМЕНИ  

              А. равномерное движение                                                      1. 0 – t1   

              Б. ускоренное движение                                                         2. t1 – t2

              В. равноускоренное движение                                               3. t2 – t3

                                                                                                                4. t3 – t4

                                                                                                                5. 0 – t4    

       Ответ       4 1 2

                   

    Алгоритм решения задач на законы Ньютона 

1. Внимательно прочитайте условие задачи

2. Сделайте краткую запись условия задачи, выразив все данные величины в

    единицах измерения СИ (все числовые значения следует записывать в

     стандартном виде – произведение десятичной дроби с одной значащей

    цифрой перед запятой на соответствующую степень десяти).

3. Сделайте схематический чертеж (с помощью чертежных инструментов),

    указав: а) все силы, действующие на каждое тело, предварительно указав

   тела, с которыми взаимодействует искомое тело б) направления векторов

   ускорения движения каждого тела, в) все скалярные величины -

    управляющие явлением по условию задачи.  

4. Выберете систему отсчета (одну из осей удобно направить по

    направлению вектора ускорения движения тела).

5. Запишите законы динамики (второй и третий законы Ньютона)  в

    векторной форме для каждого тела.

6. Запишите законы динамики (второй и третий законы Ньютона) в

    проекциях на выбранные оси координат для каждого тела.

7. Запишите недостающие уравнения из кинематики, формулами

    определения сил в механике (при необходимости).

8. Проверьте замкнутость полученной системы уравнений ( число

   неизвестных величин должно равняться числу уравнений в системе

   уравнений).

9. Решите полученную систему уравнений относительно неизвестной

   величины в общем виде, т.е. выразите неизвестную по условию задачи

   величину в буквенных обозначениях величин, заданных в условии задачи.

10. Сделайте работу с размерностью ( проверим единицу измерения

    неизвестной величины, для чего в полученную формулу вместо символов,

    обозначающих физические величины, подставьте их единицы измерения в

   СИ и убедимся, что полученная при этом единица измерения соответствует  

   единице измерения неизвестной величины).

11. Произведите расчеты, подставив в полученную формулу числовые

    значения физических величин в нее входящих.

12. Оцените, где это возможно, правдоподобность числового ответа.

13. Запишите ответ в общем и числовом виде (не забудьте указать единицу

    измерения физической величины).

                                     Примеры заданий  типа С2

Задача1         Материальные точки массами m1=100г и m2= 200г прикреплены      

                  к невесомому стержню, как показано на рисунке. К точке  m2 

                  прикреплена невесомая пружина жесткостью  k = 30 Н/м ,    

                  верхний конец  которой закреплен. Длина пружины в

                  недеформированном состоянии l0 = 20 см. В начальный момент

                  концы пружины связаны нитью длиной l = 10 см. Определить

                  силу реакции стержня, действующую на массу сразу после

                   пережигания нити.

стр12Дано                                                              Решение                                        

 m1=100г                                          1. Точка m2  взаимодействует с телами:

m2= 200г                                          пружиной, точкой m1   и Землей. На нее    

k = 30 Н/м                                        действуют силы: сила упругости Fупр ↓, сила

l0 = 20 см                                          реакции со стороны стержня Т2↑   и сила

l = 10 см                                           тяжести  m2g↓.

-------------                                            Точка m1  взаимодействует с телами:

Найти Т1                                          точкой m2   и Землей. На нее действуют

                                                          силы: сила реакции со стороны стержня 

                                                                                           Т1↓ и сила тяжести  m1g↓

    В момент пережигания нити тела будут двигаться с ускорением, направленным вниз.

2.Выбираем систему отсчета относительно Земли,  ось ординат которой направлена вниз.

3. Запишем второй закон Ньютона в векторной форме для каждого тела.

           ∑F1 = m1α1                                     ∑F2 = m2α2       

           ∑F1 = Т1+ m1g                                ∑F2 = Fупр + Т2 + m2g

4. Запишем второй закон Ньютона в проекции на ось оY для каждого тела

оY:    Т1+ m1g  = m1α1                                         (1)                                                  

          Fупр -  Т2 + m2g  = m2α2                              (2)

5.Допишем недостающие уравнения

 Fупр =  k(l0 –l)  -   закон Гука                              (3)

  Т1 = Т2  = Т          -    третий закон Ньютона      (4)

   α1= α2  = α  -   жестко связанные тела             (5)

6.Решаем полученную систему уравнений относительно неизвестной  

    величины:  

   Т + m1g  = m1α                                          (6)                

   k(l0 –l) - Т + m2g  = m2α                            (7)

Сложив уравнения 6 и 7, получим уравнение, содержащее одну неизвестную величину α .                                                  

   k(l0 –l) + m1g  + m2g =  m1α + m2α              (8)

Выразим из полученного уравнения 8 ускорение

     α =  g +  k(l0 –l) ∕( m1  + m2)                               (9)  

 Полученное выражение 9 подставим в уравнение 6 и выразим неизвестную величину Т,  которую нужно определить по условию задачи.  

     Т =  m1(α – g) =  m1(g  +   k(l0 –l) ∕( m1  + m2) – g ) =  m1 k(l0 –l) ∕( m1  + m2)

    T  =  m1 k(l0 –l) ∕( m1  + m2)

 7. Проверим размерность полученной величины 

 [ Т ] = [ кг· Н/м ·м · 1/кг = Н ]

8.Сделаем расчеты

   Т =  0,1· 30·(0.2 – 0,1)/ (0,1 + 0.2) = 1 (Н)

9. Ответ   Т= 1Н

           

                   Критерии оценки выполнения задания

Баллы

 Приведено полное правильное решение, включающее следующие элементы:

  1. Верно записаны формулы, выражающие физические законы, применению которых необходимо для решения задачи выбранным способом ( второй закон Ньютона, закон Гука)
  2. проведены необходимые математические преобразования и расчеты, приводящие к правильному числовому ответу, и представлен ответ.

 При этом допускается решение «по частям»( с промежуточными вычислениями)

3

Представлено правильное решение только в общем виде, без каких-либо расчетов.

                                   ИЛИ

Правильно записаны необходимые формулы, записан правильный ответ, но не представлены преобразования, приводящие к ответу.

                                    ИЛИ

В математических преобразованиях или вычислениях допущена ошибка, которая привела к неверному ответу.

2

В решении содержится ошибка в необходимых математических преобразованиях и отсутствуют какие-либо числовые расчеты

                                    ИЛИ

Записаны все исходные формулы, необходимые для решения задачи, но в ОДНОЙ из них допущена ошибка.

                                   ИЛИ

Отсутствует одна из формул, необходимых для решения задачи.

1

Все случаи решения, которые не соответствуют вышеуказанным критериям выставления оценок в 1,2,3 балла (использование неприменимого закона, отсутствие более одного исходного уравнения, разрозненные записи и т.д.)

0

                              

Задача 2    Как изменится сила натяжения нити, связывающей два тела одинаковой массы m, если дополнительный    груз массой М переложить с первого тела на второй. Сила тяги F и ускорение движения тел остаются    

постоянными. Трением между телами и поверхностью можно пренебречь. Нить невесома и нерастяжима.

Ответ:        T1 / Т2   = m/(m +M) 

  Сила натяжения нити при переносе груза с первого тела на второе увеличивается в (1 + M/m) раз.

   Задача3 При падении тела с большой высоты в воздухе через некоторый

 промежуток времени его скорость становится постоянной. Учитывая, что сила лобового сопротивления прямо пропорциональна площади поперечного сечения тела и квадрату его скорости, определите отношение установившихся скоростей      

 υRr для двух шаров радиусами R  и r. Шары изготовлены из одного и того же материала.

        Ответ            υRr  = √R/r

Задача 4  Два одинаковых шарика связаны невесомой нитью, перекинуто через невесомый блок, причем один из шариков погружен в сосуд с жидкостью. С какой установившейся скоростью U будут двигаться шарики, если известно, что установившаяся скорость падения одиночного шарика в той же жидкости равна  U0 . Сила сопротивления пропорциональна скорости. Плотность жидкости ρж.

                    а плотность материала шарика равна ρш.

Ответ:     U= ρo Uo/ (ρо – ρ).

Задача 5  На горизонтальном диске, который может вращаться вокруг    

 вертикальной оси, находится тело массой  m . Расстояние тела от оси вращения  R. Коэффициент трения тела о диск равен  µ. Диск начинают медленно раскручивать оси. Построит график зависимости силы трения, действующей на тело, от угловой скорости вращения  ω.

 Ответ      ωо =  ( µg/R)½     

                                   



Предварительный просмотр:

  • Законы сохранения в механике.

Перечень элементов содержания раздела «Законы сохранения в механике», проверяемых на едином государственном экзамене по физике таков:

Код

раздела

Код

контролируемого

элемента

Элементы содержания,

проверяемые заданиями КИМ

1

МЕХАНИКА

1.4

Законы сохранения в механике

1.4.1

Импульс тела

1.4.2

Импульс системы тел

1.4.3

Закон сохранения импульса

1.4.4

Работа силы

1.4.5

Мощность

1.4.6

Работа как мера изменения энергии

1.4.7

Кинетическая энергия

1.4.8

Потенциальная энергия

1.4.9.

Закон сохранения механической энергии

Обобщенный план экзаменационной работы ЕГЭ 2011 г. по физике

Обозначение

задания в

работе

Проверяемые элементы

содержания

Коды

элементов

содержания

по кодификатору

элементов

содержания

Уровень

сложности

задания

Макс. балл

за

выполнение

задания

1

А4

Силы в природе, импульс, закон сохранения импульса.

1.2.9-1.2.13,  1.4.1-1.4.2

Б

1

2

А5

Механическая энергия, работа, закон сохранения энергии

1.4.4.-1.4.9.

1.2.1.- 1.2.8.

Б

1

3

А7

Механика

1.1. - 1.5

П

1

4

А24-25

Механика –

квантовая физика

 (методы познания)

1.1. – 5.3

Б,П

1

5

В1-В4

Механика –

квантовая физика

 (задачи на соответствие)

1.1. – 5.3

Б,П

2

6

С1-С2

Механика –

квантовая физика

 (расчетные задачи)

1.1. – 5.3

В,П

3

Особое внимание при обобщении материала по данной теме необходимо обратить внимание на следующие понятия, определения и закономерности:

  • импульс тела;
  • импульс силы;
  • импульс системы тел;
  • работа;
  • мощность;
  • работа как мера изменения энергии;
  • кинетическая энергия;
  • потенциальная энергия;
  • закон сохранения импульса;
  • закон сохранения энергии;
  • переход одного вида энергии в другой;
  • принцип суперпозиции и относительности;
  • работа с векторами.

На схемах показано, что необходимо знать по данной теме.

 

                                  ИМПУЛЬС ТЕЛА

МОДУЛЬ                        НАПРАВЛЕНИЕ                   ЕДИНИЦА

                    ЗАКОН СОХРАНЕНИЯ ИМПУЛЬСА

                                   МЕХАНИЧЕСКАЯ РАБОТА

УСЛОВИЯ                                                                                         ГРАФИЧЕСКАЯ

СОВЕРШЕНИЯ         ФОРМУЛА               ЕДИНИЦА         ИНТЕРПРЕТАЦИЯ

МОЩНОСТЬ       КПД     ТЕОРЕМА О КИНЕТИ-     ТЕОРЕМА О ПОТЕНЦИ-

                                              ЧЕСКОЙ ЭНЕРГИИ          АЛЬНОЙ ЭНЕРГИИ

ФОРМУЛА                ЕДИНИЦА               ФОРМУЛА ПРИ  v=const

     

 ОБЩИЙ ЗАКОН СОХРАНЕНИЯ И ПРЕВРАЩЕНИЯ ЭНЕРГИИ

ВНУТРЕННЯЯ                         ЭНЕРГИЯ                             ЕДИНИЦЫ

ПОТЕНЦИАЛЬНАЯ            МЕХАНИЧЕСКАЯ               КИНЕТИЧЕСКАЯ

ЭНЕРГИЯ                                                                               ЭНЕРГИЯ

ПОДНЯТОГО ТЕЛА             ДЕФОРМИРОВАННОЙ ПРУЖИНЫ

ФОРМУЛА                             ФОРМУЛА                                      ФОРМУЛА

            ЗАКОН СОХРАНЕНИЯ МЕХАНИЧЕСКОЙ ЭНЕРГИИ

При повторении теории можно составить следующие опорные конспекты:

I.закон сохранения импульса

1. Закон сохранения импульса

         v1                 v2                     

                                                         

      m1             m2                             

   F1                       F2                              

                                                         

                                                                 

         u1                u2                         

 

      m1            m2

                                                                                                                         

             

                                импульс силы              изменение импульса тела

         

Геометрическая сумма импульсов тел замкнутой системы не изменяется со временем.

2.Когда можно применять закон сохранения импульса

-в замкнутой системе всегда

-в незамкнутой системе (действуют внешние силы) в случаях:

1)внешние силы уравновешиваются (н-р:N и mg)

2)внешние силы малы по сравнению с внутренними

3)внешние силы по искомому направлению отсутствуют

                             

                              например:                                                 x

                                                                        v

                                                          mg   

4) внешние силы велики, но tвзаим→0 (взрывы, выстрелы, удары…)

3.Реактивное движение

II. механическая работа и мощность.

1. Формула. Единица измерения

                               F                      

                                α       x

Если: α=0 →

          α=90° →

2. Графическое изображение работы

F

                                        F - const

                                             ч.

                                         

                                   S

                                                   

                                                   

                                                     

                                                 

                                                     

                                                         

                ∆S                   S

                                                ч.

                                         

                                                   ч.

                                         

                                             ч.

                                         

3. Работа силы тяжести

                         A

                                                                     

                                                           

                                      h                         

                                           

                                                     Amg не зависит от формы траектории

                                     

C                                   B

     

4. Работа силы трения

                    N

Fтр                                                     F

                  mg 

5. Работа силы упругости

                x2    ∆x      x1

                                                    x                                              ч.

Fy2                                           

Fy1                                                  

  Fy                                                     

6. Мощность – работа, совершенная в единицу времени

III

1. Энергия – величина, характеризующая способность тела или системы тел совершать механическую работу

2. Потенциальная энергия – энергия взаимодействия

а) поднятого тела

                             

               h

б) упруго-деформированного тела

                                x

в)связь A и ∆En

      h1   

     h2    

                                           

Итак :

3. Кинетическая энергия – энергия движения

    N

                   F            S

          mg           a

Если

4. Закон сохранения энергии

  h

 - з.с.э. для незамкн. системы

                   

Если                                  →з.с.э. для замкнут. системы.

                   

Рассмотрим некоторые задачи базового уровня по данной теме.

Часть А. Импульс тела.

Вариант 1

  1. Автомобиль массой 1 т двигается прямолинейно со скоростью 20 м/с. Импульс автомобиля равен

1)                                        2)

3)                                          4)

  1. Скорость движущейся материальной точки увеличивается за некоторое время в 4 раза, а ее импульс за это время увеличивается в

1)2 раза           2)4 раза           3)8 раз           4)16 раз

  1. Автомобиль, первоначально двигавшийся со скоростью 20 м/с, после выключения двигателя остановился через 3 секунды. Сила сопротивления, действующая на автомобиль при торможении, равна 6000 H. Масса автомобиля равна

1)600 кг               2)700 кг               3)800 кг               4)900 кг

  1. Два шарика массой 200 г и 100 г движутся со скоростью 2 м/с и 3 м/с соответственно. Направления движения шаров составляют друг с другом угол 90°. Модуль суммарного импульса шариков равен

1)0,5 кг·м/с                                            2)5 кг·м/с

3)0,7 кг·м/с                                            4)7 кг·м/с

  1. Летевший горизонтально со скоростью v пластилиновый шарик массой m ударяется о вертикальную стену и прилипает к ней. Время удара t. Чему равен модуль средней силы, действующей на стенку во время удара?

1)-2 mv/t            2)0              3)mv/t            4)2 mv/t

  1. Тело массой m брошено с горизонтальной поверхности со скоростью v0 под углом α к горизонту. Если пренебречь сопротивлением воздуха, то модуль изменения импульса тела за время полета равен

1)0                2)2 mv0                3)2 mv0·cosα                 4)2 mv0·sinα

Вариант 2

  1. Первая формулировка закона сохранения импульса принадлежит

1)Галилею                   2)Ньютону

3)Декарту                    4)Гуку

2.        Какая из перечисленных ниже величин является векторной?

        1)масса               2)путь              3)импульс             4)время

3.        Импульс первой материальной точки равен , вторая материальная точка имеет импульс . Полный импульс системы двух материальных точек равен

1)P1+P2               2)P1-P2              3) +              4) -

4.        Материальная точка двигалась по прямой и под действием единственной силы в 20 H изменила свой импульс на 40 кг·м/с. За какое время это произошло?

        1)0,5 с              2)5 с              3)2 с              4)0,2 с

5.        Материальная точка массой m равномерно движется по окружности радиуса R со скоростью v. Чему равен модуль изменения импульса за половину периода?

        1)0                 2)mv               3)2 mv                4)2 mv/R

6.        Тело массой m, брошенной горизонтально с некоторой высоты с начальной скоростью V0, через время t упало на землю. Чему равен модуль изменения импульса тела за время полета? (Сопротивлением воздуха пренебречь.)

        1)0                2)mV0              3)2 mV0               4)mgt

Закон сохранения импульса

Вариант 1

  1. Закон сохранения импульса выполняется только

1)во внешнем поле силы

2)в замкнутой системе тел

3)в неинерциальной системе отсчета

4)при отсутствии силы трения

2.        На тележку массой 20 кг, двигающуюся равномерно по горизонтальной поверхности со скоростью 0,1 м/с, с небольшой высоты осторожно опускают кирпич массой 5 кг. Какой становится скорость тележки?

        1)0,12 м/с         2)0,15 м/с          3)0,08 м/с          4)0,05 м/с

3.        Снаряд, летящий горизонтально со скоростью 200 м/с, разрывается на два одинаковых осколка, один из которых летит назад со скоростью 200 м/с. С какой скоростью летит второй осколок?

        1)200 м/с           2)400 м/с           3)600 м/с          4)800 м/с

4.        Два тела, двигаясь навстречу друг другу со скоростью 3 м/с каждое, после соударения стали двигаться вместе со скоростью 1 м/с. Отношение масс этих тел равно

        1)2                     2)3                3)4                   4)5

5.        С поверхности Земли произведен выстрел под углом к горизонту. Снаряд разрывается в верхней точке траектории на два осколка равной массы, причем один их них летит назад по первоначальной траектории снаряда. Где упадет второй осколок?

        1)в точке, где упал бы снаряд

        2)ближе к месту выстрела, чем точка, где упал бы снаряд

        3)дальше от места выстрела, чем точка, где упал бы снаряд

        4)ближе или дальше точки, где упал бы снаряд, в зависимости от начальных условий

6.        На одном конце неподвижной длинной тележки массой m1 стоит мальчик массой m2. С какой по модулю скоростью будет двигаться тележка, если мальчик побежит со скоростью V относительно тележки?

        1)              2)              3)                4)

Вариант 2

  1. Какое из выражений соответствует закону сохранения импульса для случая взаимодействия двух тел?

1)

2)

3)

4)

2.        Железнодорожный вагон массой m, движущийся со скоростью V, сталкивается с неподвижным вагоном массой 2m и сцепляется с ним. Каким суммарным по модулю импульсом обладают два выгона после столкновения?

        1)0               2)mV              3)2 mV             4)3 mV

3.        Тележка массой 2 кг, движущаяся со скоростью 3 м/с, сталкивается с неподвижной тележкой массой 4 кг и сцепляется с ней. Чему равна скорость обеих тележек после взаимодействия?

        1)0,5 м/с              2)1 м/с               3)1,5 м/с                    4)3 м/с

4.        При выстреле из пистолета вылетает пуля массой m со скоростью V. Какой по модулю импульс приобретает в результате выстрела пистолет, если его масса в 100 раз больше массы пули?

        1)0                        2)mV/100                3)mV                      4)100 mV

5.        При выстреле из пистолета вылетает пуля массой m со скоростью V. Какую по модулю скорость приобретает после выстрела пистолет, если его масса в 100 раз больше массы пули?

        1)0                         2)V/100                  3)V                       4)100 V

6.        Снаряд, летящий горизонтально со скоростью V, разрывается на два осколка одинаковой массы. Один осколок летит вертикально вниз со скоростью 2V. Под каким углом к горизонтали летит второй осколок?

        1)0°                          2)30°                       3)45°                       4)60°

Механическая работа

Вариант 1

  1. Какое из приведенных значений может выражать механическую работу?

1)5 H·c             2)-10 кг·м/с2              3)-5 кг м22               4)10 H/м

2.        Значение работы силы при прямолинейном движении тела зависит

        1)только от перемещения тела

        2)только от угла между направлениями силы и скорости тела

        3)только от модуля силы, действующей на тело

        4)от всех перечисленных выше параметров

3.        Работа силы равна нулю, если угол между направлением вектора силы и направлением вектора перемещения равен

        1)0°             2)45°            3)90°            4)180°

4.        Камень массой 0,3 кг, брошенный вертикально вверх с некоторой начальной скоростью, достигнув высоты 3 м, упал обратно. Работа силы тяжести за все время полета равна ( принять g=10 м/с2.)

        1)0 Дж                2) 0,9 Дж              3)9 Дж             4)18 Дж

5.        Спутник массой m движется по круговой орбите радиуса R вокруг Земли. (Масса Земли M, гравитационная постоянная G) Работа силы притяжения к Земле, действующей на спутник за половину оборота, равна

        1)

6.        Брусок равномерно скользит вниз по наклонной плоскости. На некотором пути сила тяжести совершает работу A1, сила трения совершает работу A2. Между этими величинами справедливо соотношение

        1)A1=-A2            2)A1=A2          3)A2=2A1             4)A1=2A2

Вариант 2

  1. Какое из приведенных выражений не является наименованием единицы работы?

1)1 кг·м22          2)1 Дж            3)1 H·м            4)1 H·с

2.        По какой формуле следует рассчитывать работу силы , если между направлениями силы и перемещения угол α?

        1)

3.        На рисунках представлены три варианта взаимного расположения векторов силы , действующей на тело, и скорости  тела при прямолинейном движении. В каком случае работа силы  отрицательна?

        1)                                          2)                                      3)                   

                                                                                                                                

                                                                                                                                     

                                                                                                                               

        4)ни в одном из случаев

4.        Лыжник может спуститься с горы от точки M до точки N по одной из траекторий, представленных на рисунке. При движении по какой траектории работа силы тяжести будет иметь максимальное по модулю значение?

                M                                                                          

 g                           2               1                                                

                                3                                                          

                                                  N                            

        1)1        2)2          3)3         4)по всем траекториям работа силы тяжести одинакова

5.        Тело массой 1 кг силой 30 H поднимается на высоту 5 м. Чему равна работа этой силы?

        1)0 Дж          2)50 Дж           3)100 Дж           4)150 Дж

6.        Работа каких перечисленных ниже сил:

        1)силы тяготения

        2)силы упругости

        3)силы трения

зависит от формы пути?

        1)только 1         2)только 2                3) только 3             4)1, 2 и 3

Кинетическая и потенциальная энергия

Вариант 1

  1. Для замкнутой системы тел в любой инерциальной системе отсчета является одинаковой

1)только кинетическая энергия

2)только потенциальная энергия

3)и кинетическая, и потенциальная энергия

4)ни кинетическая, ни потенциальная энергия

2.        Векторной величиной является

        1)кинетическая энергия                      2)потенциальная энергия

        3)механическая работа                       4)импульс

3.        Автомобиль массой 0,5 т, движущийся со скорость 72 км/ч, обладает кинетической энергией, равной

        1)100 Дж           2)10 кДж            3)100кДж              4)1 МДж

4.        Камень массой m падает с высоты h на горизонтальную поверхность. Какое из следующих утверждений является наиболее общим? В момент касания горизонтальной поверхности потенциальная энергия камня

        1)равна 0

        2)равна - m·g·h

        3)меньше на m·g·h, чем на высоте h

        4)больше на m·g·h, чем на высоте h

5.        В начальном положении пружина была сжата на 2 см. Из этого положения ее растянули на 4 см. После растяжения ее потенциальная энергия по сравнению с начальным положением

        1)увеличилась

        2)уменьшилась

        3)не изменилась

        4)увеличилась или уменьшилась в зависимости от жесткости пружины

6.        Тело, находящееся на горизонтальной поверхности, брошено под углом к горизонту. Изменение его потенциальной энергии Ep в зависимости от высоты h изображено на графике

     1)                       2)                         3)                          4)                                            

     Ep                        Ep                         Ep                               Ep                                     

                                                                                                                                   

                                                                                                                                 

       0                 h     0                h      0                h          0                   h                  

Вариант 2

  1. Единица энергии может быть выражена как

1)1 кг м/с                                    2)1 кг м/с2

3)1 кг м22                                  4)1 кг м23

2.        Какая из приведенных ниже формул определяет потенциальную энергию деформированной пружины?

        1)

3.        Сумма потенциальной и кинетической энергии движущегося тела

        1)всегда >0

        2)всегда = 0

        3)всегда <0

        4)может принять как положительное, так и отрицательное значение

4.        Кинетическая энергия тела 20 Дж, а импульс тела 10 кг м/с. Скорость тела равна

        1)2 м/с           2)4 м/с            3)5 м/с              4)10 м/с

5.        Тело, находящееся на горизонтальной поверхности, брошено вертикально вверх. Какой из графиков правильно отражает независимость кинетической энергии тела от квадрата его скорости  V2? (Сопротивление воздуха пренебрежимо мало.)

     1)                       2)                         3)                          4)                                            

     Ek                             Ek                     Ek                           Ek                                     

                                                                                                                                   

                                                                                                                                 

       0                  V2    0              V2      0               V2       0                  V2                   

6.        Тело массой 0,5 кг падает с высоты 10 м с некоторой начальной скоростью. Чему равно изменение кинетической энергии за время падения (принять g=10м/с2)?

        1)0                 2)5 Дж              3)10 Дж            4)50 Дж

Законы сохранения энергии и импульса

Вариант 1

  1. Наибольшие заслуги в открытии закона сохранения и превращения энергии принадлежат ученым

1)Гюйгенсу и Лейбницу

2)Максвеллу и Фарадею

3)Галилею и Ньютону

4)Майеру, Джоулю и Гельмгольцу

2.        В замкнутой системе, в которой действуют только силы тяготения и силы упругости,

        1)сохраняется только импульс

        2)сохраняется только механическая энергия

        3)сохраняется импульс и механическая энергия

        4)не сохраняются ни импульс, ни механическая энергия

3.        С тележки, груженной кирпичом и движущейся горизонтально, упал случайно кирпич. Кинетическая энергия тележки (без учета упавшего кирпича) при этом

        1)увеличится             2)не изменится           3)уменьшится

        4)увеличится или уменьшится в зависимости от массы кирпича

4.        Конькобежец массой 50 кг бросает горизонтально кирпич массой 5 кг со скоростью 1 м/с. При этом конькобежец приобретает кинетическую энергию

        1)25 Дж             2)5 Дж              3)2,5 Дж              4)0,25 Дж

5.        Движущийся шар сталкивается с таким же покоящимся. После столкновения шары движутся вместе. В результате кинетическая энергия системы шаров

        1)уменьшается             2)увеличивается            3)не изменяется

        4)нельзя сказать, увеличивается или уменьшается

6.        На двух нитях, прикрепленных в одной точке, подвешены два пластилиновых шарика одинаковой массы. Длина каждой нити L. Один шарик отводят до высоты h

        1)большей h                                                                                                      

        2)меньшей h                                                                                                      

        3)равной h                                                                                                        

        4)равной L                                                                                                          

                                                                                                   h

Вариант 2

  1. Мяч, брошенный вертикально вниз с высоты 5 м, после удара о землю подпрыгнул на высоту 5 м. При ударе сохранилась полная механическая энергия. Начальная скорость мяча была равна

1)0 м/с             2)10 м/с            3)20 м/с             4)30 м/с

2.        Шарик, подвешенный на нити, совершает свободные колебания в вертикальной плоскости. Сопротивление воздуха и трение пренебрежимо малы. На всем протяжении движения шарика

        1)сохраняется импульс шарика

        2)сохраняется кинетическая энергия

        3)сохраняется потенциальная энергия

        4)никакая из перечисленных величин

3.        В любой замкнутой системе тел сохраняется

        1)полная механическая энергия системы

        2)суммарная потенциальная энергия системы

        3)скорость каждого тела

        4)импульс системы

4.        Шар массой m со скоростью 2V навстречу другому, масса которого 2m, а скорость V. Модуль изменения кинетической энергии системы после соударения, в результате которого шары движутся как единое целое, равен

        1)0             2)mV2              3)2mV2             4)3mV2

5.        Бруску массой m сообщают скорость , направленную вверх вдоль наклонной плоскости. Брусок поднимается по наклонной плоскости до высоты h. Работа сил трения на этом участке равна

        1)                                                                                                         

        2)                                                            h                                        

        3)                                                                                                   

        4)

6.        После столкновения два пластилиновых шара слипаются и движутся как единое целое. При этом их суммарная кинетическая энергия уменьшается в 4 раза. Величина суммарного импульса этих шаров после столкновения

        1)не изменяется                             2)уменьшается в 2 раза

        3)уменьшается в 4 раза                4)увеличивается в 4 раза

Закон сохранения механической энергии

Вариант 1

  1. На каком этапе движении спутника его потенциальная энергия не изменяется?

1)при разгоне с помощью ракеты

2)при движении по круговой орбите

3)при переходе с орбиты одного радиуса на другую

4)при спуске на Землю

2.        Тело брошено вертикально вверх со скоростью 20 м/с. Сопротивление воздуха пренебрежимо мало. Какой высоты оно достигает?

        1)20м               2)10м               3)5м                 4)200м

3.        При взрыве снаряда массой m, летевшего горизонтально со скоростью V, образовалось 10 осколков. Их суммарная кинетическая энергия сразу после взрыва

        1)равна 0

        2)всегда

        3)больше или равна

         4)меньше или равна

4.        Камень массой 0,2 кг брошен вертикально вверх со скоростью 10 м/с и упал в том же месте со скоростью 8 м/с. Работа сил сопротивления воздуха за время движения камня равна

        1)1,8 Дж              2)-3,6 Дж             3)-18 Дж              4)36 Дж

5.        Камень массой m брошен горизонтально с башни высоты H с начальной кинетической энергией Ek. Как меняется потенциальная энергия камня в зависимости от высоты h? Высота h отсчитывается от основания башни

        1)

        2)

        3)

        4)

6.        Шарик массой m подвешивают к нерастянутой пружине жесткости k и резко отпускают. Максимально растяжение пружины при этом равно

        1)                 2)               3)                4)

Вариант 2

  1. Первая формулировка закона сохранения механической энергии принадлежит

1)Лейбницу           2)Ньютону          3)Декарту           4)Гюйгенсу

2.        Тело массой 1 кг свободно падает с высоты 5 м. Начальная скорость тела равна нулю. На расстоянии 2 м от поверхности Земли кинетическая энергия тела приблизительно равна (принять g=10 м/с2).

        1)0 Дж           2)10 Дж         3)20 Дж            4)30 Дж

3.        Брусок толкнули вдоль горизонтальной поверхности, и он под действием только силы трения проходит до остановки путь 0,5 м. Коэффициент трения между бруском и плоскостью равен 0,1. Начальная скорость бруска приблизительно равна

        1)0,5 м/с            2)1 м/с            3)1,5 м/с             4)2 м/с

4.        На рисунке изображена траектория движения тела, брошенного под углом к горизонту. В какой из точек траектории, изображенных на рисунке, кинетическая энергия тела имеет минимальное значение? (Сопротивление воздуха пренебрежимо мало.)

        1)в т.1      2)в т.2        3)в т.3       4) во всех точках одинакова

                                                 2                                  

                  1                                                                        

                                                                                 3

5.        При столкновении двух тел сохраняется полная механическая энергия системы. После столкновения тел E1=15 Дж и E2=25 Дж. До соударения полная механическая энергия первого тела E1=5 Дж, а полная механическая энергия второго тела E2 равна

        1)5 Дж            2)15 Дж              3)25 Дж            4)35 Дж

6.        Полная механическая энергия замкнутой системы тел не сохраняется, если тела взаимодействуют

        1)силами упругости

        2)силами трения

        3)силами тяготения

        4)одновременно силами упругости и тяготения

Часть В.

Задачи на соответствие условно можно разделить на следующие группы:

физическая величина

- изменение величины

физический закон

- формулировка

физическая величина

- графики

физическая величина

- формулы

физическая величина

- определение

физическая величина

- единицы измерения

1.

Брусок скользит вниз по наклонной плоскости без трения. Что происходит при этом с его скоростью, потенциальной энергией, силой реакции наклонной плоскости? Поверхность земли принять за нулевой уровень потенциальной энергии.

Для каждой величины определите соответствующий характер изменения:

1) увеличилась

2) уменьшилась

3) не изменилась

Запишите в таблицу выбранные цифры для каждой физической величины. Цифры в ответе могут повторяться.

Скорость бруска

Потенциальная энергия бруска

Сила реакции наклонной плоскости

2.

Частица массой m, имеющая заряд q, движется в однородном магнитном поле с индукцией B по окружности радиуса R со скоростью v. Что произойдет с радиусом орбиты, периодом обращения и кинетической энергией частицы при увеличении скорости движения?

Для каждой величины определите соответствующий характер ее изменения:

1) увеличилась

2) уменьшилась

3) не изменилась

Запишите в таблицу выбранные цифры для каждой физической величины. Цифры в ответе могут повторяться.

Радиус орбиты частицы

Период обращения частицы

Кинетическая энергия частицы

Получившуюся последовательность цифр перенесите в бланк ответов (без пробелов и каких-либо символов).

3.

Нитяной маятник отклонили от положения

равновесия и отпустили (см.рис.). Графики

А и Б представляют изменения физических

величин, характеризующих движение шарика

в процессе колебаний. Установите соответствие

между графиками и физическими величинами,

зависимости которых от времени эти графики

могут представлять.                                                            x

К каждой позиции первого столбца подберите

соответствующую позицию второго и запишите в таблицу выбранные цифры под соответствующими буквами. Цифры в ответе могут повторяться.

ГРАФИКИ                                                        ФИЗИЧЕСКИЕ ВЕЛИЧИНЫ

А)

                                                                          1) проекция скорости vx

 

                                                                          2) координата x

 

                                                                          3) потенциальная энергия Ep

0                  T/2                       T     t 

                                                                           4) кинетическая энергия Ek

 

 

Б)

 

 

 

 

 

 

0                                                        t

                      T/2                  T        

 

 Ответ:

А

Б

Получившуюся последовательность цифр перенесите в бланк ответов ( без пробелов и каких-либо символов).

4.

В школьной лаборатории изучают колебании пружинного маятника при различных значениях массы маятника. Если увеличить (уменьшить) массу маятника, то как изменятся: период колебаний, частота колебаний, период изменения потенциальной энергии?  

К каждой позиции первого столбца подберите нужную позицию второго и запишите в таблицу выбранные цифры под соответствующими буквами.

ФИЗИЧЕСКИЕ ВЕЛИЧИНЫ

ИХ ИЗМЕНЕНИЯ

А)

Период колебаний

1)

Увеличится

Б)

Частота колебаний

2)

Уменьшится

В)

Период изменения потенциальной энергии

3)

Не изменится

А

Б

В

5.

Одна шайба скользит по горизонтальной поверхности, а другая такая же – покоится. Как изменяется кинетическая энергия первой шайбы и их общая механическая энергия в результате абсолютно упругого столкновения шайб?

К каждой позиции первого столбца подберите нужную позицию второго и запишите в таблицу выбранные цифры под соответствующими буквами.

ФИЗИЧЕСКИЕ  ВЕЛИЧИНЫ

ИХ ИЗМЕНЕНИЕ

А)

кинетическая энергия 1-й шайбы

1)

увеличивается

Б)

их общая механическая энергия

2)

уменьшается

3)

не изменяется

А

Б

6.

Шайба массой m съезжает без трения с горки высотой h из состояния покоя. Ускорение свободного падения равно g. Чему равны модуль импульса шайбы и ее кинетическая энергия у подножия горки?

Установите соответствие между физическими величинами  и формулами, по которым их можно рассчитать. К каждой позиции первого столбца подберите соответствующую позицию второго и запишите в таблицу выбранные цифры под соответствующими буквами.

ФИЗИЧЕСКАЯ ВЕЛИЧИНА

ФОРМУЛА

А)        Модуль импульса шайбы

1)        

Б)        Кинетическая энергия шайбы        

2)        

3)        

4)        

        

А

Б

7.

Тело брошено под углом к горизонту. Как во время полета будут изменяться физические величины, перечисленные в первом  столбце? Влиянием  воздуха можно пренебречь.

        

ФИЗИЧЕСКИЕ  ВЕЛИЧИНЫ

ИХ ИЗМЕНЕНИЕ

А) Потенциальная энергия  тела

1) сначала увеличивается, а

потом уменьшается

Б) Модуль вектора скорости  тела

2) сначала уменьшается, а

потом увеличивается

В) Направление вектора ускорения тела

3) не изменяется

А

Б

В

8.

Подвешенный на пружине груз совершает вынужденные гармонические колебания под действием силы, меняющейся с частотой ν. Установите соответствие между физическими величинами этого процесса и частотой их изменения.

ФИЗИЧЕСКИЕ ВЕЛИЧИНЫ

ИХ ИЗМЕНЕНИЯ

А)

Кинетическая энергия

1)

½ ν

Б)

Скорость

2)

ν

В)

Потенциальная энергия

3)

А

Б

В

9.

Брусок скользит по наклонной  плоскости вниз без трения(толкнули вверх по гладкой наклонной плоскости). Что происходит при этом с его скоростью, потенциальной энергией, силой реакции наклонной плоскости?

К каждой позиции первого столбца подберите соответствующую позицию второго и запишите в таблицу выбранные цифры под соответствующими буквами

ФИЗИЧЕСКИЕ ВЕЛИЧИНЫ

ИХ ИЗМЕНЕНИЯ

А)

скорость

1)

Увеличится

Б)

Потенциальная энергия

2)

Уменьшится

В)

Сила реакции наклонной плоскости

3)

Не изменится

А

Б

В

10.

Камень брошен вертикально вверх (свободно падает вертикально вниз). Изменяются ли перечисленные в первом столбце физические величины во время его движения вверх и если изменяются, то как?

К каждой позиции первого столбца подберите соответствующую позицию второго и запишите в таблицу выбранные цифры под соответствующими буквами.

ФИЗИЧЕСКИЕ ВЕЛИЧИНЫ

ИХ ИЗМЕНЕНИЯ

А)

скорость

1)

Увеличится

Б)

Ускорение

2)

Уменьшится

В)

Г)

Кинетическая энергия

Потенциальная энергия  

3)

Не изменится

А

Б

В

Г

11.

Гиря массой 2 кг подвешена на длинном тонком шнуре.  Если ее отклонить от положения равновесия на 10 см, а затем отпустить, она совершает свободные колебания как математический маятник. Что произойдет с периодом колебаний гири, максимальной потенциальной энергией гири и частотой ее колебаний, если начальное отклонение гири будет равно 5 см?

       К каждой позиции первого столбца подберите соответствующую позицию второго и запишите в таблицу выбранные цифры под соответствующими буквами.

ФИЗИЧЕСКИЕ ВЕЛИЧИНЫ

ИХ ИЗМЕНЕНИЯ

А)

период

1)

Увеличится

Б)

частота

2)

Уменьшится

В)

Максимальная потенциальная энергия гири

3)

Не изменится

А

Б

В

 

 

12.

Искусственный спутник движется по эллиптической орбите вокруг Земли. Изменяются ли перечисленные в первом столбце физические величины во время его приближения к Земле и если изменяются, то как?

       К каждой позиции первого столбца подберите соответствующую позицию второго и запишите в таблицу выбранные цифры под соответствующими буквами.

ФИЗИЧЕСКИЕ ВЕЛИЧИНЫ

ИХ ИЗМЕНЕНИЯ

А)

скорость

1)

Не изменится

Б)

ускорение

2)

Только увеличивается по модулю

В)

Кинетическая энергия

3)

Только уменьшается по модулю

Г)

Потенциальная энергия

4)

увеличивается по модулю и изменяется по направлению

Д)

Полная механическая энергия

5)

уменьшается по модулю и изменяется по направлению

6)

увеличивается по модулю, не  изменяется по направлению

7)

уменьшается по модулю, не изменяется по направлению

А

Б

В

Г

Д

 

 

        

13.

Комета  движется по эллиптической орбите вокруг Солнца. Как изменяются перечисленные в первом столбце физические величины во время ее приближения к Солнцу (удаления от Солнца), если считать, что на нее действует только тяготение Солнца?

       К каждой позиции первого столбца подберите соответствующую позицию второго и запишите в таблицу выбранные цифры под соответствующими буквами.

ФИЗИЧЕСКИЕ ВЕЛИЧИНЫ

ИХ ИЗМЕНЕНИЯ

А)

скорость

1)

Не изменится

Б)

ускорение

2)

Только увеличивается по модулю

В)

Кинетическая энергия

3)

Только уменьшается по модулю

Г)

Потенциальная энергия

4)

увеличивается по модулю и изменяется по направлению

Д)

Полная механическая энергия

5)

уменьшается по модулю и изменяется по направлению

6)

увеличивается по модулю, не  изменяется по направлению

7)

уменьшается по модулю, не изменяется по направлению

А

Б

В

Г

Д

 

 

14.

Установите соответствие между понятиями и их определениями.

    К каждой позиции первого столбца подберите соответствующую позицию второго и запишите в таблицу выбранные цифры под соответствующими буквами.

ФИЗИЧЕСКИЕ ВЕЛИЧИНЫ

ИХ ИЗМЕНЕНИЯ

А)

Б)

В)

Г)

Замкнутая система

Импульс тела

Поперечная волна

Кинетическая энергия

1)

Волна, в которой движение частиц в среде происходит в направлении распространения волны

2)

Система тел, взаимодействующих только между собой и не взаимодействующих с телами, не входящими в эту систему.

3)

Величина, равная произведению массы тела на его скорость.

4)

Волна, в которой частицы среды перемещаются перпендикулярно направлению распространения волны.

5)

Системы отсчета, в которых тело сохраняет состояние покоя или равномерного прямолинейного движения до тех пор, пока на него не действуют другие тела.

6)

Величина, равная половине произведения массы тела на квадрат его скорости.

А

Б

В

Г

 

 

15.

Камень брошен  вверх под углом к горизонту. Сопротивление воздуха пренебрежимо мало. Как изменяются с набором высоты модуль ускорения камня, его кинетическая энергия и горизонтальная составляющая его скорости?

Для каждой величины определите соответствующий характер изменения и занесите в таблицу  выбранные цифры под соответствующими буквами.

ФИЗИЧЕСКИЕ ВЕЛИЧИНЫ

ХАРАКТЕР ИЗМЕНЕНИЯ

А)

Модуль ускорения камня

1)

увеличивается

Б)

Кинетическая энергия камня

2)

уменьшается

В)

Горизонтальная составляющая скорости камня

3)

Не изменяется

А

Б

В

16.

Груз массой m, подвешенный к длинной нерастяжимой нити длиной l, совершает колебания с периодом Т и амплитудой А. Что произойдет с периодом колебаний, полной механической энергией и частотой колебаний нитяного маятника, если при его неизменной амплитуде уменьшить длину нити? 

    К каждой позиции первого столбца подберите соответствующую позицию второго и запишите в таблицу выбранные цифры под соответствующими буквами.

ФИЗИЧЕСКИЕ ВЕЛИЧИНЫ

ИХ ИЗМЕНЕНИЯ

А)

Период колебаний

1)

Увеличивается

Б)

Полная механическая энергия

2)

Уменьшается

В)

Частота колебаний

3)

Не изменяется

А

Б

В

17.

Установите соответствие между физической величиной и формулой для ее определения.

Физическая величина

Формула

А) Кинетическая энергия

1)       mgh

Б) Потенциальная энергия упруго-деформированного тела

2)       m∆V2/2

В) Потенциальная энергия тела, поднятого над Землей

3)       mV2/2

Г) Работа

4)         kx2/2

Д)Мощность

5)         A= F · S

Е)Импульс силы

6)         N=A/t

Ж)Импульс тела

7)          m V

8)         F·t

А

Б

В

Г

Д

Е

Ж

18.Установите соответствие между единицами измерения

А) Дж

1)    Дж/с

Б) Вт

2)    кг·м/с2·м

3)    Н·м/с

А

Б

Часть С

Задание№1.

Два шарика, массы которых m1 = 200 г и m2 = 600 г, висят, соприкасаясь на одинаковых нитях длиной l = 80 см. Первый шар отклонили на угол 90° и отпустили. На какую высоту поднимутся шарики после удара, если этот удар абсолютно неупругий?

Оценивание

Один из вариантов верного решения задачи

 и указания по оцениванию 

(допускается иная запись или ход решения, приводящие к верному ответу)

Баллы

до удара

при ударе

после удара

1. Полное правильное решение (рисунок обязателен)

3

2. Правильно записаны исходные формулы, необходимые для решения задачи, (законы сохранения энергии и импульса), но допущена ошибка в математических преобразованиях.

2

3. При записи одного из законов допущена ошибка.

1

4. Любой другой случай представления учеником своего решения (более одной ошибки в исходных формулах; отсутствие одной из исходных формул и другие варианты, не предусмотренные п/п 1-3) или отсутствие решения

0

Максимальный балл

3

Задание№2

В тело массой 4,9 кг, лежащее на гладком участке горизонтальной поверхности попадает снаряд массой 0,1 кг, летящий под углом 60° к горизонту со скоростью 60 м/с, и застревает в нем. Какой путь пройдет тело до остановки, попав на шероховатую часть поверхности, если коэффициент трения скольжения между телом и поверхностью равен 0,25?

Оценивание

Образец возможного решения

По закону сохранения импульса находим

 

Путь, пройденный телом, будет равен

Критерии оценки выполнения задания

Код

Приведено полное правильное решение, включающее следующие элементы:

1)верно записаны формулы, выражающие физические законы, применение которых необходимо для решения задачи выбранным способом (в данном решении - законы сохранения импульса, закон Ньютона; уравнения кинематики и расчета силы трения);

2)проведены необходимые математические преобразования и расчеты, приводящие к правильному числовому ответу, и представлен ответ.

3

-Представлено правильное решение только в общем виде, без каких-либо числовых расчетов.

ИЛИ

-Правильно записаны необходимые формулы, записан правильный ответ, но не представлены преобразования, приводящие к ответу.

ИЛИ

-В математических преобразованиях или вычислениях допущена ошибка, которая привела к неверному ответу.

2

-В решении содержится ошибка в необходимых математических преобразованиях, и отсутствуют какие-либо числовые расчеты.

ИЛИ

-Записаны и использованы не все исходные формулы, необходимые для решения задачи.

ИЛИ

-В одной из исходных формул допущена ошибка.

1

Все случаи решения, которые не соответствуют вышеуказанным критериям выставления оценок в 1, 2, 3 балла.

0

Задание№3

Шар массой 1кг, подвешенный на нити длиной 90 см, отводят от положения

равновесия на угол 60° и отпускают. В момент прохождения шаром положения равновесия в него попадает пуля массой 10 г, летящая навстречу шару со скоростью 300 м/с. Она пробивает его и вылетает горизонтально со скоростью 200 м/с, после чего шар продолжает движение в прежнем направлении. На какой максимальный угол отклонится шар после попадания в него пули? (Массу шара считать неизменной, а диаметр шара – пренебрежимо малым по сравнению с длиной нити.)

                       

                           α=60°         l=90см

                                                                         М=1 кг

v1=300м/с                 v2=200 м/с

m=10 г

Образец возможного решения (рисунок не обязателен)

Из закона сохранения механической энергии

находится скорость шара в нижней точке до                                   α

попадания пули:                                                                                            l

Из закона сохранения импульса определяется                                 H

скорость шара в нижней точке после попадания           h          

и вылета пули:

Закон сохранения механической энергии для шара после попадания и вылета пули: Следовательно, угол отклонения определяется равенством:

Критерии оценивания выполнения задания

Баллы

Приведено полное правильное решение, включающее следующие элементы:

1) верно записаны формулы, выражающие физические законы, применение которых необходимо для решения задачи выбранным способом ( в данном решении – закон сохранения импульса и закон сохранения энергии);

2)проведены необходимые математические преобразования и расчеты, приводящие к правильному числовому ответу, и представлен ответ. При этом допускается решение «по частям» (с промежуточными вычислениями).

3

-Представлено правильное решение только в общем виде, без каких-либо числовых расчетов.

ИЛИ

-Правильно записаны необходимые формулы, записан правильный ответ, но не представлены преобразования, приводящие к ответу.

ИЛИ

-В математических преобразованиях или вычислениях допущена ошибка, которая привела к неверному ответу.

2

-В решении содержится ошибка в необходимых математических преобразованиях, и отсутствуют какие-либо числовые расчеты.

ИЛИ

-Записаны и использованы не все исходные формулы, необходимые для решения задачи, или в ОДНОЙ из них допущена ошибка.

1

Все случаи решения, которые не соответствуют вышеуказанным критериям выставления оценок в 1, 2, 3 балла (использование неприменимого закона, отсутствие более одного исходного уравнения, разрозненные записи и т.п.).

0

Задание№4

На космическом аппарате, находящемся вдали от Земли, начал работать реактивный двигатель. Из сопла ракеты ежесекундно выбрасывается 2 кг газа (2 кг/с) со скоростью v = 500 м/с. Исходная масса аппарата М = 500 кг. Какую скорость приобретает аппарат, пройдя расстояние S = 36 м? Начальную скорость аппарата принять равной нулю. Изменением массы аппарата за время движения пренебречь.

Оценивание

Образец возможного решения

Закон сохранения импульса для системы «аппарат + газ, выброшенный за интервал времени » : ; формулы кинематики:

Из уравнений:  Из закона сохранения импульса

 

Следовательно  Отсюда

.

Критерии оценки выполнения задания

Баллы

Приведено полное правильное решение, включающее следующие элементы:

1. верно записаны формулы, выражающие физические законы, применение которых необходимо для решения задачи выбранным способом ( в данном решении – закон сохранения импульса, формулы для расчета ускорения аппарата и его скорости);

2. проведены необходимые математические преобразования и расчеты, приводящие к правильному числовому ответу, и представлен ответ. При этом допускается решение «по частям» (с промежуточными вычислениями).

3

- Представлено правильное решение только в общем виде, без каких-либо числовых расчетов.

ИЛИ

- Правильно записаны необходимые формулы, записан правильный ответ, но не представлены преобразования, приводящие к ответу.

ИЛИ

- В математических преобразованиях или вычислениях допущена ошибка, которая привела к неверному ответу.

2

- В решении содержится ошибка в необходимых математических преобразованиях, и отсутствуют какие-либо числовые расчеты.

ИЛИ

- Записаны и использованы не все исходные формулы, необходимые для решения задачи.

ИЛИ

- В одной из исходных формул допущена ошибка.

1

Все случаи решения, которые не соответствуют вышеуказанным критериям выставления оценок в 1, 2, 3 балла.

0

Задание№5.

Брусок массой m1= 600 г, движущийся со скоростью v1=2 м/с, сталкивается с неподвижным бруском массой m2= 200 г. Какой будет скорость первого бруска после столкновения? Удар считать центральным и абсолютно упругим.

Оценивание.

Один из вариантов верного решения задачи

и указания по оцениванию

(допускается иная запись или ход решения, приводящие к верному ответу)

Баллы

             m1                   m2                   m1                  m2

                         v1                                      v1'                   v2'

           

        0                                                                                X

Записан закон сохранения импульса

Записан закон сохранения механической энергии

Проведены преобразования и получен ответ:

1.Полное правильное решение (рисунок не обязателен)

3

2.Правильно записаны исходные формулы, необходимые для решения задачи ( законы сохранения импульса и механической энергии ), но допущена ошибка в математических преобразованиях

2

3.При записи одного из законов допущена ошибка

1

4.Любой другой случай представления учеником своего решения (более одной ошибки в исходных формулах; отсутствие одной их исходных формул и другие варианты, не предусмотренные пп 1-3) или отсутствие решения

0

Максимальный балл

3



Предварительный просмотр:

  • Механические колебания.

Перечень элементов содержания раздела «Кинематика материальной точки», проверяемых на едином государственном экзамене по физике таков:

Код

раздела

Код

контролируемого

элемента

Элементы содержания,

проверяемые заданиями КИМ

1

МЕХАНИКА

1.5

Механические колебания и волны

1.5.1

Гармонические колебания

1.5.2

Амплитуда и фаза колебаний

1.5.3

Период колебаний

1.5.4

Частота колебаний

1.5.5

Свободные колебания (математический и пружинные маятники)

1.5.6

Вынужденные колебания

1.5.7

Резонанс

Обобщенный план экзаменационной работы ЕГЭ 2011 г. по физике

Обозна-чение

задания в

работе

Проверяемые элементы

содержания

Коды

элементов

содержания

по кодификатору

элементов

содержания

Коды

прове-ряемых

умений

Уровень

сложности задания

Макс.

балл

за

выпол-

нение

задания

А4

Механические колебания

1.5.1-1.5.7

1, 2.1–2.4,

Б

1

А5

Свободные колебания

1.5.1-1.5.2

1, 2.1–2.4

Б

1

А 7

Вынужденные колебания

1.5.2-1.5.3

1, 2.1–2.4

Б

1

А 23-25

Затухающие колебания

1.5.4-1.5.5

1, 2.1–2.4,

Б

1

В1-В4

Гармонические колебания

1.5.1-1.5.5

1. 2.1–2.4,

П

2

С 1-С2

Уравнение движения груза , подвешенного на пружине. Уравнение движения математического маятника.

1.5.5-1.5.7

2.4

п

3

Теперь четко выясним, как планировать подготовку к экзаменам:

  1. Повторение теории (лучше всего таблицы, схемы, дающие возможность систематизировать материал).
  2. Тренировка в выполнении тестовых заданий.
  3. Самостоятельное выполнение теста из заданий с выбором  ответа по каждой из выделенных мини-тем (Свободные механические колебания, Гармонические колебания, затухающие колебания, вынужденные колебания.).
  4. Решение типичных задач (часть В и С).
  5. Тренировочная контрольная работа по решению задач.
  6. Обобщающее повторение всей темы с разбором основных ошибок.
  7. Самостоятельное выполнение тренировочного тематического теста по типу ЕГЭ (например, из 24 заданий; из  которых 18-20 с выбором ответа,  2-3 с кратким ответом, 2-3 с развернутым ответом).
  8. Анализ ошибок.

Прежде всего, остановимся на мотивации изучения  данной темы, ее значении и методах, которые используются.

Особое внимание необходимо обратить на следующие понятия, определения и закономерности:

  • Колебания;
  • Амплитуда;
  • Период;
  • Частота
  • Гармонические колебания;
  • Математический маятник;
  • Пружинный маятник;
  • Свободные колебания;
  • Вынужденные колебания;

На схемах показано, что необходимо знать по данной теме

Механические колебания

Колебательное движение характеризуют амплитудой, периодом и частотой колебаний:

   А – амплитуда;

   Т – период;

   v – частота;  

Механические колебания

 Амплитуда колебаний (А)– это максимальное расстояние, на которое удаляется колеблющееся тело от своего положения равновесия. Амплитуда колебаний измеряется в единицах длины.

 Период колебаний (Т)– это время,  в течение которого тело совершает одно полное колебание. Период колебаний измеряется в единицах времени.

 Частота колебаний (v)– это физическая величина, равная  числу колебаний, совершаемых за одну секунду. Частота измеряется в герцах (Гц).  

 Формулы :

  Период колебаний

           

     

     

Виды колебаний

Затухающие колебания – это колебания, амплитуда которых, под действием сил трения или сопротивления, со временем уменьшается, и через некоторое время колебания прекращаются.

Незатухающие колебания – это колебания. амплитуда которых со временем не изменяется, силы трения, сопротивления отсутствуют.            
Колебания ,  происходящие только благодаря начальному запасу энергии, называются свободными колебаниями.

Колебания, совершаемые телом под действием внешней периодически

изменяющейся силы, называются вынужденными колебаниями.        

Формула периода колебаний пружинного маятника

Формула периода колебаний математического маятника


Рассмотрим некоторые задачи базового уровня по теме Механические колебания.

Часть А.

Вариант 1

  1. Какие из перечисленных колебаний являются свободными?
    1. Колебания листьев на деревьях во время ветра.
    2. Биение сердца.
    3. Колебания качелей.
    4. Колебания тела на пружине.
    5. Колебания струны после того, как ее выведут из положения равновесия и предоставят самой себе.
    6. Колебания поршня и цилиндре.
    7. Колебания шарика, подвешенного на нити.

А) 1, 4, 6, 7.
Б) 2, 5, 7.
В) 4, 5, 7.
Г) 3, 4, 5, 7.

  1. Что такое амплитуда?
    А) Смещение колеблющейся точки от положения равновесия в любой момент времени.
    Б) Смещение колеблющейся точки через 1/2 Т.
    В) Наибольшее отклонение колеблющейся точки от положения равновесия.
  2. Пружинный маятник совершает колебания с амплитудой 10 см. Масса тела 1 кг. Коэффициент жесткости пружины равен 100 Н/м. Какова максимальная скорость тела?
    А)
    form_1= 0,2 м/с.
    Б)
    form_1= 0,02 м/с.
    В)
    form_1= 2 м/с.
  3. Максимальная кинетическая энергия колеблющегося тела равна 2 Дж. В какой-то момент времени потенциальная энергия этого тела равна 0,5 Дж. Какова кинетическая энергия тела в этот момент времени?
    А) 1,5 Дж.
    Б) 2,5 Дж.
    В) 1 Дж.
  4. За равные промежутки времени два тела совершили колебания: n1 = 50, n2 = 10. Каково отношение частот этих колебаний?
    А)
    form_2
    Б)
    form_3
    В)
    form_4
  5. Маятник массой 100 г отклонили от положения равновесия на угол form_5= 30°. Найдите силу, возвращающую маятник в положение равновесия (g = 10 м/с2).
    А) 500 Н.
    Б) 0,5 Н.
    В) 0,86 Н.
  6. Тело на пружине совершает колебания. Куда направлена равнодействующая сила, действующая на тело, при прохождении положения равновесия (см. рис.)? 


А) 1.
Б) Равнодействующая равна нулю.
В) 2.

pic%201

  1. Два математических маятника, имеющие одинаковые массы, но разные длины нитей (l1 = 1, l2 = 2·l), колеблются с одинаковыми амплитудами. У какого из маятников механическая энергия колебаний больше?
    А) У первого.
    Б) У второго.
    В) Одинакова.
  2. Уравнение гармонических колебаний может быть записано как в виде form_6, так и в виде form_7. Являются ли эти записи эквивалентными для гармонических колебаний?
    А) Нет.
    Б) Эквивалентны только для математического маятника.
    В) Эквивалентны для любых гармонических колебаний.
  3. Один ученик, объясняя физический смысл фазы колебаний для математического маятника, утверждал, что фаза колебаний математического маятника - это угол отклонения от вертикали. Другой же ученик утверждал, что фаза колебаний - это угловая мера времени, которое прошло от начала колебаний. Какой из учащихся дал правильное объяснение?
    А) Первый.
    Б) Оба ученика.
    В) Ни один из учеников не дал верного определения.
    Г) Второй.

Вариант 2

  1. Какие из перечисленных колебаний являются вынужденными?
    1. Колебания травы в поле на ветру.
    2. Колебания струны, издающей звук.
    3. Колебания иглы в швейной машине.
    4. Колебания тела, подвешенного на нити.
    5. Колебания поршня в цилиндре двигателя.
    6. Колебания голосовых связок.

А) Только 3, 2.
Б) Только 1, 3, 5.
В) Только 1, 5, 4.

  1. Что определяет фаза?
    А) Величину, показывающую, какая часть периода прошла от момента начала колебаний до данного момента времени.
    Б) Время, за которое совершается одно полное колебание.
    В) Наибольшее отклонение колеблющейся точки от положения равновесия.
  2. Математический маятник имеет массу 0,5 кг. При максимальном отклонении он увеличил свою потенциальную энергию до 1 Дж. Найдите максимальную скорость тела.
    А) 4 м/с.
    Б) 1 м/с.
    В) 2 м/с.
  3. Через какое-то время form_8после начала колебания тело имело потенциальную энергию, равную 4 Дж, кинетическую - 1 Дж. Какова максимальная кинетическая энергия колеблющеюся тела?
    А) 5 Дж.
    Б) 3 Дж.
    В) 4 Дж.
  4. За равные промежутки времени два тела совершили колебания: n1 = 100, n2 = 400. Каково отношение периодов этих колебаний?
    А)
    form_9
    Б)
    form_10
    В)
    form_11
  5. Маятник массой 100 г отклонили от положения равновесия на угол form_12. Найдите силу натяжения нити.
    А) 0,5 Н.
    Б) 1 Н.
    В) 0,86 Н.
  6. Математический маятник совершает колебания. Каково направление равнодействующей сил, действующих на тело, при прохождении телом положения равновесия (см. рис.)? 

А) 1.
Б) Равна нулю.
В) 2.
Г) 3.

pic%202

  1. Тело, подвешенное к пружине, колеблется по вертикали с амплитудой 5 см. Жесткость пружины равна 1 кН/м. Полная энергия тела равна:
    А) 125 Дж.
    Б) 1250 Дж.
    В) 1,25 Дж.
  2. Уравнение гармонических колебаний может быть записано как в виде form_13, так и в виде form_14. Какое из этих уравнений является верным для маятника, если начало отсчета времени соответствует положению равновесия?
    А) 1.
    Б) 2.
    В) 1, 2.
  3. Начальная фаза для математического маятника определяет:
    А) Угол отклонения нити от вертикали.
    Б) Величину смещения в любой момент времени.
    В) Величину смещения в момент начала отсчета времени (t = 0).

Часть В.

Необходимо найти соответствие между величинами или логические пары.

B1

Задачи на соответствие условно можно разделить на следующие группы:

физическая величина

- изменение величины

физический закон

- формулировка

физическая величина

- графики

физическая величина

- формулы

физическая величина

- определение

физическая величина

- единицы измерения

Частица массой m, имеющая заряд q, движется в однородном магнитном поле с индукцией B по окружности радиуса R со скоростью v. Что произойдет с радиусом орбиты, периодом обращения и кинетической энергией частицы при увеличении скорости движения?

Для каждой величины определите соответствующий характер ее изменения:

1) увеличилась

2) уменьшилась

3) не изменилась

Запишите в таблицу выбранные цифры для каждой физической величины. Цифры в ответе могут повторяться.

Радиус орбиты частицы

Период обращения частицы

Кинетическая энергия частицы

Получившуюся последовательность цифр перенесите в бланк ответов (без пробелов и каких-либо символов).

В2.

Нитяной маятник отклонили от положения

равновесия и отпустили (см.рис.). Графики

А и Б представляют изменения физических

величин, характеризующих движение шарика

в процессе колебаний. Установите соответствие

между графиками и физическими величинами,

зависимости которых от времени эти графики

могут представлять.                                                            x

К каждой позиции первого столбца подберите

соответствующую позицию второго и запишите в таблицу выбранные цифры под соответствующими буквами. Цифры в ответе могут повторяться.

ГРАФИКИ                                                        ФИЗИЧЕСКИЕ ВЕЛИЧИНЫ

А)

                                                                          1) проекция скорости vx

 

                                                                          2) координата x

 

                                                                          3) потенциальная энергия Ep

0                  T/2                       T     t 

                                                                           4) кинетическая энергия Ek

 

 

Б)

 

 

 

 

 

 

0                                                        t

                      T/2                  T        

 

 Ответ:

А

Б

Получившуюся последовательность цифр перенесите в бланк ответов ( без пробелов и каких-либо символов).

В3.

В школьной лаборатории изучают колебании пружинного маятника при различных значениях массы маятника. Если увеличить (уменьшить) массу маятника, то как изменятся: период колебаний, частота колебаний, период изменения потенциальной энергии?  

К каждой позиции первого столбца подберите нужную позицию второго и запишите в таблицу выбранные цифры под соответствующими буквами.

ФИЗИЧЕСКИЕ ВЕЛИЧИНЫ

ИХ ИЗМЕНЕНИЯ

А)

Период колебаний

1)

Увеличится

Б)

Частота колебаний

2)

Уменьшится

В)

Период изменения потенциальной энергии

3)

Не изменится

А

Б

В

В4.  Гиря массой 2 кг подвешена на длинном тонком шнуре.  Если ее отклонить от положения равновесия на 10 см, а затем отпустить, она совершает свободные колебания как математический маятник. Что произойдет с периодом колебаний гири, максимальной потенциальной энергией гири и частотой ее колебаний, если начальное отклонение гири будет равно 5 см?

       К каждой позиции первого столбца подберите соответствующую позицию второго и запишите в таблицу выбранные цифры под соответствующими буквами.

ФИЗИЧЕСКИЕ ВЕЛИЧИНЫ

ИХ ИЗМЕНЕНИЯ

период

1)

Увеличится

частота

2)

Уменьшится

Максимальная потенциальная энергия гири

3)

Не изменится

Б

В

 

 

Груз массой m, подвешенный к длинной нерастяжимой нити длиной l, совершает колебания с периодом Т и амплитудой А. Что произойдет с периодом колебаний, полной механической энергией и частотой колебаний нитяного маятника, если при его неизменной амплитуде уменьшить длину нити? 

    К каждой позиции первого столбца подберите соответствующую позицию второго и запишите в таблицу выбранные цифры под соответствующими буквами.

ФИЗИЧЕСКИЕ ВЕЛИЧИНЫ

ИХ ИЗМЕНЕНИЯ

Период колебаний

1)

Увеличивается

Полная механическая энергия

2)

Уменьшается

Частота колебаний

3)

Не изменяется

А

Б

В

Часть С.

Задача №1

Определите период Т малых колебаний и положение равновесия математического маятника длины L, находящегося в вагоне постоянно движущегося с постоянным  горизонтальным ускорением а.

                                                Решение:

 в положении равновесия нить подвеса отклонена от вертикали на угол  составляя угол  с направлением ускорения .

Задача №2

К маятнику АВ с шариком массой М подвешен маятник ВС с шариком массой . Точка А совершает колебания в горизонтальном положении с периодом Т. Найти длину нити ВС, если известно ,что нить АВ все время остается вертикальной (см. рис.).

Решение:

т.к. нить АВ вертикальна, то центр тяжести системы перемещается в горизонтальном направлении с тем же периодом Т, что и точка А.

Определим положение центра масс системы:

, где  – искомая длина маятника BC.

,

откуда длина , или , так как .

Ответ:

Задача №3

Математический маятник длиной  совершает колебания вблизи вертикальной доски. Под точкой подвеса маятника на расстоянии , равной половине длины маятника, в доску вбит гвоздь. Определите период колебания маятника.

Дано:

-длина маятника;

- расстояние от точки подвеса до гвоздя.

Найти:

Анализ и решение:

  1. Период колебаний такого маятника равен сумме периодов:

  1. Период колебаний маятника с длиной нити  будет равен:

  1. Период колебания маятника с длиной маятника нити в два раза меньше будет равен:

  1. Тогда общий период колебания маятника:

Ответ:

Задача №4.

Жидкость объемом  налита в изогнутую -образную трубку площадью поперечного сечения  канала . Определите период малых колебаний в жидкости (вязкости жидкости пренебречь).

Дано:

Найти:

Решение:

;

;

;

;

Ответ:

Задача №5

Определите максимальную амплитуду гармонических колебаний системы стоящей из двух брусков и двух колебательных систем (пружин), при которых будут совершать колебания по горизонтальной плоскости без проскальзованний относительно друг друга. Жесткость пружин  и , масса нижнего бруска , а верхнего . Коэффициент трения между брусками равно . Положение равновесия пружины не деформированы. Трения между нижним бруском и плоскостью отсутствует.

Дано:

.

Найти: -?

Решение:

Нижний груз движется только за счет силы трения:

;

;

Ответ:

 C1

 Напишите уравнение гармонического колебания точки вдоль оси  с амплитудой м, с и начальной фазой . Найдите амплитуду, скорость и ускорение.

Образец возможного решения

Дано:                                      

Найти:

Решение:

.

Критерии оценки выполнения задания

Баллы

Приведено полное правильное решение, включающее следующие элементы:

1. правильно записаны формулы, выражающие физические законы, применение которых необходимо для решения задачи выбранным способом (в данном решении – запись гармонического колебания);

2. проведены необходимые математические преобразования и расчеты, приводящие к правильному числовому ответу, и представлен ответ. При этом допускается решение "по частям" (с промежуточными вычислениями).

3

— Представлено правильное решение .

ИЛИ

— Правильно записаны необходимые формулы, записан правильный ответ, но не представлены преобразования, приводящие к ответу.

ИЛИ

— В математических преобразованиях или вычислениях допущена ошибка, которая привела к неверному ответу.

2

– В решении содержится ошибка в необходимых математических преобразованиях и отсутствуют какие-либо числовые расчеты.

ИЛИ

– Записаны все исходные формулы, необходимые для решения задачи, но в ОДНОЙ из них допущена ошибка.

ИЛИ

– Отсутствует одна из формул, необходимых для решения задачи.

1

Все случаи решения, которые не соответствуют вышеуказанным  критериям  выставления оценок в 1, 2, 3 балла (использование неприменимого закона, отсутствие более одного исходного уравнения, разрозненные записи и т.п.).

0

С2

Груз массой 0,4 кг, подвешенный к невесомой пружине, совершает 30 колебаний в минуту. Чему равна жесткость пружины?

  Дано:

m = 0,4 кг;

N = 30;

t =1 мин = 60 с

Найти:

k - ?

Решение:

Период колебаний груза, подвешенного на пружине, T =2π. Отсюда ,возведя обе части равенства в квадрат, выразим жесткость пружины:

Период – это продолжительность одного полного колебания, поэтому

T =

Отсюда

k = k =

k =

Ответ: k 4 H/

С3

Определите период колебаний механической системы изображенной на рисунке. Стержень невесомый и не деформируется. Колебания считать малыми. Силу тяжести не учитывать.

 и

-это производная дифф. по

 

Полная энергия системы:

.

, ;

Запишем полную энергию системы:

;

Ответ:

С4

Математический маятник длиной 2.45 м совершил 100 колебаний за 314 с. Определить ускорение свободного падения для данной местности.

Дано:

l = 2.45 м;

N = 100;

t =314 c

Найти:

g - ?

Решение:

Период колебаний математического маятника T = 2π зависит от длины нити и ускорения свободного падения. По определению период колебаний – время одного полного колебания, то есть T = . Приравнивая два выражения, получим:

= 2π.

Возведем обе части в квадрат.

= 4π, отсюда

g = ; g =

g = .

Ответ: g = .

Образец возможного решения

Критерии оценивания выполнения задания

Баллы

Приведено полное правильное решение, включающее следующие элементы:

  правильно записаны формулы, выражающие физические законы,
применение которых необходимо для решения задачи выбранным
способом .

  проведены необходимые математические преобразования и расчеты, приводящие к правильному числовому ответу, и представлен
ответ (с указанием единиц измерения). При этом допускается решение «по частям» (с промежуточными вычислениями).

3

— Представлено правильное решение только в общем виде, без каких-либо числовых расчетов.

ИЛИ

— Правильно записаны необходимые формулы, записан правильный ответ, но не представлены преобразования, приводящие к ответу.

ИЛИ

— В математических преобразованиях или вычислениях допущена ошибка, которая привела к неверному ответу.

2

— В решении содержится ошибка в необходимых математических преобразованиях и отсутствуют какие-либо числовые расчеты.

ИЛИ

— Записаны и использованы не все исходные формулы, необходимые для решения задачи, или в ОДНОЙ из них допущена ошибка.

1

Все случаи решения, которые не соответствуют вышеуказанным  критериям  выставления оценок в 1, 2, 3 балла (использование неприменимого закона, отсутствие более одного исходного уравнения, разрозненные записи и т.п.).

0

Определите максимальную амплитуду гармонических колебаний системы стоящей из двух брусков и двух колебательных систем (пружин), при которых будут совершать колебания по горизонтальной плоскости без проскальзованний относительно друг друга. Жесткость пружин  и , масса нижнего бруска , а верхнего . Коэффициент трения между брусками равно . Положение равновесия пружины не деформированы. Трения между нижним бруском и плоскостью отсутствует.

Образец возможного решения

Дано:

.

Найти: -?

Решение:

Нижний груз движется только за счет силы трения:

;

;

Ответ:

Качественные задачи

Задача №1

Как будет меняться период колебаний ведерка с водой, подвешенного на длинном шнуре, если из отверстия в его дне постепенно будет вытекать вода?

Решение:

Для системы, описанной в задаче, хорошим приближением является модель математического маятника. Если ведро было заполнено целиком, то сначала период будет увеличиваться, так как центр тяжести системы «ведро-вода» при вытекании воды сначала будет понижаться и в следствии этого будет расти длина маятника. Затем будет происходить уменьшение периода вследствие повышения положения центра тяжести системы «ведро-вода». Когда вода выльется целиком, период колебаний станет равен первоначальному, так как восстановится первоначальная длина.

Задача №2

Как изменится амплитуда колебаний груза на пружине, если жесткость пружины уменьшить в 4 раза, а скорость движения груза через положение равновесия останется прежней?

Решение:

Если уменьшить жесткость пружины в 4 раза, то амплитуда колебаний увеличится в 2 раза, при этом максимальная скорость груза останется прежней. Амплитуда колебаний груза зависит от жесткости пружины: , то есть при уменьшении жесткости, увеличивается амплитуда в раз.

Задача №3

Как изменится период колебаний маятника, если его перенести из воздуха в воду или в вязкое масло?

Решение:

Если маятник перенести в воду или в вязкое масло, то его период колебаний увеличится, так как сила сопротивления жидкости больше силы сопротивления жидкости больше силы сопротивления воздуха и маятник постепенно будет останавливаться. Такие колебания называются затухающими. В воде колебания затухают быстрее, чем в воздухе. Сила сопротивления вязкой среды (в данном случае масла) гораздо больше, чем у воды, поэтому в вязком масле период колебаний маятника еще сильнее увеличится и колебания будут затухать быстрее.

Задача №4

Как изменится период колебаний маятника с железным шаром, если под ним поместить электромагнит?

Решение:

Если под железным шаром маятника поместить электромагнит, то шар начнет притягиваться к электромагниту и период колебаний маятника уменьшится, а частота колебаний, соответственно, увеличится, то есть маятник будет совершать большее число колебаний за 1 секунду.

Задача №5

Изменится ли период колебаний качелей, если на доску качелей положить груз?

Решение:

Качели можно рассматривать как математический маятник. Поэтому период колебаний качелей не изменится, так как он от массы не зависит.



Предварительный просмотр:

  • МКТ идеального газа. Газовые законы.

Перечень элементов содержания раздела «Кинематика материальной точки», проверяемых на едином государственном экзамене по физике таков:

Код

раздела

Код

контролируемого

элемента

Элементы содержания,

проверяемые заданиями КИМ

2

Молекулярная физика и термодинамика

2.1

Молекулярная физика

2.1.1

Модели строения газов, жидкостей и твердых тел

2.1.2

Тепловое движение атомов и молекул вещества

2.1.3

Броуновское движение

2.1.4

Диффузия

2.1.5

Экспериментальное доказательство атомистической теории. Взаимодействие частиц вещества

2.1.6

Модель идеального газа

2.1.7

Связь между давлением и средней кинетической энергией теплового движения молекул идеального газа.

2.1.8

Абсолютная температура

2.1.9

Связь температуры газа со средней кинетической энергией его частиц

2.1.10

Уравнение р=nkT

2.1.11

Уравнение Менделеева-Клапейрона

2.1.12

Изопроцессы: изотермический, изохорный, изобарный, адиабатный процессы

2.1.13

Насыщенные и ненасыщенные пары

2.1.14

Влажность воздуха

2.1.15

Изменение агрегатных состояний вещества: испарение и конденсация, кипение жидкости.

2.1.16

Изменение агрегатных состояний вещества: плавление и кристаллизация

2.1.17

Изменение энергии в фазовых переходах

Обобщенный план экзаменационной работы ЕГЭ 2011 г. по физике

Обозна-чение

задания в

работе

Проверяемые элементы

содержания

Коды

элементов

содержания

по кодификатору

элементов

содержания

Коды

прове-ряемых

умений

Уровень

сложности задания

Макс.

балл

за

выпол-

нение

задания

А 8

МКТ

2.1.1–2.1.10

1, 2.1–2.4, 3

Б

1

А 9

МКТ

2.1.7–2.1.12

1, 2.1–2.4

Б

1

А 10

МКТ, термодинамика

2.1.13–2.1.17,

2.2.1–2.2.6

1, 2.1–2.4

Б

1

А 11

Термодинамика

2.2.1 –2.2.10

1, 2.1–2.4, 3

Б

1

А 12

Молекулярная физика,

Термодинамика

2.1-2.2

2.1–2.4, 2.6

П

1

С 3

Молекулярная физика

(расчетная задача)

2.1–2.2

2.6

В

3

Особое внимание при обобщении материала по данной теме необходимо обратить внимание на следующие понятия и определения:

  • Повторение теории (лучше всего таблицы, схемы, дающие возможность систематизировать материал).
  • Тренировка в выполнении тестовых заданий.
  • Самостоятельное выполнение теста из заданий с выбором  ответа по каждой из выделенных мини-тем (основные положения МКТ, физические величины, характеризующие  молекулы; основное  уравнение МКТ; уравнение  состояния идеального газа; газовые законы).
  • Решение типичных задач (часть В и С).
  • Тренировочная контрольная работа по решению задач.
  • Обобщающее повторение всей темы с разбором основных ошибок.
  • Самостоятельное выполнение тренировочного тематического теста по типу ЕГЭ (например, из 24 заданий; из  которых 18-20 с выбором ответа,  2-3 с кратким ответом, 2-3 с развернутым ответом).
  • Анализ ошибок.

Прежде всего, остановимся на мотивации изучения  данной темы, ее значении и методах, которые используются.

        МКТ – одна из фундаментальных научных теорий, утверждающих древнейшую научную идею – идею о дискретности вещества. Она служит основой для объяснения многих физических, химических, биологических явления; без нее не может обойтись ни одна из естественных наук. О значение этой теории своеобразно сказал известный американский физик Ричард Фейнман. Он задал вопрос: «какое утверждение, составленное из наименьшего количества слов, содержало бы наибольшую информацию для передачи грядущим поколениям, если бы в результате какой-то мировой катастрофы все накопленные поколениями научные знания оказались уничтоженными?». И сам ответил: «это – атомная гипотеза».

В молекулярной физике и термодинамике используются 2 метода:

Статистический метод (молекулярно-кинетический)

Термодинамический метод (феноменологический (явление))

1)Опирается на детали микроскопического строения вещества. Используются методы теории вероятности и статистики на основе анализа движения отдельных частиц.

1)Не рассматривает внутреннее строение вещества. Базируется на аксиомах, названных законами термодинамики, установленных опытным путем.

Обобщенная таблица для повторения

Название

Обозначение

Единицы

Формулы

масса частицы

m O

кг

m o =

число частиц

N

=N=

число Авогадро

N A

моль -1

N A  = 6,02∙10 23  моль-1 

масса тела

m

кг

m = m O N

m = Vρ

плотность вещества

ρ

ρ =  = = m O n

объем тела

V

м3

1л=0,001м3

газ занимает весь предоставленный ему объем

постоянная Больцмана

K

K = 1,38 ∙10-23

универсальная газовая постоянная

R

R = 8,31

количество вещества

υ

моль

υ = ;

υ=

относительная молекулярная масса

Mr

____

Mr o2 = 32

молярная масса

M

M o2   = 0,032

M  co2  = 0,044

M возд. = 0, 029

M =  m O N A            

средняя квадратичная скорость движения молекул

=

=

квадрат средней скорости

2

средняя кинетическая энергия движения молекул

Ē

Дж =

= H ∙ м =

 =

потенциальной нет.

Ē = kT

одной молекулы.

Ē =

внутренняя энергия

U

Дж

 U = NĒ

температура

T

Цельсий

Кельвин

T = t+273º

T= ok

t= - 273º

давление

р

πа

==

p=

p= nkT;  p= nĒ

p= m O n2;

p= ρ2

концентрация

n

или м

n=;    

n= =

Среднее расстояние между атомами; размер атома

d

d

                d

V= d d= =

V=

Газы, твердые тела (кристаллическая решетка)

Закон Дальтона

Закон Дальтона

Уравнение КлапейронаУравнение Менделеева-Клапейрона

Закон Гей-Люссака. Изобары.Объединенный газовый закон.

Закон Бойля - Мариотта. Изотермы.Закон Шарля. Изохоры.

Часть А.

Задания на понимание основных положений МКТ:

1. Какое из приведенных ниже явлений или фактов является наиболее   наглядным опытным подтверждением существования атомов и молекул?

А. Капля масла растекается по поверхности воды так, что толщина масленой пленки имеет некоторое минимальное значение.

Б. Наблюдение с помощью оптического микроскопа.

В. Диффузия.

Г. Возникновение сил упругости при деформации твердого тела.

2. Какое из приведенных ниже явлений или фактов является наиболее наглядным опытным подтверждением  движения молекул?

А. Существование твердых тел.

Б. Возникновение сил упругости при деформации твердого тела.

В. Броуновское движение.

Г. Наблюдение с помощью оптического микроскопа.

        3. Какое из приведенных ниже явлений или фактов является наиболее наглядным опытным подтверждением   взаимодействия между молекулами?

        А. Растекание масла по поверхности воды.

        Б. Нагревание тел.

        В. Фотография, полученная с помощью электронного микроскопа

Г. Возникновение сил упругости при деформации твердого тела.

4. Двигаясь во всех направлениях и почти не взаимодействуя друг с другом, молекулы быстро распределяются по всему объему сосуда. В каком состоянии находится вещество?

1) в газообразном 2) в жидком 3) в твердом 4) в газообразном или в жидком

5.  При каком из перечисленных ниже из процессов внутренняя энергия постоянной массы идеального газа остается неизменной?

1) изохорное нагревание 2) изотермическое сжатие

3) изобарное расширение 4) адиабатное расширение

6. Расстояние между соседними частицами вещества в среднем во много раз превышает размеры самих частиц. Это утверждение соответствует

1) только модели строения газов 2) только модели строения аморфных тел

3) модели строения газов и жидкостей 4) модели строения газов, жидкостей и твердых тел

Графические задачи

1.  На диаграмме V-T представлен график изменения объема идеального газа постоянной массы при изменении его температуры. Как изменяется давление газа в этом процессе?

1) все время увеличивается 2) все время уменьшается 3) сначала уменьшается, а затем увеличивается

4) сначала увеличивается, затем уменьшается

2.  На рисунках приведены графики зависимости объема  1 моля идеального газа от температуры для различных процессов. Какой график соответствует изобарному процессу?

1)

2)

3)

4)

3.   На рисунке показаны графики четырех  процессов изменения состояния постоянной массы идеального газа. Изохорным нагреванием является процесс

1) а   2) б   3) в  4) г

        

4. На рисунке А в системе координат p - V изображён график изобарного процесса идеального газа. Какой из графиков —1,2,3 или 4 — соответствует этому процессу в системе координат V Т (рис.Б)?

Безымянный5.bmp

1) 1         2) 2         3) 3         4) 4

5. На рисунке показаны графики четырех процессов изменения состояния идеального газа. Изотермическим расширением является процесс

1. а               2.  б                3. в              4.  г

Безымянный6.bmp

6. В сосуде, закрытом поршнем, находится идеальный газ. График зависимости давления газа от температуры при изменениях его состояния представлен на рисунке. Какому состоянию газа соответствует наименьшее значение объема?

Безымянный7.bmp

1) А     2) В      3) С      4) D

7. В сосуде, закрытом поршнем, находится идеальный газ. На рисунке изображена зависимость объема газа от температуры. В каком состоянии давление газа наибольшее?

Безымянный8.bmp

1) А      2) В     3) С     4) D

8. Постоянная масса идеального газа участвует в процессе, показанном на рисунке. Наименьшее давление газа в процессе достигается

Безымянный9.bmp

1) в точке 1

2) на всем отрезке 1—2

3) в точке 3

4) на всем отрезке 2—3

Задачи на установление причинно-следственных связей между входящими в них величинами.

1. Как изменится давление идеального одноатомного газа при увеличении средней кинетической энергии теплового движения его молекул в 2 раза и уменьшении концентрации молекул в 2 раза?

1) увеличится в 4 раза                             3) уменьшится в 4 раза,

2) увеличится в 2 раза                             4) не изменится

2. При температуре Т0 и давлении р0 один моль идеального газа занимает объем V0. Каков объем этого же газа, взятого в количестве 2 моль, при том же давлении p0 и температуре 2Т0?

1)4V0                            2)2V0                           3)V0                                4)8V0

3. В сосуде неизменного объема находится идеальный газ в количестве 2 моль. Как надо изменить абсолютную температуру сосуда с газом после добавления в сосуд еще одного моля газа, чтобы давление газа на стенки сосуда увеличилось в 3 раза?

1) уменьшить в 3 раза        2) уменьшить в 2 раза

3) увеличить в 3 раза        4) увеличить в 2 раза

4. В баллоне находится идеальный газ. Когда часть газа выпустили, температура газа в баллоне уменьшилась в 3 раза, а давление уменьшилось в 4 раза. Какую часть газа (в процентах) выпустили?

1) 25%                2) 50 %                3) 10%                4) 30%                5) 80 %

5.  При неизменной концентрации частиц абсолютная температура неона увеличилась в 4 раза. Давление газа при этом

1) увеличилось в 4 раза  2) увеличилось в 2 раза 3) уменьшилось в 4 раза 4) не изменилась

6.  В одном из опытов стали закачивать воздух в стеклянный сосуд, одновременно охлаждая его. При этом температура воздуха в сосуде понизилась в 2 раза, а его давление возросло в  3 раза. Во сколько раз увеличилась масса воздуха в сосуде?

1) в 1,5 раза          2) в 2 раза                  3) в 3 раза                  4) в 6 раз

7.  Какова температура таяния льда при нормальном атмосферном давлении по абсолютной шкале температур?

1) 373 К         2) 273 К         3) 173 К         4) 0 К

8. Воздух в комнате состоит из смеси газов: водорода, кислорода, азота, водяных паров, углекислого газа и др. При тепловом равновесии у всех этих газов одинаковое(ая)

1) давление             2) температура        3) концентрация молекул        4) теплоемкость

9.В цилиндрическом сосуде под поршнем находится идеальный газ, давление которого 4·105 Па и температура 300 К. Как надо изменить объем газа, не меняя его температуры, чтобы давление увеличилось до 0,8·106 Па?

1)увеличить в 2 раза                 2) увеличить в 4 раза

3) уменьшить в 2 раза                 4) уменьшить в 4 раза

Часть В.

Необходимо найти соответствие между величинами или логические пары.

B1

В сосуде неизменного объема находилась при комнатной температуре смесь двух идеальных газов, по 2 моль каждого. Половину содержимого сосуда выпустили, а затем добавили в сосуд 2 моль первого газа. Как изменились в результате парциальные давления газов и их суммарное давление, если температура газов в сосуде поддерживалась неизменной?

К каждой позиции первого столбца подберите нужную позицию второго и запишите в таблицу выбранные цифры под соответствующими буквами.

ФИЗИЧЕСКИЕ  ВЕЛИЧИНЫ

ИХ ИЗМЕНЕНИЕ

А)

парциальное давление первого газа

1)

увеличилось

Б)

парциальное давление второго газа

2)

уменьшилось

В)

давление газа в сосуде

3)

не изменилось

Получившуюся последовательность цифр перенесите в бланк ответов (без пробелов и каких-либо символов).

B2

В сосуде неизменного объема находилась при комнатной температуре смесь двух идеальных газов, по 1 моль каждого. Половину содержимого сосуда выпустили, а затем добавили в сосуд 1 моль первого газа. Температура газов в сосуде поддерживалась неизменной. Как изменились в результате парциальные давления газов и их суммарное давление?

Для каждой величины определите соответствующий характер изменения:

1)Увеличилось 2)Уменьшилось 3)Не изменилось

Парциальное давление первого газа

Парциальное давление второго газа

Давление смеси газов в сосуде

B3

Идеальный газ сжимают таким образом, что выполняется соотношение pV² = const.  Как при этом изменяются следующие физические величины?

ФИЗИЧЕСКИЕ  ВЕЛИЧИНЫ

ИХ ИЗМЕНЕНИЕ

А)

Температура

1)

увеличивается

Б)

давление

2)

уменьшается

В)

внутренняя энергия  

3)

не изменяется

А

Б

В

Идеальный одноатомный газ  переходит из состояния 1 в состояние 2 (см. диаграмму). Масса газа не меняется. Как ведут себя перечисленные ниже величины, описывающие этот газ в ходе указанного на диаграмме процесса?

B4

ФИЗИЧЕСКИЕ  ВЕЛИЧИНЫ

ИХ ИЗМЕНЕНИЕ

А)

Давление газа

1)

увеличивается

Б)

Объем газа

2)

уменьшается

В)

внутренняя энергия  

3)

не изменяется

C:\Documents and Settings\Владелец\Local Settings\Temporary Internet Files\Content.Word\сканирование0009.tif

А

Б

В

B5

В сосуде находится 3 моль гелия. Что произойдет с давлением газа на стенки сосуда, температурой и объемом газа при его изотермическом расширении?    

    К каждой позиции первого столбца подберите соответствующую позицию второго и запишите в таблицу выбранные цифры под соответствующими буквами.

ФИЗИЧЕСКИЕ ВЕЛИЧИНЫ

ИХ ИЗМЕНЕНИЯ

А)

Давление газа

1)

Не изменится

Б)

Температура газа

2)

Уменьшится

В)

Объем газа

3)

Увеличится

А

Б

В

B6

На рисунке изображен классический опыт. Толстостенный сосуд соединен с насосом. Внутрь сосуда капают несколько капель воды и сверху закрывают пробкой. С помощью насоса в сосуд закачивают воздух. При определенном давлении воздуха резиновая пробка выскакивает, а в сосуде образуется туман. Как при этом изменяются внутренняя энергия воздуха в сосуде и его температура? Теплообменом с окружающей средой пренебречь.

   C:\Documents and Settings\Владелец\Local Settings\Temporary Internet Files\Content.Word\сканирование0011.tif

 К каждой позиции первого столбца подберите соответствующую позицию второго и запишите в таблицу выбранные цифры под соответствующими буквами.

ФИЗИЧЕСКИЕ ВЕЛИЧИНЫ

ИХ ИЗМЕНЕНИЯ

А)

Внутренняя энергия воздуха

1)

увеличивается

Б)

Температура воздуха

2)

уменьшается

3)

Не изменяется        

А

Б

Часть С

C1

Воздушный шар объемом 2500 м3  с массой оболочки 400 кг имеет внизу отверстие, через которое воздух в шаре нагревается горелкой. До какой минимальной температуры нужно нагреть воздух в шаре, чтобы шар взлетел вместе с грузом (корзиной и воздухоплавателем) массой 200 кг?  Температура окружающего воздуха  7°С, его плотность 1,2 кг/м3. Оболочку шара считать нерастяжимой.

Образец возможного решения

 Шар поднимет груз при условии:  (М + m)g + mшg = ρVg,  где  M  и  m — масса оболочки шара и масса груза,  mш — масса воздуха в шаре и ρV = ma  — масса такого же по объему воздуха вне шара. Сокращая уравнение на  g, имеем:   M + m = ma – mш.

При нагревании воздуха в шаре его давление р и объем V не меняются. Следовательно, согласно уравнению Клапейрона-Менделеева,

pV =  = ,  где  μ — средняя молярная масса воздуха,  Тш  и  Та — его температуры внутри и вне шара.  Отсюда:  mш = ma = ρV,
где
ρ – плотность окружающего воздуха;

ma – mш = ρV(1 – );     M + m = ρV(1 – ). Следовательно,

(1 – ) =  =  = 0,2;      = 1 – 0,2 = 0,8;

Тш =  =  = 350 (К).                Ответ:  tш = 77°С.

Критерии оценки выполнения задания

Баллы

Приведено полное правильное решение, включающее следующие элементы:

1) верно записаны формулы, выражающие физические законы, применение которых необходимо для решения задачи выбранным способом (в данном решении — формула, выражающая условие плавания тел в газе, и уравнение Менделеева-Клапейрона);

2) проведены необходимые математические преобразования и расчеты, приводящие к правильному числовому ответу, и представлен ответ. При этом допускается решение "по частям" (с промежуточными вычислениями).

3

— Представлено правильное решение только в общем виде, без каких-либо числовых расчетов.

ИЛИ

— Правильно записаны необходимые формулы, записан правильный ответ, но не представлены преобразования, приводящие к ответу.

ИЛИ

— В математических преобразованиях или вычислениях допущена ошибка, которая привела к неверному ответу.

2

– В решении содержится ошибка в необходимых математических преобразованиях и отсутствуют какие-либо числовые расчеты.

ИЛИ

– Записаны все исходные формулы, необходимые для решения задачи, но в ОДНОЙ из них допущена ошибка.

ИЛИ

– Отсутствует одна из формул, необходимых для решения задачи.

1

Все случаи решения, которые не соответствуют вышеуказанным  критериям  выставления оценок в 1, 2, 3 балла (использование неприменимого закона, отсутствие более одного исходного уравнения, разрозненные записи и т.п.).

0

 C2

В цилиндре, закрытом подвижным поршнем, находиться воздух. Во время опыта и объем воздуха в цилиндре, и его абсолютную температуру увеличили в 2 раза. Оказалось, однако, что воздух мог просачиваться сквозь зазор вокруг поршня, и за время опыта его давление в цилиндре не изменилось. Во сколько раз изменилась внутренняя энергия воздуха в цилиндре? (Воздух считать идеальным газом.)

Образец возможного решения

Внутренняя энергия идеального газа пропорциональна его температуре и числу молей газа. Запишем уравнение Клапейрона – Менделеева: pV = νRT (p – давление газа, V – объем сосуда, R – газовая постоянная, T – температура газа, ν – число молей газа). Из него видно, что произведение νT пропорционально произведению pV. Значит, согласно условиям задачи, внутренняя энергия воздуха (как и произведение pV) увеличилась в 2 раза.   Ответ:   внутренняя энергия  увеличилась в 2 раза.

Критерии оценки выполнения задания

Баллы

Приведено полное правильное решение, включающее следующие элементы:

1. правильно записаны формулы, выражающие физические законы, применение которых необходимо для решения задачи выбранным способом (в данном решении – связь внутренней энергии идеального газа с его температурой и числом молей газа, уравнение Клапейрона – Менделеева);

2. проведены необходимые математические преобразования и расчеты, приводящие к правильному числовому ответу, и представлен ответ. При этом допускается решение "по частям" (с промежуточными вычислениями).

3

— Представлено правильное решение только в общем виде, без каких-либо числовых расчетов.

ИЛИ

— Правильно записаны необходимые формулы, записан правильный ответ, но не представлены преобразования, приводящие к ответу.

ИЛИ

— В математических преобразованиях или вычислениях допущена ошибка, которая привела к неверному ответу.

2

– В решении содержится ошибка в необходимых математических преобразованиях и отсутствуют какие-либо числовые расчеты.

ИЛИ

– Записаны все исходные формулы, необходимые для решения задачи, но в ОДНОЙ из них допущена ошибка.

ИЛИ

– Отсутствует одна из формул, необходимых для решения задачи.

1

Все случаи решения, которые не соответствуют вышеуказанным  критериям  выставления оценок в 1, 2, 3 балла (использование неприменимого закона, отсутствие более одного исходного уравнения, разрозненные записи и т.п.).

0

Воздушный шар с газонепроницаемой оболочкой массой 400 кг заполнен гелием. Он может удерживать в воздухе на высоте, где температура воздуха  17°С,  а давление  105 Па, груз массой 225 кг. Какова масса гелия в оболочке шара? Считать, что оболочка шара не оказывает сопротивления изменению объема шара.

C3

Образец возможного решения

Шар с грузом удерживается в равновесии при условии, что сумма сил, действующих на него, равна нулю: , где  M  и  m — массы оболочки шара и груза,  mг  – масса гелия, а  – сила Архимеда, действующая на шар. Из условия равновесия следует:  

M + m = mв – mг.

Давление р гелия и его температура Т равны давлению и температуре окружающего воздуха. Следовательно, согласно уравнению Клапейрона-Менделеева,     ,  где  μг — молярная масса гелия,

μв — средняя молярная масса воздуха, V – объем шара.

Отсюда: ;   mв – mг = mг ( – 1) = mг ( – 1) = 6,25mг;

M + m = 6,25mг.

Следовательно, mг =  =  (кг).       Ответ:   mг = 100 кг.

Критерии оценки выполнения задания

Баллы

Приведено полное правильное решение, включающее следующие элементы:

— верно записаны формулы, выражающие физические законы, применение которых необходимо для решения задачи выбранным способом (в данном решении — условия равновесия тела, закон Архимеда и уравнение Менделеева-Клапейрона);

— проведены необходимые математические преобразования и расчеты, приводящие к правильному числовому ответу, и представлен ответ. При этом допускается решение "по частям" (с промежуточными вычислениями).

3

— Представлено правильное решение только в общем виде, без каких-либо числовых расчетов.

ИЛИ

— Правильно записаны необходимые формулы, записан правильный ответ, но не представлены преобразования, приводящие к ответу.

ИЛИ

— В математических преобразованиях или вычислениях допущена ошибка, которая привела к неверному ответу.

2

— В решении содержится ошибка в необходимых математических преобразованиях и отсутствуют какие-либо числовые расчеты.

ИЛИ

— Записаны и использованы не все исходные формулы, необходимые для решения задачи, или в ОДНОЙ из них допущена ошибка.

1

Все случаи решения, которые не соответствуют вышеуказанным  критериям  выставления оценок в 1, 2, 3 балла (использование неприменимого закона, отсутствие более одного исходного уравнения, разрозненные записи и т.п.).

0

В понтон, лежащий на дне моря, закачивается сверху воздух. Вода вытесняется из понтона через нижнее отверстие (см. рисунок), и когда объем воздуха в понтоне достигает 28 м3, понтон всплывает вместе с прикрепленным к нему грузом. В момент начала подъема расстояние от поверхности воды в понтоне до поверхности воды в море равно 73,1 м. Масса оболочки понтона 2710 кг. Определите массу поднимаемого груза. Температура воды равна 7°С, атмосферное давление на уровне моря равно 10эПа. Объемом груза и стенок понтона пренебречь.

Образец возможного решения

Понтон с грузом начнет всплывать при условии: ρVg = Mg + mrg + mBg, где M и mв — масса оболочки понтона и масса груза, V и mв — объем и масса воздуха в понтоне, ρ — плотность воды. Следовательно, mr = ρV - М - mв.

Согласно уравнению Менделеева-Клапейрона, для воздуха в понтоне имеем:

pV = RT, причем давление воздуха равно давлению воды на заданной глубине h:

р = ра + ρgh, где ра — атмосферное давление. Отсюда:

mв = (ра + ρgh) =(1+7.3)*105≈290(кг). Следовательно,

mг= 103*28 -2,71*103 -0,29*103 ≈ 25 103 (кг).

Ответ: М = 25*103 кг.

Критерии оценивания выполнения задания

Баллы

Приведено полное правильное решение, включающее следующие элементы:

 — правильно записаны формулы, выражающие физические законы,
применение которых необходимо для решения задачи выбранным
способом (в данном решении -
второй закон Ньютона, уравнение
Менделеева—Клапейрона и формула расчета давления на заданной
глубине)',

 — проведены необходимые математические преобразования и расчеты, приводящие к правильному числовому ответу, и представлен
ответ (с указанием единиц измерения). При этом допускается решение «по частям» (с промежуточными вычислениями).

3

— Представлено правильное решение только в общем виде, без каких-либо числовых расчетов.

ИЛИ

— Правильно записаны необходимые формулы, записан правильный ответ, но не представлены преобразования, приводящие к ответу.

ИЛИ

— В математических преобразованиях или вычислениях допущена ошибка, которая привела к неверному ответу.

2

— В решении содержится ошибка в необходимых математических преобразованиях и отсутствуют какие-либо числовые расчеты.

ИЛИ

— Записаны и использованы не все исходные формулы, необходимые для решения задачи, или в ОДНОЙ из них допущена ошибка.

1

Все случаи решения, которые не соответствуют вышеуказанным  критериям  выставления оценок в 1, 2, 3 балла (использование неприменимого закона, отсутствие более одного исходного уравнения, разрозненные записи и т.п.).

0

В вакууме закреплен горизонтальный цилиндр. В цилиндре находится гелий в количестве 0,1 моль, запертый поршнем. Поршень удерживается упорами и может скользить влево вдоль стенок цилиндра без трения. В поршень попадает пуля массой 10 г, летящая горизонтально со скоростью 400 м/c, и застревает в нем. Температура гелия в момент остановки поршня в крайнем левом положении возрастает на 64 К. Какова масса поршня? Считать, что за время движения поршня газ не успевает обменятся теплом с поршнем и цилиндром.

Образец возможного решения

        Закон сохранения импульса при неупругом соударении:

m= (m + M). Отсюда:  =  , где m и M – соответственно масса пули и масса поршня ,  - скорость пули,  - скорость поршня после попадания пули .

Для внутренней энергии одноатомного идеального газа:

U =  vRT

Механическая энергия поршня с пулей превратится во внутреннюю ≈энергию гелия. Поэтому: ΔU =  vRΔT =  .

Решив систему уравнений, получаем:  M =   - m.

Ответ: M ≈ 90 г.

Методика решения задач по теме: «Идеальный газ и его свойства»

Задачи, в которых предлагается произвести расчет параметров состояния идеального газа, можно условно разделить на следующие группы. К первой группе относят задачи, в которых рассматривается изменение состояния некоторой массы газа, причем значение массы газа нигде не используется и чаще всего неизвестно. В результате таких процессов газ переходит из состояния 1 с параметрами р1, V1, T1  в состояние 2 с параметрами р2, V2, T2. Задачи этой группы удобно решать с применением объединенного газового закона (соотношения Клапейрона):

.

Если какой-либо параметр остается неизменным, то записанное соотношение переходит в один из газовых законов для описания изопроцессов: изотермического при Т = const; изобарного при р = const; изохорного при V = const.

Вторую группу составляют задачи, в которых состояние газа (его параметры р, V, T) не изменяются, но изменяется масса газа (плотность ρ, концентрация частиц n или число молей ν). Здесь для решения задачи применяют соотношение (1).

Решение задач на расчет параметров состояния газа, изменившихся в результате равновесного газового процесса, при котором изменяется и масса газа (газ расходуется или подкачивается в сосуд) осуществляется всегда с помощью уравнения газового состояния  –  уравнения Менделеева-Клапейрона:

,                                         (1)

которое иногда удобно трансформировать в соотношение вида

,

где p, V, ν = m / μ = N / NA , T – параметры состояния идеального газа – давление, объем, число молей, масса, число молекул (атомов) газа, абсолютная температура; μ и NA – молярная масса газа и число Авогадро, соответственно. Такие задачи, как правило, не имеют готовых рецептов решения и требуют индивидуального подхода. При этом алгоритм решения большинства таких задач все же существует.

Рассмотрев состояния газа и записав для каждого из них уравнение состояния (1), получаем, как правило, систему уравнений, решая которую приходим к ответу.

При этом следует иметь в виду, что уравнение (1) связывает параметры конкретного состояния газа и не рассматривает, каким способом газ эти параметры изменил в ходе термодинамического процесса. Сам процесс перехода газа из состояния 1 с параметрами p1 ,V1 ,ν1 , T1  в состояние 2 с параметрами p2 , V2 , ν2 , T2 описывается соответствующими газовыми законами, и сами процессы, в общем случае, далеко не всегда являются изопроцессами.

Для решения задач всех трех групп можно использовать следующую схему решения:

  • проанализировать условие задачи: установить, какой газ участвует в процессе (одноатомный или многоатомный, идеальный или со свойствами близкими или отличными от свойств идеального газа, химическая природа газа может быть известна и тогда с помощью таблицы Менделеева может быть определена молярная масса газа; если имеем дело со смесью газов, то надо вспомнить закон Дальтона для смеси газов;
  • уточнить из условия задачи, какие параметры меняются, какие остаются неизменными, то есть по возможности выяснить характер процесса (изопроцесс или нет, каковы абсолютные величины изменяющихся параметров, их абсолютные изменения или соотношения изменившихся величин);
  • сделать, если возможно, схематический чертеж, указав при этом, какие параметры характеризуют каждое состояние газа;
  • особое внимание уделить параметрам, заданным неявно: например, иногда для нахождения объема газа нужно использовать формулы геометрии (объемы простейших тел – шара, куба, цилиндра и др.), для нахождения давления – закон Паскаля и формулу гидростатического давления, переводные соотношения внесистемных единиц измерения давления, атмосфера, мм рт.ст. в паскали и т.д.);
  • для каждого состояния записать нужные соотношения, получить систему уравнений относительно искомой величины и решить ее, используя математические приемы и средства.

Задача 1. Идеальный газ расширяется по закону , где b – некоторая константа. Если объем газа увеличился в 4 раза, то как изменилась абсолютная температура?

Дано:

V2 = 4V1

T2 / T1 - ?

Решение.

Особая группа задач – графические задачи, для решения которых следует хорошо знать графическое изображение на плоскости всех изопроцессов в любых координатных осях:

 Безымянный4.bmpБезымянный2.bmp

В ряде качественных задач приходится оценивать одни параметры состояния газов по графической зависимости двух других. И в этом случае следует опираться на уравнение Менделеева-Клапейрона, из которого как частные случаи получаются законы идеальных газов для изопроцессов

Если в задаче рассматриваются последовательные переходы газа через ряд состояний, при этом газовые процессы различны, но известны, и приведены лишь соотношения некоторых параметров промежуточных состояний газа, а не их числовые значения, то решить задачу поэтапно нельзя. Решение проводится в общем виде, с получением конечной формулы. Для того, чтобы выстроить цепочку рассуждений, не упустить никаких нюансов сложного суммарного процесса и получить пригодную для решения систему уравнений относительно искомых параметров, удобно представить процессы графически, лучше – в координатах р-V. При этом все происходящее с газом становится наглядным.

hbc2.bmp

Задача 2. С одним молем идеального газа совершается тепловой процесс, который на V-T диаграмме имеет вид окружности. В каких из указанных точек давление будет иметь максимальное и минимальное значения? Во сколько раз отличаются эти давления? Рассчитать это давление.

Решение. Применение уравнения состояния дает:  , то есть чем меньше угол наклона кривой

V = f(T) к оси абсцисс, тем при прочих равных условиях давление газа больше.

Следовательно, максимальное давление газа в состоянии 1, а минимальное – в состоянии 2. Они отличаются в 3 раза:

Задача 3. Газ, занимающий при температуре Т1 = 400 К и давлении р1 = 0,1 МПа объем V1 = 2 л, изотермически сжимают до объема V2 и давления р2. Затем газ изобарно охлаждают  до температуры Т3 = 200 К, после чего изотермически изменяют объем газа до объема V4 = 1 л. Определить установившееся давление.

Решение.

Задача 4. Определите температуру идеального газа в состоянии 2, если состояния 2 и 4 лежат на изотерме, а температуры в состояниях 1 и 3 равны соответственно Т1 = 240 К и Т3 = 540 К.

Решение.

Из диаграммы p-V следует, что: Т2 = Т4 ; V1 = V2 ; V3 = V4 .

Для изобарных процессов 2-3 и 4-1 выполняется закон Гей-Люссака:

Интересна группа задач на расчет параметров состояния идеального газа, когда изменение параметров связано с увеличением объема системы за счет присоединения к сосуду с  газом, находящемуся в одном состоянии, другого сосуда с газом в отличном от первого газа состоянии. Начнем с простейшего случая.

               Дано:

       р1 = 1105 Па

       р2 = 3105 Па

       Т=const

       m1 = m2 = m

       p - ?

Задача 5. Два сосуда, содержащих одинаковую массу одного и того же газа, соединены трубкой с краном. В первом сосуде давление р1 = 1105 Па, во втором – р2 = 3105 Па. Определить давление в системе при изотермическом перемешивании газов после открывания крана.

Решение.

Запишем уравнения Менделеева-Клапейрона для каждого состояния газа:

Получена система уравнений. Решение системы:

Усложним задачу.

Задача 6. Два одинаковых сосуда, содержащих одинаковое число молекул водорода, соединены трубкой с краном. Средняя квадратичная скорость молекул в первом сосуде υ1 = 400 м/с, во втором сосуде υ2 = 600 м/с. Какова будет средняя квадратичная скорость молекул газа в системе из двух соединенных сосудов? Теплообмена с окружающей средой нет. Здесь и далее идеальный газ считать одноатомным.

               Дано:

       V1 = V2

       N1 = N2 = N

       μ = 210 –3 кг/моль

       υ1 = 400 м/с

       υ2 =600 м/с

       p - ?

Решение.

В таких задачах, когда из двух газовых систем образуется третья в результате смешения, при этом параллельно с процессом смешения происходит установление теплового равновесия в системе (выравнивание температур), традиционный метод решения через описание газовых состояний уравнением Менделеева-Клапейрона оказывается непригодным. Например,

Решение полученной системы уравнений приводит к тождеству, так как применение уравнений типа р = р1 + р2 и Т = Т1 + Т2 невозможно – интенсивные характеристики «давление» и «температура» не являются аддитивными. В любом случае записанную систему уравнений потребуется дополнить аддитивными величинами. Таковой является внутренняя энергия. Внутренняя энергия – экстенсивная величина, то есть зависит от числа частиц, входящих в систему. Поэтому, для первого состояния внутренняя энергия системы складывается из внутренних энергий газов в первом состоянии, U1 и U2. Во втором состоянии значение внутренней энергии останется тем же, так как теплообмена с внешней средой нет и энергия в изолированной системе, согласно закону сохранения энергии, остается величиной постоянной: U = U1 + U2 . В этом случае:

Усложним задачу, взяв, в отличие от задачи 10, разные количества разных газов.

Задача 7. Теплоизолированный сосуд объемом V = 1м3 разделен перегородкой на две равные части. В одной из них находится гелий массой m1 = 0,3 кг, в другой – аргон массой m2 = 0,6 кг. Средняя квадратичная скорость молекул обоих газов составляет υ1 = 300 м/с и υ2 = 400 м/с, соответственно. Рассчитайте парциальное давление аргона в сосуде после удаления перегородки.

Дано:

   

    V = 1 м3

    m1 = 0,3 кг

   μ1 = 0,004 кг/моль

   m2 = 0,6 кг

  μ2= 0,04 кг/моль

  υ1 = 300 м/с

  υ2 = 400 м/с

     

 Решение.

Предлагаемая задача является, по сути, аналогом предыдущей, но в отличие от нее, здесь массы газов и химическая природа газов различны. Поэтому появляется необходимость учета массовых и молярных долей компонентов газовой смеси (по ходу вычислений мы дадим промежуточные расчеты некоторых величин, не требующие поиска по условию задачи, для иллюстрации введенных понятий):

- массовая доля компонента 1 (гелия)     ;

- массовая доля компонента 2 (аргона)   ;

- молярная доля компонента 1 (гелия)  ;

- молярная доля компонента 2 (аргона)  ;

.

Тогда молярная масса смеси газов:

.

Используя свойство аддитивности внутренней энергии системы, запишем:

Здесь учтено, что

Сравните уравнения  ()  и (∗∗). Если бы массы были одинаковы, m1 = m2 = m, то уравнение (∗∗) переходит в уравнение (), а в формуле (∗∗) не молярные, а массовые доли. Связано это с тем, что средняя квадратичная  скорость теплового движения молекул идеального газа определяется внутренней энергией, а та, в свою очередь, величина экстенсивная и определяется массой системы.

Но вернемся к нашей задаче.

Так как    с учетом формулы для молярной массы смеси   , получаем:        

Парциальное давление аргона из уравнения Менделеева-Клапейрона

.

Еще большее затруднение вызывают задачи с компрессором, с помощью которого откачивают или нагнетают газ в сосуд, как правило, в изотермическом процессе. Эти задачи из той – же группы, что и предыдущие, поэтому методы их решения должны быть аналогичными.

Например:

Задача 8. Компрессор засасывает каждую секунду объем V0 воздуха, который подается затем в баллон объемом V. Через какое время давление в баллоне будет выше атмосферного р0 в m раз? Начальное давление в баллоне равно атмосферному. Процесс накачки считать изотермическим.

Начальное состояние

           p0 V                   p0 V0

Решение.

В начальный момент и камера насоса, и баллон соединены с атмосферой:

p0 V = (m/μ)RT;  p0 V0 = (m0 /μ)RT,

где Т – температура процесса (атмосферы);

За одну секунду масса воздуха увеличивается в объеме баллона на ,

то есть в 1-м состоянии (через одну секунду):

,

что вытекает и из изотермического характера процесса.

Повторив рассуждения для второй секунды, получаем:

,

и так далее.

После  t секунд:            

Задача 9. Из сосуда объемом V, давление в котором р0, откачивают воздух. Сколько качаний должен сделать поршневой насос объемом камеры V0 для того, чтобы в сосуде давление уменьшилось в m раз? Процесс откачки считать изотермическим.

Решение.

Для первого качания: .

После второго качания: и так далее.

После n качаний:

По условию задачи: .

Преобразуем выражение: для этого прологарифмируем левую и правую части уравнения и используем свойства логарифма:

Из полученного соотношения следует, что, например, для уменьшения давления в 100 раз (m = 100), p = p0 /100    V0 / V = 0,1   .

К этой же группе задач следует отнести и задачи, связанные с истечением газа из сосуда. При этом приходится пользоваться элементарными методами решения задачи, так как интегральное исчисление, используемое при решении задач на неравновесные процессы, учащиеся в своем большинстве знают не в той мере, чтобы его успешно применять к решению физических задач, либо не знают вовсе, как это имеет место в 10-м классе. Тем не менее, покажем, что если изменение параметров состояния, например, давления, при уменьшении массы газа в сосуде вследствие неравновесного истечения газа через отверстие в стенке сосуда, невелико, то грубые приближения, используемые  при элементарном способе решения такой задачи, не приводят к существенным ошибкам при получении числового ответа.



Предварительный просмотр:

  • Термодинамика.

Перечень элементов содержания раздела «Кинематика материальной точки», проверяемых на едином государственном экзамене по физике таков:

Код раздела

Код контролируемого элемента

Элементы содержания, проверяемые заданиями КИМ

2.2

Термодинамика

2.2.1

Внутренняя энергия

2.2.2

Тепловое равновесие

2.2.3

Теплопередача. Виды теплопередачи

2.2.4

Количество теплоты. Удельная теплоемкость вещества

2.2.5

Работа в термодинамике

2.2.6

Уравнение теплового баланса

2.2.7

Первый закон термодинамики

2.2.8

Второй закон термодинамики

2.2.9

КПД  тепловой  машины

2.2.10

Принципы действия тепловых машин

2.2.11

Проблемы энергетики и охрана окружающей среды

Обобщённый план экзаменационной работы ЕГЭ-2011 по физике.

Обозна-чение задания в работе

Проверяемые элементы содержания

Коды элементов содержания по кодификатору элементов содержания

Коды проверяемых умений

Уровень сложности задания

Макс. балл за выполнение задания

10

А10

МКТ, термодинамика

2.1.13-2.1.17, 2.2.1-2.2.6

1, 2.1-2.4

Б

1

11

А11

Термодинамика

2.2.1 -2.2.10

1, 2.1-2.4, 3

Б

1

12

А12

Молекулярная физика, термодинамика

2.1-2.2

2.1-2.4, 2.6

П

1

Часть А.

А 1. Если в некотором процессе газ совершил работу, равную 5 кДж, а его внутренняя энергия уменьшилась на 5 кДж, то такой процесс является:

1) изотермическим                         2) изохорическим

3) адиабатическим                         4) изобарическим

5) такой процесс невозможен

А 2. Температура кристаллического тела при плавлении не изменяется Внутренняя энергия тела при плавлении:

  1. увеличивается; 2) уменьшается; 3) не изменяется;

4) может увеличиваться, а может и уменьшаться.

А 3. Работа газа в замкнутом процессе 1–2–3–4 (рис.), равна:

1) 100 кДж;                2) 200 кДж;                3) 300 кДж;                 4) 400 кДж.

Рисунок 

А 4. Испарение жидкости происходит потому, что:

  1. самые медленные молекулы покидают жидкость;
  2. разрушается кристаллическая решетка;
  3. самые быстрые молекулы покидают жидкость;

4) ни одна из указанных ранее причин не верна.

А 5. Тела, имеющие разные температуры, привели в соприкосновение двумя способами А и В (рис. 73). Какое из перечисленных ниже утверждений является верным?

1) В положении А теплопередача осуществляется от тела 2 к телу 1; 2) в положении В теплопередачу осуществляется от тела 2 к телу 1; 3) в любом положении теплопередача осуществляется от тела 1 к телу 2; 4) теплопередача осуществляется только в положении А.

А 6. Идеальный газ получил количество теплоты 300 Дж и совершил работу  100 Дж. При этом внутренняя энергия газа

  1. увеличилась на 400 Дж
  2. увеличилась на 200 Дж
  3. уменьшилась на 200 Дж
  4. уменьшилась на 400 Дж.

А 7. Температура холодильника идеального теплового двигателя равна 27оС, а температура нагревателя на 90оС больше. Каков к.п.д. этого двигателя?

1) 23%        2) 46%        3) 77%        4) 30%        5) 66%

А 8. Газ в сосуде сжали, совершил работу 20 Дж. Внутренняя энергия газа при этом увеличилась на 30 Дж. Следовательно:

1) газ получил количество теплоты, равное 10 Дж; 2) газ отдал окружающим телам количество теплоты, равное 10 Дж; 3) газ отдал окружающим телам количество теплоты, равное 50 Дж; 4) газ получил количество теплоты, равное 50 Дж.

А 9. Температура нагревателя идеального теплового двигателя 127оС, а холодильника 7оС. Количество теплоты, получаемое двигателем ежесекундном от нагревателя, равно 50 кДж. Какое количество теплоты отдается холодильнику за 1 секунду?

1) 35 кДж        2) 20 кДж        3) 10 кДж        4) 25 кДж        5) 27 кДж

А 10. При теплопередаче твердому телу массой m количества теплоты Q температура тела повысилась на ΔT. Какое из приведенных ниже выражений определяет удельную теплоёмкость вещества этого тела?

1)                 2)                 3)                 4)        Q· m ·ΔT

Часть В.

В1. Одноатомный идеальный газ в изотермическом процессе совершает работу  А > 0. Как меняются в этом процессе объем, давление и внутренняя энергия газа, если его масса остается неизменной?

Для каждой величины определите соответствующий характер изменения:

                                1) увеличилась

                                2) уменьшилась

                                3) не изменилась

Запишите в таблицу выбранные цифры для каждой физической величины. Цифры в ответе могут повторяться.

Объем газа

Давление газа

Внутренняя энергия газа

В2. Температуру нагревателя тепловой машины уменьшили, оставив температуру холодильника неизменной. Количество теплоты, полученное газом от нагревателя за цикл, не изменилось. Как изменились при этом КПД теплового двигателя, количество теплоты, отданное газом холодильнику, и работа газа за цикл?

Для каждой величины определите соответствующий характер изменения:

                                1) увеличилась

                                2) уменьшилась

                                3) не изменилась

Запишите в таблицу выбранные цифры для каждой физической величины. Цифры в ответе могут повторяться.

КПД тепловой машины

Количество теплоты, отданное газом холодильнику

Работа газа за цикл

В3. Зависимость давления газа от его объема выражается формулой р = αV. Чему равна работа, совершенная газом при его расширении от объема V1 до объема V2?

А. .                Б. .                В..

Г. .                Д. 0.

В4. Как изменится КПД идеальной тепловой машины, работающей по циклу Карно: 1) при повышении температуры нагревателя на ΔT; 2) при понижении температуры холодильника на ΔT?

  1. В первом случае повысится, во втором понизится.

Б. В обоих случаях понизится.

B. Повысится одинаково в обоих случаях.

Г. Повысится в обоих случаях, но больше при повышении температуры нагревателя.

Д. Повысится в обоих случаях, но больше при понижении температуры холодильника.

В5. Какое значение имеет молярная теплоемкость водорода при постоянном давлении и температуре около 300 К?

А. .        Б. .        В. .                Г. 3R.

Часть С.

С 1. Определить массу m воды, которая может быть превращена в лед при 0оС испарением эфира, масса которого M = 0,1 кг, а температура to1 = 20оС. Теплообмен происходит только между эфиром и водой. Начальная температура воды также to1 = 20оС. Удельная теплота испарения эфира r = 3,8 · 105 Дж/кг, удельная теплота плавления льда λ = 3,3 · 105 Дж/кг, удельная теплоемкость воды   св = 4200 Дж/кг · К, эфира сэ = 2100 Дж/кг · К.

C 2. Вертикальная трубка, не содержащая воздуха, соединена с широким сосудом, наполненным ртутью. Когда между ними открыли кран К, в трубку вошел столб ртути высотой h и массой m. Сколько при этом выделилось тепла?

С 3. Температура некоторой массы m идеального газа с молярной массой μ меняется по закону T =αV2, где α = const > 0. Найти работу, совершенную газом при увеличении объема от V1 до V2. Поглощается или выделяется теплота при таком процессе?

С 4. Какова температура воды в водоемах под слоем льда?

С5. Вычислить шаг резьбы сверла, если при сверлении в медном цилиндре осевого отверстия цилиндр нагрелся на 43 К. Вращающий момент, приложенный к воротку, равен 16,2 Н м; 70% затрачиваемой энергии превращается во внутреннюю энергию цилиндра; диаметр отверстия равен  25 мм.

С6. Найти отношение  для смеси газов, состоящей из m1 = 20 г гелия и  m2 = 8 г водорода.

ОТВЕТЫ И РЕШЕНИЯ

Часть С.

С 1. Решение. Количество теплоты, необходимое для испарения эфира, равно Q1 = Qпол = Mr > 0. Это количество теплоты отдается водой и эфиром при их охлаждении до 0оС и при замерзании воды:

,

где λ – удельная теплота плавления льда.

Тогда уравнение теплового баланса запишется в виде

Q1 + Q2 = 0,

или

Qпол = Qотд,

,

или

.

Отсюда ,

.

С 2. Атмосферный воздух совершил работу

А = p0Sδ,

где р0 – атмосферное давление, S – площадь поверхности ртути в широком сосуде и δ – понижение уровня в этом сосуде. Так как p0 = ρgh и Sδ = sh, где ρ – плотность ртути, a s – площадь внутреннего сечения трубки, то эту работу можно представить в виде

А = ρghsh = mgh,

где т = ρhs – масса вошедшей ртути. Учтя, что потенциальная энергия ртути увеличилась на

,

приходим к выводу, что искомое количество тепла равно

.

С 3. Процесс T = αV2 не является ни изобарным, ни изохорным, ни, тем более, изотермическим. Запишем для любого состояния в этом процессе уравнение Менделеева-Клайперона: pV = (m/μ)RT. Так как T = αV2, то после подстановки получим зависимость давления от объема в виде p(V) =α (m/μ)RT. График этой зависимости представлен на рисунке .

Рисунок  

Совершенная газом работа

A = SABCD = (p1 + p2)(V2 – V1)/2 =

.

Для ответа на второй вопрос задачи воспользуемся первым законом термодинамики: Q = ΔU + A. Так как газ расширяется, то его работа A > 0. Изменение внутренней энергии идеального газа пропорционально изменению температуры: ΔU ~ ΔT. Так как T = αV2 и объем возрастает, то возрастает и температура, поэтому ΔU > 0. Тогда и Q > 0, что соответствует поглощению газом теплоты.

С 4. Известно, что вода обладает наибольшей плотностью при 4°С. При понижении температуры от более высокой до 4°С вода сжимается, но при дальнейшем остывании от 4°С до точки замерзания, т.е. до 0°С, она опять расширяется. Когда наступают морозы, то прежде всего охлаждаются и сжимаются поверхностные слои воды. Сжимаясь, они становятся тяжелее, чем нижележащие, более теплые слои; поэтому поверхностные слои опускаются вниз, а на их место поднимаются глубинные, более легкие слои воды. Когда вся масса воды охладится до 4°С, процесс охлаждения с поверхности и смена слоев прекратятся. Начиная с этой температуры, верхние слои воды уже не будут опускаться, потому что при дальнейшем охлаждении они становятся легче лежащих ниже, т.е. эти слои останутся наверху до замерзания. Самый верхний слой льда примет температуру окружающего воздуха, затем в глубину температура постепенно повышается и слои воды, лежащие глубоко, будут иметь температуру +4°С даже в самые сильные морозы (рис. ).

Рисунок  

С 5. Решение. Искомый шаг резьбы p можно найти, наделив высоту цилиндра h на число оборотов сверла n, которое необходимо сделать, чтобы просверлить цилиндр насквозь:

p = h/n.

При сверлении выделяется количество теплоты

Q = cm ΔT = cShρΔT,

где S = πd2/4. Отсюда h = Q/(cSρΔT).

Величина n определяется из выражения для работы A, совершенной при сверлении цилиндра:

A = M2πn, откуда n = A/(2πM).

Подставив значения h и n  в выражение для p и приняв во внимание, что по условию Q/A = k, получим

.

В окончательном виде

.

Подставляя числовые значения, получаем

.

Ответ. Шаг резьбы сверла – около 1,0 мм.

С 6. Количество теплоты, необходимое для нагревания m1 граммов гелия при постоянном объеме, .

Количество теплоты, необходимое для нагревания m2 граммов водорода при тех же условиях,  .  

Аналогично для смеси газов , где cV – удельная теплоёмкость смеси при постоянном объёме;  m = m1 + m2 – масса смеси.

Согласно закону сохранения энергии можно записать

ΔQv = ΔQ1 + ΔQ2;

cv1m1Δt + cv2m2Δt = cv (m1 + m2)Δt;

.

Путем аналогичных рассуждений, получаем:

cp1m1Δt + cp2m2Δt = cp (m1 + m2)Δt;

.

Удельные и молярные теплоёмкости связаны следующими соотношениями:

; .

В свою очередь, ;  ,  

где i – число степеней свободы молекулы.

Гелий – одноатомный газ, следовательно, для него i1 = 3; водород – двухатомный газ, для него  i = 5.

С учётом (1) и (2) отношение:

.



Предварительный просмотр:

  • Электродинамика. Электрическое поле.

Перечень элементов содержания раздела «Кинематика материальной точки», проверяемых на едином государственном экзамене по физике таков:

Код

раздела

Код

контролируемого

элемента

Элементы содержания,

проверяемые заданиями КИМ

3

ЭЛЕКТРОДИНАМИКА

3.1

ЭЛЕКТРИЧЕСКОЕ ПОЛЕ

3.1.1

Электризация тел

3.1.2

Взаимодействие зарядов. Два вида заряда

3.1.3

Закон сохранения электрического заряда

3.1.4

Закон Кулона

3.1.5

Действие электрического поля на электрические заряды

3.1.6

Напряженность электрического поля

3.1.7

Принцип суперпозиции электрических полей

3.1.8

Потенциальность электростатического поля

3.1.9

Потенциал электрического поля. Разность потенциалов

3.1.10

Проводники в электрическом поле

3.1.11

Диэлектрики в электрическом поле

3.1.12

Электрическая емкость. Конденсатор

3.1.13

Энергия электрического поля конденсатора

Обобщенный план экзаменационной работы ЕГЭ 2011 г. по физике

Обозначение

задания в

работе

Проверяемые элементы

содержания

Коды

элементов

содержания

по кодификатору

элементов

содержания

Уровень

сложности

задания

Макс. балл

за

выполнение

задания

1

А19

Электродинамика

3.1.-.3.6.

П

1

2

А24

Механика  – квантовая физика

 (методы научного познания)

1.1. – 5.3

Б

1

3

А25

Механика – квантовая физика

 (методы научного  познания)

1.1. – 5.3

П

1

4

В1-В4

Механика – квантовая физика

 (задачи на соответствие)

1.1. – 5.3

Б,П

2

5

С1

Механика – квантовая физика

 (качественная  задача)

1.1. – 5.3

П

3

6

С4,С5

Электродинамика

(расчетная задача)

3.1.-.3.6.

В

3

1. Типичные задачи электроки состоят в том, чтобы:

а) По заданному распределению зарядов в пространстве найти созданное ими поле — вычислить напряженность и потенциал поля в произвольной точке, или, наоборот, зная характеристики поля, найти создающие его заряды.
б) По заданному расположению и форме проводников, зная потенциал каждого проводника или их общий заряд, найти распределение зарядов в проводниках и вычислить поля, создаваемые этими проводниками.

В курсе элементарной физики, за небольшим исключением, рассматривают наиболее простые случаи: задачи о точечных зарядах, заряженных проводящих сферах, плоскостях и конденсаторах.
Иногда в эти задачи включают элементы механики, и задачи получаются комбинированными, однако главное внимание в них стараются уделять идеям электричества.

2. Задачи по электродинамике в курсе элементарной физики удобно разделить на две группы. 
К первой группе можно отнести задачи о точечных зарядах и системах, сводящихся к ним, ко второй — все задачи о заряженных телах, размерами которых нельзя пренебречь.

Решение задач первой группы основано на применении законов механики с учетом закона Кулона и вытекающих из него следствий.
Такие задачи рекомендуется решать в следующем порядке:

  • Расставить силы, действующие на точечный заряд, помещенный в электрическое поле, и записать для него уравнение равновесии или основное уравнение динамики материальной точки.
  • Выразить силы электрического взаимодействия через заряды и поля и подставить эти выражения в исходное уравнение.

Если при взаимодействии заряженных тел между ними происходит перераспределение зарядов, к составленному уравнению добавляют уравнение закона сохранения зарядов.
Далее, как обычно, надо записать вспомогательные формулы и полученную систему уравнений решить относительно неизвестной величины.

Задачи на расчет полей, созданных точечными зарядами, заряженными сферами и плоскостями, — нахождение напряженности или потенциала в какой-либо точке пространства основаны на использовании формул для расчета этих величин.
Особое внимание следует обращать на векторный характер напряженности
http://tvsh2004.narod.ru/if_11/02-1.gifи помнить, что знак перед потенциалом φ определяется знаком заряда, создающего поле.

Вычисление работы, совершенной полем над точечным зарядом, а также энергии, которую приобретает заряд в результате действия сил поля, особых затруднений не представляет.
Эти величины могут быть найдены с помощью
формул и уравнения закона сохранения и превращения энергии А = W1W2.
Как и раньше, под
W1 и W2 здесь можно понимать только полную механическую энергию заряженного тела, под А — работу внешних сил, к которым можно отнести и силы электрического поля.

Решение задач второй группы основано на использовании формул для расчета энергии (работы) электрического поля и емкости заряженного конденсатора.
В задачах на систему заряженных тел (обычно плоских конденсаторов) прежде всего необходимо установить тип соединения; выяснить, какие из конденсаторов соединены между собой последовательно, какие параллельно.

Соединение элементов цепи, в том числе и конденсаторов, может не относиться ни к последовательному, ни к параллельному.
Общую емкость такого сложного соединения методами элементарной физики можно найти сравнительно просто лишь в тех случаях, когда в схеме есть точки с одинаковыми потенциалами. Такие точки можно соединять и разъединять, распределение зарядов и потенциалов на конденсаторах от этого не изменяется.
Соединяя или разъединяя точки с одинаковыми потенциалами, можно сложное включение конденсаторов свести к комбинации последовательных и параллельных соединений.
Точки с одинаковым потенциалом всегда есть в схемах, обладающих осью или плоскостью симметрии относительно точек подключения источника питания. Здесь можно различать два случая.

Если схема симметрична относительно оси (плоскости), проходящей через точки входа и выхода тока (имеется продольная плоскость симметрии), то точки одного потенциала находятся на концах симметричных сопротивлений, поскольку по ним идут одинаковые токи.

Если схема симметрична относительно оси (плоскости), перпендикулярной линии, на которой лежат точки входа и выхода тока — в схеме имеется поперечная ось (плоскость) симметрии, то одинаковым потенциалом обладают все точки, лежащие на пересечении этой оси (плоскости) с проводниками.
Это почти очевидное обстоятельство вытекает из того, что работа электрических сил над зарядами не зависит от формы пути.
Когда установлен тип соединения (последовательный или параллельный) конденсаторов и ясно, как найти их общую емкость, дальнейший расчет сведется к тому, чтобы определить связь между зарядами и напряжениями на конденсаторах и выразить через них емкости конденсаторов.

3. При решении задач электростатики и ответах на отдельные качественные вопросы полезно иметь в виду следующее:

  1. Положительные электрические заряды, предоставленные самим себе, движутся в электрическом поле от точек с большим потенциалом к точкам, где потенциал меньше. Отрицательные заряды перемещаются в обратном направлении.
  2. Напряженность электрического поля внутри статически заряженного проводника равна нулю. Этот результат не зависит от того, наложено ли на проводник внешнее электрическое поле или нет. Потенциал всех точек, лежащих на проводнике, имеет при этом одинаковое значение, т.е. поверхность проводника является эквипотенциальной.
  3. Потенциал земли и всех тел, соединенных проводником с землей, принимается равным нулю.
  4. Работа сил электростатического поля по любому замкнутому контуру равна нулю.
  5. Если два уединенных шара соединить тонким и длинным проводом, то их общая емкость будет равна сумме емкостей отдельных шаров, поскольку потенциалы шаров будут одинаковыми, а общий заряд системы равен сумме зарядов шаров.
    По этой же причине уединенный шар можно рассматривать как два конденсатора, соединенные между собой параллельно, с емкостями, равными
    http://tvsh2004.narod.ru/if_11/12-0.gif.
  6. Если конденсатор состоит из двух проводящих сфер радиусами R и r с общим центром (сферический конденсатор), то его емкость равна:
    http://tvsh2004.narod.ru/if_11/12-1.gif
    где ε — диэлектрическая проницаемость среды, разделяющей сферы.
  7. Электрическое поле заряженного конденсатора можно рассматривать как результат наложения двух полей, созданных каждой обкладкой конденсатора. Если поля, создаваемые обкладками плоского заряженного конденсатора, можно считать однородными, то напряженность поля в конденсаторе будет в 2 раза больше напряженности поля, создаваемое одной бесконечной заряженной плоскостью.
  8. В плоском конденсаторе одну пластину можно рассматривать как тело с зарядом q, помещенное в однородное электрическое поле с напряженностью Е1,созданное другой пластиной. Тогда со стороны первой пластины на вторую (и наоборот) будет действовать сила:
    http://tvsh2004.narod.ru/if_11/12-2.gif
    Если плоский конденсатор подключить к источнику питания, зарядить его и затем отключить, то при изменении емкости
    С конденсатора вследствие раздвижения (сближения) или смещения пластин, внесения (удаления) диэлектрика заряд на конденсаторе не меняется.
    Что при этом происходит с величинами
    q, U, Е, F или W, установают, анализируя формулы связи напряженности электрического поля с разностью потенциалов, определения емкости, емкости плоского конденсатора.
    В том случае, когда между пластинами конденсатора вставляют (или вынимают) незаряженную металлическую пластинку, не замыкающую конденсатор, область поля конденсатора уменьшается на величину объема этой пластинки. Все величины будут при этом изменяться точно так же, как если бы мы сближали (или раздвигали) обкладки. Если конденсатор подключен к источнику постоянного напряжения, то при всех указанных выше изменениях емкости конденсатора между его пластинками остается неизменным напряжение. Величины
    q, С, Е и F могут при этом меняться.
  9. Если батарею конденсаторов, подключить к источнику напряжения и сообщить ей некоторый заряд, то алгебраическая сумма зарядов любой группы обкладок, изолированных от источника, всегда должна быть равна нулю, поскольку заряды в этой группе пластин разделяются вследствие индукции.

При расчете полей, возникающих в системе заряженное тело — незаряженная проводящая поверхность, удобно использовать метод зеркального изображения зарядов. Этот метод основан на следующем принципе:
      Если в электрическом поле заменить какую-либо эквипотенциальную поверхность проводником, имеющим потенциал и форму этой поверхности, то электрическое поле после такой замены останется прежним. Отсюда, в частности, следует, что при помещении точечного заряда вблизи бесконечной проводящей плоскости на последней заряды перераспределяются так, что электрическое поле между плоскостью и зарядом оказывается тождественным полю, создаваемому рассматриваемым зарядом и его зеркальным изображением в проводящей плоскости.

Часть А

А1. Разность потенциалов между пластинами 100 В. Одна из пластин заземлена. (см. рисунок). Определите потенциал точек А,В,С,D,Е,К.

Безымянный

Ответ

φА

φВ

φС

φD

φЕ

φК

1

50

50

75

25

100

0

2

50

50

25

75

0

100

3

100

100

100

100

0

0

4

0

0

75

25

100

0

Ответ: 1.

А2. Точечный заряд 1*10-7 Кл помещён в вакууме, а точечный заряд 3*10-7 Кл – в некоторой жидкости. Напряжённости поля в точках, равноотстоящих от зарядов, одинаковы. Определите диэлектрическую проницаемость жидкости.

1) 9

2)1/9

3)3

4) 1/3

Ответ: 1.

А3. Какую кинетическую энергию приобретёт заряженная частица, пройдя в электрическом поле разность потенциалов 100 В? (Заряд частицы 2 мкКл, начальная скорость равна нулю).

1) 0,0001 Дж

2) 200 Дж

3) 0,0002 Дж

4) 100 Дж

Ответ: 3.

А4. Как называется отношение работы, совершаемой электрическим полем при перемещении положительного заряда, к значению заряда?

1) потенциальная энергия электрического поля

2) напряжённость электрического поля

3) электрическое напряжение

4) электроёмкость

Ответ: 2.

А 5. Какая из приведённых ниже математических записей определяет энергию заряженного конденсатора?

1)  

2)  

3) к2πσ

4)  

Ответ: 3.

Часть В.

В1. Два точечных заряда 4*10-8 Кл и 9*10-8 Кл находятся на расстоянии 50 см. Где находится точка, в которой напряжённость электрического поля равна нулю?

Дано:                                               Решение:        

q1 = 4*10-8 Кл                        Напряжённость электрического поля в данной точке

q2 = 9*10-8 Кл                        равна нулю, т.е.

E = 0                                      

R = 50 см = 0,5 м

х = ?

 

В.2.На расстоянии 2 м от заряда q напряжённость электрического поля равна 100 В/м. Определить потенциал электрического поля этого заряда на расстоянии 8 м от него.

Дано:                                                  Решение:

R1 = 2 м.                   Напряжённость электрического поля равна: Е =  , потенциал

Е = 100 В/м.             точки поля: φ = , отсюда:

R2 = 8 м.                  

φ = ?                                      Значит, φ = (100*4)/8 = 50 В.         

В3. Плоский воздушный конденсатор зарядили до некоторой разности потенциалов и отключили от источника тока. Как изменятся перечисленные в первом столбце физические величины, если пластины конденсатора раздвинуть на некоторое расстояние?

Физические величины

Их изменение

А) Заряд на обкладках конденсатора

1) увеличится

Б) Электроёмкость конденсатора

2) уменьшится

В) Энергия электрического поля конденсатора

3) не изменится

Ответ: А-3, Б -2, В – 1.

В4.  Два конденсатора, ёмкости которых С1 и С2  соединены последовательно и подключены к источнику с напряжением U. Определить напряжение на конденсаторах.

Дано:                                           Решение:

С1                                Пусть U1  и U2    - напряжения на конденсаторах, тогда заряды на них  

С2                            равны С1U1  и С2U2. Используя свойства последовательного соединения

U                             конденсаторов, получим систему уравнений:

U1 -?

U2 -?

                               

                                 

В5. На нити подвешен шарик массой 0,3 г, которому сообщили заряд 4*10-9 Кл. На какое расстояние необходимо поднести к нему равный ему заряд, чтобы сила натяжения нити уменьшилась в 4 раза?

Дано:                                               Решение:

m = 0,3 г = 3*10-4 кг     Первый шарик висит на нити в равновесии, следовательно, Т = mg,

q = 4*10-9 Кл.                где Т – сила натяжения нити. Снизу поднесли шарик с одноимён-

T1  = T/4                         ным зарядом, между ними возникает кулоновская сила отталкива-

R = ?                               ния.                                                                            

       

                                                              

Т1 + FK + mg = 0. В проекции на ось х:

Т1 + FK = mg, но так как T1  = T/4, то T/4 + FK = mg,  отсюда      FK = ¾ mg  .

Следовательно,      ¾ mg.      Отсюда   R =8*10-3 м = 8 мм.    

Часть С.

С1. Конденсатор ёмкостью 100 мкФ зарядили до 200 В и подсоединили параллельно с незаряженным конденсатором ёмкостью 300 мкФ. Определить заряд, который появился на втором конденсаторе.

Дано:                                            Решение:

U = 200 B                                     При подключении незаряженного конденсатора

C1 = 100 мкФ = 10-4Ф                  параллельно заряженному произойдёт перераспределение

C2 = 300 мкФ = 3*10-4Ф              зарядов. Так как соединение параллельное, то

q2 =?                                              U1 = U2 =U,  q1 = C1U, q2 = C2U,

 - заряды распределяются прямо пропорционально их ёмкостям.

q = q1+ q2 ,   q1 = q - q2. Следовательно, и их ёмкостям.

. Значит, q2 = , так как q = C1U, то q2 == 1,5*10-2 Кл.

С2.   В плоском конденсаторе, находящемся в вакууме, в равновесии находится маленький шарик. Расстояние между пластинками конденсатора равно 2 см, приложенная разность потенциалов 800 В. Внезапно разность потенциалов падает до 780 В. Через какое время шарик достигнет нижней пластины, если вначале он находился посередине конденсатора?

Дано:                                                             Решение:

d = 2 см = 0,02 м                      До падения напряжения капелька находилась в равновесии,

U1 = 800 В                                 то есть: mg = Fэл.

U2 = 780 В                                 mg = qE, где E =  (1)

t = ?

Тогда mg = q. После того, как упало напряжение, капелька стала двигаться с ускорением, которое определим по второму закону Ньютона:

ma  = mg - Fэл = mg - q, а = g - .

Из (1) определим: = .

Так как = , то t = =2,83 c.

С3. Определить разность потенциалов между точками А и В в схеме, изображённой на рисунке.

Дано:                                          Решение:

C1 , C2 ,                           Так как конденсаторы соединены последовательно, то

ε 1 2 ,                            U 1 + U 2 =  ε 1 + ε 2 , q 1 =q 2 , C1q1 = C2q2.  

φВ  φА = ?                      U 2 =  U1.

                                        Разность потенциалов между точками А и В: φВ  φА = ε 1 - U 1

U 1 = ε 1 + ε 2 - U 2 =  ε 1 + ε 2 -U1.

U 1 +U1 = ε 1 + ε 2

U 1 =  ( ε 1 + ε 2)

φВ  φА = ε 1 -( ε 1 + ε 2) =( ε 1 C1 - ε 2 C2)

С4. По проволочному кольцу радиусом 4 см равномерно распределён заряд 8*10-8Кл. Каков потенциал в центре колеса, в точке, находящейся на оси колеса на расстоянии 3 см, и разность потенциалов между этими точками?            

Дано:                                                  Решение:

q  =8*10-8Кл.                       Потенциал в центре кольца: φ1 = = 1,8*104  В/м.                      

R = 4 см = 0,04 м                Потенциал в точке, находящейся на оси колеса на расстоянии

d = 3 см = 0,03 м                 d от неё: φ2 =  = 1,44*104  В/м.                                          

φ1 = ?                                     φ1  φ2 = 3,6* 103  В/м.                                                                

φ2 = ?                      

φ1  φ2 = ?                      

С5.Плоский воздушный конденсатор имеет электроемкость C и заряжен до напряжения U. Какую работу надо совершить, чтобы увеличить расстояние между его обкладками вдвое?

Решение:

При увеличении расстояния между обкладками в два раза, в два раза уменьшается электроемкость конденсатора. Так как при этом заряд на пластинах не изменяется, в два раза увеличивается напряжение между ними. Следовательно:

А = ΔW = -  =



Предварительный просмотр:

  • Законы постоянного тока.

Перечень элементов содержания раздела «Законы постоянного тока», проверяемых на едином государственном экзамене по физике таков:

Код

раздела

Код

контролируемого

элемента

Элементы содержания,

проверяемые заданиями КИМ

3.2

Законы постоянного тока

3.2.1

Постоянный электрический ток. Сила тока

3.2.2

Постоянный электрический ток. Напряжение

3.2.3.

Закон Ома для участка цепи.

3.2.4

Электрическое сопротивление

3.2.5

Электродвижущая сила. Внутреннее сопротивление источника тока

3.2.6

Закон Ома для полной цепи

3.2.7

Параллельное и последовательное соединение проводников

3.2.8

Смешанное соединение проводников

3.2.9

Работа электрического тока

3.2.10

Мощность электрического тока

3.2.11

Носители свободных электрических зарядов в металлах, жидкостях и газах

3.2.12

Полупроводники. Собственная и примесная проводимость полупроводников

Обобщенный план экзаменационной работы ЕГЭ 2011 г. по физике

Обозначение

задания в

работе

Проверяемые элементы

содержания

Коды

элементов

содержания

по кодификатору

элементов

содержания

Уровень

сложности

задания

Макс. балл

за

выполнение

задания

1

А14

Постоянный ток

3.2.1-3.2.10

Б

1

2

А19

Электродинамика

3.1-3.6

П

1

3

А24-25

Механика –

квантовая физика

 (методы познания)

1.1. – 5.3

Б,П

1

4

В1-В4

Механика –

квантовая физика

 (задачи на соответствие)

1.1. – 5.3

Б,П

2

5

С4-С5

Электродинамика (расчетная задача)

1.1. – 5.3

В

3

Особое внимание при обобщении материала по данной теме необходимо обратить внимание на следующие понятия и определения:

  • Электрический ток, условия существования, действия  электрического тока;
  • Источники тока, виды ;
  • Электрический заряд, мгновенное значение силы тока, постоянный электрический ток, плотность тока;
  • Закон Ома для участка цепи, зависимость сопротивления от температуры, сверхпроводимость;
  • Последовательное и параллельное соединение проводников;
  • Шунт к амперметру и добавочное сопротивление к вольтметру;
  • Закон Ома для полной цепи, короткое замыкание;
  • Последовательное и параллельное соединение источников тока;
  • Правила Кирхгофа;
  • Работа и мощность электрического тока;
  • Закон Джоуля – Ленца
  • Мощность источника тока
  • Коэффициент полезного действия источника тока
  • Электрический ток в жидкостях, первый  и второй законы Фарадея, объединенный закон Фарадея, техническое применение электролиза;
  • Электрический ток в газах, молния;

Часть А.

А1

В течение 20 с сила тока на участке цепи возрастала от 0 до 5 А. Какое количество электричества было перенесено?

1)100 Кл    2) 50 Кл    3) 25 Кл    4) 200 Кл

А2

Лампа сопротивлением R находится под напряжением U. Параллельно ей подключают вторую лампу сопротивлением 3R. Сравните значения силы тока в каждой лампе после соединения.

  1. I1>I2  в 3раза 2) I1 < I2  в 3 раза 3) I1 > I2 в 6 раз 4) I1 = I2 

А3

Лампа сопротивлением R находится под напряжением U. Последовательно ей подключают вторую лампу сопротивлением 3R. Сравните значения напряжения в каждой лампе после соединения.

  1. U12 в 2 раза2) U1>U2 в 3 раза 3)U12 в 4 раза 4) U1 < U2 в 3 раза    

А4

Отрезок однородной проволоки разрезали на 8 одинаковых частей и соединили эти части параллельно. Сопротивление такой системы оказалось равным 1  Ом. Каким было сопротивление проволоки до того, как ее разрезали?

  1.  28 Ом    2) 64 Ом    3) 32 Ом   4) 16 Ом

А5

Медная проволока при температуре 00 С имеет сопротивление R. До какой температуры надо нагреть проволоку, чтобы ее сопротивление увеличилось в 2 раза? Температурный коэффициент сопротивления 0,004 К-1.

  1. 5000 С  2) 2500 С   3)  3000 С   4) 4500 С

А6

Батарея ЭДС которой 16 В, замкнута на резистор сопротивлением 6 Ом. Сила тока в цепи 2 А. Каково внутреннее сопротивление источника тока?

  1. 2 Ом  2) 4 Ом  3) 0,5 Ом  4) 1 Ом

А7

К батарее с ЭДС равным 3 В подключили резистор сопротивлением R = 20 Ом, при этом напряжение на резисторе оказалось равным U=2 В.Определить ток короткого  замыкания батареи.

  1. 0,3 А   2)  3 А  3)  1,5 А  4) 2 А

А8

Вольтметр показывает 3 В. Что показывает амперметр?

R1= 2 Ом,  R2 = 2 Ом   R3=  1 Ом

  1. 4А    2)   6А  3) 4,5А    4) 5,5А

Схема1

 

А9

Амперметр имеет сопротивление 0,02 Ом, его шкала рассчитана на 1,2 А. Каково должно быть сопротивление шунта, чтобы с помощью этого амперметра можно было измерять токи, силой до 6А?

 1)Rш = 4х10-3 Ом                     2) Rш = 5х102 Ом

3) Rш = 6 х10-3 Ом                    4) Rш =12х10-3 Ом

А10

Какое добавочное сопротивление необходимо подсоединить к вольтметру с полной шкалой 3 В, чтобы можно было измерять напряжение до 30 В, если сопротивление вольтметра 600 Ом ?

1)Rд = 6 х103 Ом                     2) Rд =0,54 х102 Ом

3) Rд = 5,4 х10-3 Ом                    4) Rд =540 Ом

Ответы на задания блока А

А1

А2

А3

А4

А5

А6

А7

А8

А9

А10

2

1

4

2

2

1

1

3

1

1

Часть В.

Задачи на соответствие условно можно разделить на следующие группы:

физическая величина

- изменение величины

график

- физическая величина

физическая величина

- приборы

физическая величина

- формулы

физическая величина

- определение

физическая величина

- единицы измерения

ученые

- открытия

В1.

Параллельно лампочке включен реостат. Что произойдет с накалом  лампочки, общим сопротивлением участка цепи, напряжением на лампочке, если, перемещая ползунок реостата, уменьшить его сопротивление? Напряжение на участке цепи не изменяется.

ФИЗИЧЕСКИЕ

ВЕЛИЧИНЫ

A. Накал лампочки

Б. Общее сопротивление

В. Напряжение на лампочке

ИЗМЕНЕНИЕ

ВЕЛИЧИНЫ

1) увеличится

2) уменьшится

3) не изменится

А

Б

В

3

2

3

        

В2.    

В2

 

ФИЗИЧЕСКИЕ ВЕЛИЧИНЫ                            Графики

                                                                                     1)

А. Зависимость силы тока от напряжения

Б) Зависимость силы тока от сопротивления

А

Б

4

2

          2)

          3)

          4)

В3.

Установите соответствие между физическими величинами и приборами,

с помощью которых эти величины определяются.

К каждой позиции первого столбца подберите нужную позицию из второго и запишите в таблицу выбранные цифры под соответствующими буквами.

ФИЗИЧЕСКИЕ

ВЕЛИЧИНЫ

А) сила тока

Б) время

В) напряжение

 ПРИБОРЫ

1) часы

2) секундомер

3) амперметр

4) вольтметр

5) гальванометр

А

Б

В

3

2

4

В4.

Установите соответствие между физическими величинами и формулами, по которым эти величины определяются.

К каждой позиции первого столбца подберите нужную позицию из второго и запишите в таблицу выбранные цифры под соответствующими буквами.

ФИЗИЧЕСКИЕ

ВЕЛИЧИНЫ

А) мощность

Б) работа

В) ЭДС источника тока

А

Б

В

2

4

5

ФОРМУЛЫ

1. P=UR

2. P=UI

3. A=U2It

4. A=I2Rt

5. ε=U+Ir

6. ε = U-IR


В5

Установите соответствие между физическими величинами и их определениями.

К каждой позиции первого столбца подберите нужную позицию из второго и запишите в таблицу выбранные цифры под соответствующими буквами

ФИЗИЧЕСКИЕ

ВЕЛИЧИНЫ

ОПРЕДЕЛЕНИЯ

А)  Сила тока

Б)  Работа тока

  1. Сила тока равна отношению заряда, переносимого через поперечное сечение проводника за интервал времени, к этому интервалу времени.

А

Б

1

3

 

  1. Силой тока называют движение заряженных   частиц.
  2. Работа тока на участке цепи равна произведению силы тока, напряжения и времени, в течение которого совершалась работа.

Работа равна произведению силы тока, сопротивления и времени, в течение которого совершалась работа.

В 6

Установите соответствие между физическими величинами и их единицами измерения.

К каждой позиции первого столбца подберите нужную позицию из второго и запишите в таблицу выбранные цифры под соответствующими буквами

ФИЗИЧЕСКИЕ  ВЕЛИЧИНЫ

ЕДИНИЦЫ ИЗМЕРЕНИЯ

А) Кулон

  1. Омхм

Б) Сила тока

  1. Омхмм2

В) Удельное сопротивление

  1. А/с

  1. Ахс

  1. А

А

Б

В

4

5

1

В 7

Установите соответствие между физическими  открытиями и  именами ученых.

К каждой позиции первого столбца подберите нужную позицию из второго и запишите в таблицу выбранные цифры под соответствующими буквами

ИМЕНА УЧЕНЫХ

ОТКРЫТИЯ, ЗАКОНЫ

А) Ш.Кулон

  1.  ε = U +Ir

Б) Ом

  1. R = U/I + ε

  1. F = k q1q2/r2

  1.  F= q2/r2

  1. RI = U+I

А

Б

3

1

Часть С.

Критерии оценки выполнения задания

Баллы

Приведено полное правильное решение, включающее следующие элементы:

— верно записаны формулы, выражающие физические законы, применение которых необходимо для решения задачи выбранным способом (в данном решении — формулы кинематики для равноускоренного движения, тригонометрические соотношения для проекций величин);

— проведены необходимые математические преобразования и расчеты, приводящие к правильному числовому ответу, и представлен ответ. При этом допускается решение "по частям" (с промежуточными вычислениями).

3

— Представлено правильное решение только в общем виде, без каких-либо числовых расчетов.

ИЛИ

— Правильно записаны необходимые формулы, записан правильный ответ, но не представлены преобразования, приводящие к ответу.

ИЛИ

— В математических преобразованиях или вычислениях допущена ошибка, которая привела к неверному ответу.

2

– В решении содержится ошибка в необходимых математических преобразованиях и отсутствуют какие-либо числовые расчеты.

ИЛИ

– Записаны все исходные формулы, необходимые для решения задачи, но в ОДНОЙ из них допущена ошибка.

ИЛИ

– Отсутствует одна из формул, необходимых для решения задачи.

1

Все случаи решения, которые не соответствуют вышеуказанным  критериям  выставления оценок в 1, 2, 3 балла (использование неприменимого закона, отсутствие более одного исходного уравнения, разрозненные записи и т.п.).

0

Задачи и образцы возможного решения

С1

Определить изменение энергии электрического поля конденсатора при перекидывании ключа с контакта  1 на контакт 2 в схеме на рисунке. Внешнее сопротивление R=300 Ом, ЭДС источника тока ε = 10 В, емкость конденсатора С = 10 мкФ, внутреннее сопротивление источника тока r = 20 Ом.

Дано:                                                                                      Решение

R=300 Ом                                                     В первом случае (схема 1), ток в цепи

ε = 10 В                                                 отсутствует и напряжение на обкладках                      С = 10 мкФ=10х10-6Ф                        конденсатора равно ε, энергия конденсатора

r= 20 Ом                                               W1=C ε2/2

Найти:                                                  Во втором случае (схема 2) ток в цепи равен  

ΔW                                                         I= ε/(R+r)

                                                                        Разность потенциалов на обкладках              

    конденсатора равна падению напряжения на сопротивлении R:                                                                      

                                                                      U=IR=R ε/(R+r)

     Энергия конденсатора в этом случае равна

                                                                        W2=CU2/2 =  C ε2R2/2(R+r)2

Найдем изменение энергии конденсатора

                                                      ΔW=W2-W1=  C ε2R2/2(R+r)2 - C ε2/2

                                                      ΔW =  C ε2/2(R2/(R+r)2 - 1)  

                                                      ΔW = -6,05х10-5Дж                                                            

С1

С2

Определить эквивалентное сопротивление схемы, изображенной на рисунке, в случае подключения ее к клеммам АВ.

С3

С 3

С6

С4

Чему равно эквивалентное сопротивление кубика, сделанного из проволоки, если его подключить к источнику питания в точка А и D (см. рисунок).

С2

С5

Во внешней цепи при силе тока  I1 = 5А  выделяется мощность P1=9.5 Вт, а при токе I2 =8А   выделяется мощность P2=14,4 Вт. Найти ток короткого замыкания.

С4

С6

Проволочное кольцо сопротивлением 120 Ом подключили к источнику тока с напряжением 24 В как показано на рисунке (в точках А и В), причем соотношение длин участков L2/L1 = 3. Как изменилось сопротивление кольца (увеличилось или уменьшилось)? Определите силу тока в обеих частях кольца.

 

С5

Дано:                                              Решение

R=120 Ом                         Результатом такого включения является схема,    

U=24 В                             состоящая из двух параллельных участков. Обозначим

L2/L1 = 3                           R1 – верхняя часть окружности, R2 - нижняя   часть

Найти:                               окружности.  

Rобщ - ?                             Как видно из формулы R = ρl/S,сопротивление зависит

I1 - ?                                только от длины проводника, так как ни материал, ни  

I2 - ?                                площадь сечения не изменялись. Тогда R1=R/4 (R1=30 Ом),  

                                      R2 =3R/4 (R2 = 90 Ом).

   

                                     1/ Rобщ = 4/R +4/3R

                                     Rобщ = 3R/16

                                     Rобщ = 22,5 Ом

                                     R/  Rобщ =120/22.5=5.3 -сопротивление уменьшилось в

                                     5,3раза.

     

                                     I=U/R

                                     I1=0,8 А

                                     I2=0,27А



Предварительный просмотр:

  • Основы специальной теории относительности.

Перечень элементов содержания раздела «Основы специальной теории относительности», проверяемых на едином государственном экзамене по физике таков:

Код

раздела

Код

контролируемого

элемента

Элементы содержания,

проверяемые заданиями КИМ

4

Основы специальной теории относительности

4.1

Инвариантность скорости света. Принцип относительности Эйнштейна.

4.2

Полная энергия

4.3

Связь массы и энергии. Энергия покоя.

Обобщенный план экзаменационной работы ЕГЭ 2011 г. по физике

Обозначение

задания в

работе

Проверяемые элементы

содержания

Коды

элементов

содержания

по кодификатору

элементов

содержания

Уровень

сложности

задания

Макс. балл

за

выполнение

задания

А18

Элементы СТО

4.1

Б

1

А24-25

Механика –

квантовая физика

 (методы познания)

1.1. – 5.3

Б,П

1

В1-В4

Механика –

квантовая физика

 (задачи на соответствие)

1.1. – 5.3

Б,П

2

С1-С2

Механика –

квантовая физика

 (расчетные задачи)

1.1. – 5.3

Б,П

3

Программа по физике предусматривает изложение элементов теории относительности, как завершающей темы электродинамики, что соответствует истории возникновения данной теории.

          Основная трудность изложения идей теории относительности заключается в кажущейся парадоксальности ряда её положений, и в их противоречии «здравому смыслу». Преодоление этого психологического барьера связано с определёнными трудностями.

 Изучение релятивистских эффектов требует достаточно точного и однозначного разбора. Если чётко сформулировать основные положения, определить основные понятия, логично и доступно вывести следствия, никакие парадоксы не возникнут.

       Особое внимание при обобщении материала по данной теме необходимо обратить внимание на следующие понятия и определения:

  • Принцип относительности, утверждающий, что все законы физики одинаковы во всех инерциальных системах отсчёта.
  • Принцип постоянства скорости света, утверждающий, что скорость света одинакова во всех инерциальных системах отсчёта.
  • Преобразования Лоренца, устанавливающие между пространственными и временными координатами событий в одной системе отсчёта и пространственными и временными координатами тех же событий в другой системе отсчёта
  • Относительность одновременности, заключающаяся в том, что два события одновременные в одной системе отсчёта, могут оказаться неодновременными в другой системе отсчёта.
  • Релятивистский эффект замедления времени (часы, движущиеся относительно неподвижного наблюдателя, по его измерениям, замедляют свой ход).                   
                                                                     
  •  Релятивистское сокращение длины   
                                                                 
  • Релятивистское увеличение массы:

                                                                             

  • Релятивистский импульс:

                                                       

  • Релятивистские законы сложения скоростей

                                                                             

  • Закон взаимосвязи массы и энергии.
                                                                                 
  • Закон сохранения энергии:

                                                             =T+

Часть А

  1. В инерциальной системе отсчета свет распространяется в вакууме со скоростью с. В некоторой системе отсчета с одинаковыми скоростями υ движутся навстречу две светящиеся кометы. Скорость света, испущенного первой кометой, в системе отсчета, связанной с другой кометой, равна
  1. c                    2) c+v                       3)  2c+v                                 4)   2v+c

  1. На неподвижное зеркало перпендикулярно поверхности падает свет от источника, приближающегося к зеркалу со скоростью υ  (см. рисунок). Какова скорость отраженного света в инерциальной системе отсчета, связанной сзеркалом? (В инерциальной системе отсчета свет от неподвижного источника в вакууме распространяется со скоростью  с).

1)

с

2)

с – υ

3)

c + υ

4)

c

  1. Два автомобиля движутся в одном и том же направлении со скоростями v1 и v2 относительно поверхности Земли. Скорость света от фар первого автомобиля в системе отсчета, связанной с другим автомобилем, равна  

1)  с - (v1 + v2)        2) с + (v1 + v2)              3)  c                          4)  с + (v1 - v2)

  4. Скорость света во всех инерциальных системах отсчёта

1)

не зависит ни от скорости приёмника света, ни от скорости источника света

2)

зависит только от скорости движения источника света

3)

зависит только от скорости приёмника света

4)

зависит как от скорости приёмника света, так и от скорости источника света

                                                                                                                                                                                                                                                                                                                                         

 5.     Один ученый проверяет закономерности колебания пружинного маятника в лаборатории на Земле, а другой ученый – в лаборатории на космическом корабле, летящем вдали от звезд и планет с выключенным двигателем. Если маятники одинаковые, то в обеих лабораториях эти закономерности будут

1) одинаковыми при любой        скорости корабля

  1. разными, так как на корабле время течет медленнее

  1. одинаковыми только в том случае, если скорость корабля мала

  1. одинаковыми или разными в зависимости от модуля и направления скорости корабля  

Методика решения задач части А

Задача  1.  Две ракеты удаляются от Земли в противоположных направлениях со скоростями v=0,8с относительно Земли. Определите скорость u относительного движения ракет.

Решение: Свяжем неподвижную систему отсчёта К с Землёй. Система отсчёта движется со скоростью v в направлении, совпадающем со скоростью движения второй ракеты.

Y                     Y/ 

                                    

        

      O                   O/                             X        X/                  

                                                                         

Обозначим скорость движения первой ракеты , а скорость движения второй ракеты .

                                                                           

Запишем закон релятивистский закон сложения скоростей для втор
                                                                         

С учётом проекций скоростей на ось х получим

  ;

 

                                                                       

                            

 Задача №2. Ракета, удаляющаяся от Земли со скоростью v, испустила пучок фотонов в направлении Земли со скоростью с относительно ракеты. Определите скорость u фотонов относительно Земли.

Решение: Свяжем неподвижную систему отсчёта К с Землёй, а подвижную систему отсчёта  с ракетой.

        Y                   Y/

                                             

                                                                       

                                                             

                                                                    X                  X/                                           

                                             

Запишем релятивистский закон сложения скоростей для пучка фотонов. 
                                                                       

 – проекция скорости фотонов на ось х в системе отсчёта К,  - проекция скорости фотонов в системе отсчёта .

                                                                 

                                             

                                                     

    Задача №3 Масса тела, движущегося с некоторой постоянной скоростью, возросла на 20% по сравнению с массой покоя. Во сколько раз при этом изменилась при этом его длина L по сравнению с собственной длиной ?

Решение: Пусть масса покоя тела равна, тогда при движении со скоростью v

Масса и длина тела, относительно неподвижной системы отсчёта будут определяться соотношениями:

                                ;  

умножим первое выражение на второе

                                                          ;                  

                                                             
                                                                                 

               

Длина тела сократится в 1,2 раза.

Методика решения задач части С.

Задача № C-1. Неподвижная частица массы M0 распадается на две одинаковые частицы, масса покоя которых. Найдите скорость v, c которой движутся эти частицы.

Решение: Распад частицы описывается законами сохранения импульса и энергии.

                                                                                                   

                                                              X     

                                           

По закону сохранения импульса

таким образом, скорости частиц после распада одинаковы.

Запишем закон сохранения энергии для распада исходной частицы:

энергия покоя исходной частицы превращается в полную энергию дочерних частиц:

                                                                     

где       – масса дочерней частицы.

         ;

;    

                                                   

Задача №С-2.Релятивистская частица с кинетической энергией T  и массой покоя  налетает на такую же покоящуюся частицу. Определите кинетическую энергию Т и массу покоя М составной частицы, образовавшейся при взаимодействии.

Решение: Соударение релятивистских частиц описывается законами сохранения импульса и энергии.

     

                                                                                                               Х

                                                           

По закону сохранения импульса, , где p1- импульс исходной частицы до соударения, р – импульс составной частицы.

В проекции на ось х получаем.

Импульсы частиц можно определить из выражения

;

 где E=T+moc2- полная энергия частицы

= (T+)- ; =;

таким образом, мы получаем для исходной частицы

и для составной частицы аналогичное выражение

.

Так как левые части этих выражений равны, правые части можно приравнять

 (1).

По закону сохранения энергии полная энергия исходной частицы и энергия покоя неподвижной частицы превращаются в полную энергию составной частицы                    

                                       (2).

Решим систему уравнений:

Возведём (2) в квадрат и вычтем из полученного выражения (1)

2;

из данного выражения получаем формулу для вычисления массы:

                                                               

Из (2) выразим Т:

подставим выражение для М:


Предварительный просмотр:


Предварительный просмотр:

Оптика.

Перечень элементов содержания раздела «Кинематика материальной точки», проверяемых на едином государственном экзамене по физике таков:

Код

раздела

Код

контролируемого

элемента

Элементы содержания,

проверяемые заданиями КИМ

3.6

Оптика

3.6.1

Прямолинейное распространение света

3.6.2

Закон отражения света.

3.6.3

Построение изображений в плоском зеркале.

3.6.4

Закон преломления света.

3.6.5

Полное внутреннее отражение.

3.6.6

Линзы. Оптическая сила линзы.

3.7.7

Формула тонкой линзы.

3.6.8

Построение изображений в линзах.

3.6.9

Оптические приборы. Глаз как оптическая система.

3.6.10

Интерференция света

3.6.11

Дифракция света

3.6.12

Дифракционная решётка

3.6.13

Дисперсия света

Обобщенный план экзаменационной работы ЕГЭ 2012 г. По физике

Обозна-чение

задания в

работе

Проверяемые элементы

содержания

Коды

элементов

содержания

по кодификатору

элементов

содержания

Коды

прове-ряемых

умений

Уровень

сложности задания

Макс.

Балл

за

выпол-

нение

задания

А 15

Оптика

3.6.1-3.6.4, 3.6.6, 3.6.8, 3.6.9

1, 2.1–2.4, 3

Б

1

А 16

Оптика

3.6.10-3.6.13

1, 2.1–2.4

Б

1

А 24

Оптика

3.6

2.6

П

1

А 25

Оптика

3.6

2.6

П

1

С 4

Оптика

3.6

2.6

В

3

С 5

Оптика

3.6

2.6

В

3

При подготовке к экзамену необходимо обратить особое внимание на повторение следующих явлений, понятий и определений:

Оптика (греч. Optike — наука о зрительных восприятиях, от  optos — видимый, зримый), раздел физики, в котором изучаются оптическое излучение (свет), процессы его распространения и явления, наблюдаемые при взаимодействии света и вещества.

Свет – электромагнитные волны, обладающие волновыми и квантовыми свойствами. В различных средах свет распространяется с различной скоростью; скорость света в вакууме и в воздухе составляет 300000 км/с.

В геометрической оптике часто пользуются понятием светового луча, т. е. узкого пучка световых волн.

Закон прямолинейного распространения света: Свет в однородной среде распространяется прямолинейно.

На прямолинейном распространении света основываются:

На границе раздела двух сред одновременно происходят два явления: отражение и преломление света.

Отражение света-физическое явление, при котором свет, падающий из одной среды на границу раздела с другой средой, возвращается назад (отражается) в первую среду.

 Различают:

Закон отражения света

Зеркало-тело с гладкой поверхностью, отражающей большую часть падающего на нее света. Зеркальные тела в основном состоят из стекла, кварца или металла и обладают полированной поверхностью. Гладкие поверхности жидкостей также выступают в качестве зеркал.

При описании прохождения света в зеркалах используются следующие понятия:

Возникновение изображения в плоском зеркале

При отражении возникает мнимое изображение (В) предмета (G). Расстояние s от предмета до зеркала равно расстоянию s' от зеркала до изображения.  Размер предмета у также равен размеру изображения у'.  Изображение прямое, мнимое.

        Если зеркальная отражающая поверхность представляет собой часть сферы, то такое зеркало называется сферическим. Центр сферической поверхности, точка С, называется оптическим центром зеркала. Вершина шарового сегмента, точка О, называется полюсом зеркала. Прямая, проходящая через оптический центр и полюс зеркала, называется главной оптической осью зеркала. Любая прямая, проходящая через оптический центр зеркала, называется побочной оптической осью. Сферические зеркала делятся на вогнутые и выпуклые.

Если на вогнутое зеркало падает пучок параллельных лучей, то после отражения от зеркала они пересекаются в одной точке — фокусе зеркала. Фокус F, лежащий на главной оптической оси, называется главным фокусом зеркала . Геометрическое место всех фокусов образует фокальную поверхность.

В случае выпуклого зеркала параллельный пучок лучей отражается так, что продолжения отраженных лучей пересекаются в одной точке F — мнимом фокусе.

Фокусное расстояние F зеркала, радиус кривизны которого R, равно

 

Если предмет находится на расстоянии  d  от зеркала, а его изображение — на расстоянии f от зеркала, то выполняется следующее соотношение:

      или                   

При построении изображений в сферических зеркалах  удобно использовать следующие лучи:                                                                              

а)  луч, параллельный главной оптической оси зеркала,  после отражения  проходит через главный фокус;                                                                                      

б) луч, проходящий через главный фокус, после отражения  идет параллельно главной оптической оси;                                                                                                 в) луч, проходящий через оптический центр зеркала,  после отражения идет по тому же направлению назад.                                                            

 Отношение линейных размеров  H изображения к линейным размерам  h самого предмета называется увеличением  зеркала  Г:                                            

Преломление света - физическое явление при переходе пучка света из одной оптической среды в другую. На границе раздела двух сред свет изменяет свою скорость и вместе с тем (для угла падения α≠0) направление своего распространения.

Законы преломления:

1. Луч падающий, луч преломленный и перпендикуляр к границе раздела двух сред лежат в одной плоскости.

2. Отношение синусов углов падения и преломления есть величина постоянная для данных сред (относительный показатель преломления).

sinα/sinβ=n21

Показатель преломления равен отношению скоростей света в данных средах: n21=v1/v2

Показатель преломления относительно вакуума называется  абсолютным показателем преломления и показывает, во сколько раз скорость света в данной среде  меньше скорости света в вакууме:  n=c/v

Среда с большим показателем преломления называется оптически более плотной. При переходе света из оптически более плотной среды в оптически менее плотную угол преломления больше угла падения.

Если угол падения превышает  

α0=arcsin n2/n1 (этот угол называется предельным углом полного отражения) происходит полное отражение света от границы раздела сред.

Прохождение света через плоскопараллельную пластинку: луч смещается, но сохраняет направление

Преломление света в линзах.

Прозрачное тело, ограниченное двумя сферическими или сферической и плоской поверхностями, называется линзой.

Линзы, превращающие падающий на них параллельный пучок лучей в пучок сходящихся лучей, называются собирающими. Это выпуклые линзы.  

Линзы, превращающие падающий на них параллельный пучок лучей в пучок расходящихся лучей, называются рассеивающими. Это вогнутые линзы.

Для построения изображений в линзах удобно использовать следующие лучи:                                                                                                                          

а) луч, параллельный главной оптической оси, после преломления проходит через главный фокус;                                                            

б) луч, проходящий через главный фокус, после преломления проходит параллельно главной оптической оси;                                                                

 в) луч, проходящий через оптический центр линзы, сохраняет направление  распространения.  

        Формула линзы и правило знаков совпадают с аналогичными формулой и правилом знаков для сферических зеркал, т. е.  

               

Отношение линейных размеров  H изображения к линейным размерам  h самого предмета называется увеличением  линзы  Г:      

Если предмет расположен на оптической оси линзы, то поперечное увеличение определяется соотношением:

k = F2 : (a – F)( b – F),

где  a  и  b – расстояния от концов предмета до линзы.

Полное увеличение оптической системы линз равно произведению увеличений, даваемых каждой линзой в отдельности:

Г =Г1*Г2*Г3

Фокусное расстояние F линзы, радиусы R1 и R2 кривизны сферических поверхностей и абсолютные показатели преломления n1 и п2 вещества линзы и окружающей среды связаны между собой отношением

где знаки перед слагаемыми, содержащими  R1  и R2 берутся положительными для выпуклых поверхностей и отрицательными - для вогнутых.

Величина, обратная фокусному расстоянию, называется оптической силой линзы:

Оптическая сила системы линз равна сумме оптических сил линз, входящих в систему:

D = D1+D2+D3 + ...

Такие явления, как интерференция, дифракция и дисперсия света, определяют волновые свойства света.

        Дисперсией света называется зависимость скорости света (и, следовательно, показателя преломления) от длины волны.

Белый свет (например, солнечный) представляет собой совокупность электромагнитных монохроматических (в переводе с латыни "одноцветных") волн, т. е. волн, имеющих определенную длину волны. В вакууме волны с любой длиной волны распространяются с одной и той же скоростью  

с=3*108 м/с. В веществе (стекло, вода и т. п.) скорость электромагнитных волн меньше, причем она зависит от длины волны.

В результате дисперсии пучок белого света

после преломления в призме разлагается

в разноцветный спектр.

Лучи спектра можно снова собрать

с помощью зеркал или другой призмы

и получить белый свет.

Интерференцией света называется явление наложения когерентных световых волн, в результате которого в одних местах пространства возникают максимумы, а в других— минимумы интенсивности света; при этом происходит перераспределение световой энергии в пространстве.

Когерентными называются световые волны, разность фаз которых остается постоянной во времени.

Условие максимума интенсивности света

Δd = kλ

где Δd - оптическая разность хода волн;  

λ- длина волны;        

k - целое число.

Условие минимума интенсивности света

Δd = (2k+1)λ/2

При интерференции света в проходящем свете условия максимума и минимума меняются местами.

Дифракция представляет собой отклонение от прямолинейного распространения света.

Наблюдается при прохождении света через малые отверстия или при огибании светом препятствий, размеры которых сравнимы с длиной световой волны.

Дифракционная решётка представляет собой пластинку с большим числом чередующихся прозрачных и непрозрачных полос (число полос может доходить до нескольких тысяч на 1 мм).

Используется для исследования спектрального состава света и измерения длины световой волны.

Максимум наблюдается под углом  φ, определяемым условием:

d sinφ = kλ,

 где k — порядок максимума (целое число),

λ — длина волны света,

d — период дифракционной решетки.

Это соотношение называется формулой дифракционной решетки.

Если направить на дифракционную решетку пучок белого света, на экране будет наблюдаться резкий белый центральный максимум (при k = 0), а максимумы других порядков будут наблюдаться при различных углах, благодаря чему на экране возникают разноцветные "спектральные" полосы.

В отличие от разложения света с помощью призмы при дифракции больше всего отклоняется не фиолетовый, а красный свет, поскольку у него больше длина волны.

Давлением света называют механическое действие, производимое электромагнитными волнами при падении на какую – либо поверхность. По характеру отражения и поглощения волн используют модели абсолютно черного тела ( ачт ) и абсолютно отражающего тела (аот ).  

Для ачт:                                      R = 0,

Для аот: :                    R = 1,

где Е – энергия волны, R – коэффициент отражения.

        Поглощение света веществом вызывает распад молекул и образование новых. Химические реакции, протекающие под действием света, называют фотохимическими,  а само действие света называют химическим.

Примеры тренировочных заданий.

Часть А.

А1. Поставим перед прозрачным стеклом 1 зажженную свечу 2. В стекле мы увидим изображение свечи. По другую сторону стекла (там, где мы видим изображение) поставим такую же, но незажженную свечу и будем передвигать ее до тех пор, пока она не покажется зажженной 3. Это будет означать, что изображение зажженной свечи находится там, где стоит незажженная свеча. С помощью линейки определим расстояние от свечи до стекла и от стекла до изображения. Какой(-ие) метод(-ы) изучения светового явления при этом используется(-ются)?

1) наблюдение

2) измерение

3) наблюдение и измерение

4) моделирование

А2.    Какая из точек — 1, 2, 3, 4 или 5 — является изображением точки S в плоском зеркале?

1) 1                2) 2                3) 3                4)  4 и 5        

A3. Мальчик стоит перед плоским зеркалом на расстоянии 1 м от него. Чему будет равно расстояние между ним и его изображением, если он подойдет на 0,5 м ближе к зеркалу?

1) 2 м                        2) 1 м                        3) 1,5 м                        4) 0,5 м

А4. Световой луч падает на границу раздела двух сред: воздух— стекло. Какое направление — 1, 2, 3 или 4 — правильно указывает ход преломленного луча?

1) 1                2) 2                3) 3                4) 4

А5. При попадании солнечного света на капли дождя образуется радуга. Это объясняется тем, что солнечный свет состоит из электромагнитных волн с разной длиной волны, которые каплями воды по-разному

1) поглощаются

2) отражаются

3) преломляются

4) рассеиваются

А6.    Зеленый цвет листвы деревьев объясняется тем, что

1) световые  лучи,   имеющие длину волны,   соответствующую зеленому цвету, поглощаются листвой

2) световые  лучи,   имеющие длину волны,   соответствующую зеленому цвету, отражаются листвой

3) листья деревьев содержат хлорофилл, излучающий зеленый свет

4) листья деревьев содержат хлорофилл, отражающий зеленый свет

А7. Предмет АВ, расположенный между фокусным расстоянием и двойным фокусным расстоянием (F < d1 < 2F) тонкой собирающей линзы, передвигают на двойное фокусное расстояние линзы (d2 = 2F). Изображение предмета при этом

1) удаляется от двойного фокусного расстояния линзы ,

2) приближается к двойному фокусному расстоянию линзы

3) удаляется от оптического центра линзы

4) приближается к фокусу линзы

А8. На рисунке изображено преломление светового пучка на границе воздух—стекло. Чему равен показатель преломления стекла? Ответ запишите с точностью до десятых.

1) 0,8                2) 1,0                3) 1,4                4) 12,0

А9. Вы многократно наблюдали за радужными переливами цветов тонкой пленки керосина или нефти на поверхности воды. Радужная окраска пленки на поверхности воды является проявлением…

1) отражения света от тонких пленок

2) преломления света в тонких пленках

3) интерференции света в тонких пленках

4) дисперсии света

А10. На плоскую непрозрачную пластину с двумя узкими параллельными щелями падает перпендикулярно плоская монохроматическая волна из красной части видимого спектра. За пластиной на экране наблюдается интерференционная картина. Если использовать монохроматический свет из фиолетовой части видимого спектра, то

1) расстояние между интерференционными полосами увеличится

2) расстояние между интерференционными полосами уменьшится

3) расстояние между интерференционными полосами не изменится

4) интерференционная картина повернется на 90°

А11. Одна сторона толстой стеклянной пластины имеет ступенчатую поверхность (см. рис.). На пластину, перпендикулярно ее поверхности, падает световой пучок. Длина падающей световой волны равна X. Световой пучок после отражения от пластины собирается тонкой линзой. При каком наименьшем из указанных значений высоты ступеньки d интенсивность света в фокусе линзы будет минимальной?

1) 1/4λ                2) 1/8λ                3) 1/2λ                4) λ

12. Непрозрачный круг освещается точечным источником света и отбрасывает круглую тень на экран. Определите диаметр тени, если диаметр круга 0,1 м. Расстояние от источника света до круга в 3 раза меньше, чем расстояние до экрана.

1) 0,03 м        2) 0,1 м                        3) 0,3 м                        4) 3 м

13.  Предмет, освещенный маленькой лампочкой, отбрасывает тень на стену. Высота предмета 0,07 м, высота его тени  0,7 м. Расстояние от лампочки до предмета меньше, чем от лампочки до стены в

1) 7 раз                2) 9 раз                        3) 10 раз                        4) 11 раз

14. Солнце находится над горизонтом на высоте 45°. Определите длину тени, которую отбрасывает вертикально стоящий шест высотой 1 м.

15. Какая часть изображения стрелки в зеркале видна глазу?

1) вся стрелка         2) 1/2        

3) 1/4                 4) стрелка не видна вообще

16. При каком из перечисленных ниже перемещений зеркала наблюдатель увидит стрелку в зеркале целиком?

1) стрелка уже видна глазу полностью

2) на 1 клетку влево

3) на 1 клетку вверх

4) на 1 клетку вниз

Часть В.

Задачи на соответствие условно можно разделить на следующие группы:

физическая величина

- изменение величины

физическая величина

- приборы

физическая величина

- формулы

физическая величина

- определение

физическая величина

- единицы измерения

ученые

- открытия

     

В1.

Установите соответствие между физическими величинами и приборами, с помощью которых эти величины определяют. К каждой позиции первого столбца подберите нужную позицию из второго и запишите в таблицу выбранные цифры под соответствующими буквами.

ФИЗИЧЕСКИЕ

ВЕЛИЧИНЫ

А) сравнительная сила света                    

Б) освещенность

В) фокус линзы

ПРИБОРЫ

1) люксметр

2) тонометр

3) фотометр

4) измерительная лента

5) транспортир

Ответ:

ФИЗИЧЕСКИЕ  ВЕЛИЧИНЫ                                    ПРИБОРЫ                                                                    

А) сравнительная сила света                  -               3) фотометр

Б) освещенность                                      -               1) люксметр

В) фокус линзы                                        -               4) измерительная лента

В2.

Установите соответствие между физическими величинами и формулами, по которым эти величины определяются.

К каждой позиции первого столбца подберите нужную позицию из второго и запишите в таблицу выбранные цифры под соответствующими буквами.

ФИЗИЧЕСКИЕ    ВЕЛИЧИНЫ  

 А) формула тонкой линзы

Б) условие максимума интерференции      

В) условие полного отражения  

Г)формула сферического зеркала

Д) оптическая сила системы линз                                                                                                         

ФОРМУЛЫ                                                 1. d = kλ

2.    =    +    

3. D = D1+D2+D3 +                 4.0 = n2 : n1     

5.    =    +        

Ответ:

ФИЗИЧЕСКИЕ    ВЕЛИЧИНЫ                                       ФОРМУЛЫ

А) формула тонкой линзы                                 -               2 .     =    +                    

Б) условие максимума интерференции            -               1.  d = kλ

В) условие полного отражения                         -                   4.sin i0 = n2 : n1     

Г)формула сферического зеркала-                                    5.   =    +            

Д) оптическая сила системы линз                   -                 3. D = D1+D2+D3 + …

В 3

Установите соответствие между физическими величинами и их единицами измерения.

К каждой позиции первого столбца подберите нужную позицию из второго и запишите в таблицу выбранные цифры под соответствующими буквами

ФИЗИЧЕСКИЕ  ВЕЛИЧИНЫ

ЕДИНИЦЫ ИЗМЕРЕНИЯ

А) Фокус линзы

  1. 1) лм

Б) Длина световой волны

  1. 2) м

В) Световой поток

  1. 3) рад

Г) Освещенность

  1. 4) лк

 Ответ:

ФИЗИЧЕСКИЕ  ВЕЛИЧИНЫ

ЕДИНИЦЫ ИЗМЕРЕНИЯ

А) Фокус линзы

  1. м

Б) Длина световой волны

  1. м

В) Световой поток

  1. лм

Г) Освещенность

     4.лк

 

В4

Установите соответствие между физическими величинами и явлениями и  их определениями.

К каждой позиции первого столбца подберите нужную позицию из второго и запишите в таблицу выбранные цифры под соответствующими буквами

ФИЗИЧЕСКИЕ

ВЕЛИЧИНЫ  И ЯВЛЕНИЯ

ОПРЕДЕЛЕНИЯ

А)  Увеличение линзы

Б) Предельный угол падения

В)  Дифракция света

Г) Когерентные волны

Д) Интерференция света

1) явление зависимости показателя преломления света от длины световой волны.

2) явление, возникающее при распространении света в среде с резко выраженными оптическими неоднородностями.

3) явление наложения когерентных световых волн, в результате которого в одних местах пространства возникают максимумы, а в других— минимумы интенсивности света;

4) угол, при котором преломленный луч скользит по границе двух сред.

5)   отношение линейных размеров  H изображения к линейным размерам  h самого предмета.

6)   световые волны, разность фаз которых остается постоянной во времени    

   7) величина, обратная фокусному расстоянию линзы

 Ответ:

А

Б

В

Г

Д

5

4

2

6

3

                 

В 5

Установите соответствие между физическими  открытиями и  именами ученых.

К каждой позиции первого столбца подберите нужную позицию из второго и запишите в таблицу выбранные цифры под соответствующими буквами

ИМЕНА УЧЕНЫХ

ОТКРЫТИЯ, ЗАКОНЫ

А) Максвелл

  1. 1) Дифракция света

Б) Френель

  1. 2) Интерференция света

В) Ньютон

  1. 3) Давление света

   

Ответ:

ИМЕНА УЧЕНЫХ

       ОТКРЫТИЯ, ЗАКОНЫ

А) Максвелл

          3. Давление света

Б) Френель

1.Дифракция света

В) Ньютон

2. Интерференция света

                                                   

В6

Свет длиной волны λ падает на дифракционную решетку, у которой  N штрихов приходится на 1 мм длины. Что произойдет с углом дифракции при увеличении длина световой волны?

К каждому элементу первого столбца подберите соответствующий элемент из второго и внесите в строку ответов выбранные цифры под соответствующими буквами.

ФИЗИЧЕСКИЕ ВЕЛИЧИНЫ

A. Угол дифракции

Б. Порядок спектра

ИЗМЕНЕНИЕ ВЕЛИЧИНЫ

1) увеличится

2) уменьшится

3) не изменится

Ответ:

А

Б

1

2

Часть С

С1. Телескоп имеет объектив с фокусным расстояние 1 м и окуляр с фокусным расстоянием 5 см. Какого диаметра изображение Солнца можно получить с помощью этого телескопа, если есть возможность удалять экран от окуляра до расстояния 1,5 м? Угловой диаметр Солнца 30'.

Решение:

№ этапа

Содержание этапа решения

Чертеж, график, формула

Оценка этапа в баллах

1

Объектив телескопа строит действительное изображение Солнца в фокальной плоскости, поэтому диаметр D1 созданного им изображения равен:

D1 = F1tgα

1

2

Ход лучей при получении изображения Солнца с помощью объектива и окуляра представлен на рисунке:

Из подобия треугольников следует:

1

3

Расстояние d от окуляра до изображения Солнца, построенного объективом, находим, используя формулу линзы:

Подставляя числовые значения величин, вычисляем диаметр изображения D2 Солнца на экране:

1

Максимальный балл

3

С2. Человек читает книгу, держа ее на расстоянии 50 см от глаз. Если это для него расстояние наилучшего видения, то какой оптической силы очки позволят ему читать книгу на расстоянии 25 см?

Решение:

№ этапа

Содержание этапа решения

Чертеж, график, формула

Оценка этапа в баллах

1

Обозначим оптическую силу глаза D1, оптическую силу очков D2, расстояние от центра хрусталика до сетчатки глаза обозначим f, расстояние до книги без очков d1, с очками d2. Тогда для случая чтения без очков по формуле линзы следует:

1

2

Для случая чтения с очками:

1

3

Из равенств 1 и 2 следует:

Подставляя числовые значения d1 и d2, получаем:

1

Максимальный балл

3


С3. Для наблюдения явления интерференции света используется точечный источник света и небольшой экран с двумя малыми отверстиями у глаза наблюдателя. Оцените максимальное расстояние d между малыми отверстиями в экране, при котором может наблюдаться явление интерференции света. Разрешающая способность глаза равна 1', длина световой волны 5,8 • 10-7 м.

Решение:

№ этапа

Содержание этапа решения

Чертеж, график, формула

Оценка этапа в баллах

1

Параллельные пучки света от двух отверстий как от когерентных источников фокусируются глазом в одну точку на сетчатке. Лучи, перпендикулярные    плоскости  экрана, не имеют разности хода. Лучи, выходящие из отверстий под углом ϕ к   перпендикуляру, имеют разность хода:

1

2

Первый  интерференционный максимум должен наблюдаться под углом φ1 к перпендикуляру,  удовлетворяющим условию равенства разности хода Δ одной длине λ световой волны:

Отсюда минимальное расстояние d равно:

1

3

Для  малых  значений угла значение  синуса угла примерно равно значению угла,  выраженному в радианах, поэтому:

Тогда для  расстояния  d  между  отверстиями при значении длины световой волны 5,8*10 -7 м получаем значение

1

Максимальный балл

3

Также рекомендуется разобрать решения следующих задач:

С4. В аквариум налита вода, в воду погружен источник света. Какие световые явления изображены на рисунке? Ответ обоснуйте. Цифрами обозначены световые пучки: 1, 2, 3, 4 и 5.

Ответ:

1, 5 лучи-явление полного отражения, т.к. эти лучи не выходят из первой среды во вторую

2, 4 лучи-явление преломления, при этом угол преломления больше угла падения, т.к. свет переходит в оптически менее плотную среду.

3 луч не преломляется, т.к. он падает перпендикулярно границе раздела двух сред (угол падения равен 0)

C5. На оси Ох в точке х1 = 0 находится оптический центр тонкой рассеивающей линзы с фокусным расстоянием F1 = -20 см, а в точке х2 = 20 см — тонкой собирающей линзы. Главные оптические оси обеих линз лежат на оси Ох. На рассеивающую линзу вдоль оси Ох падает параллельный пучок света из области х < 0. Пройдя данную оптическую систему, лучи собираются в точке с координатой х3 = 60 см. Найдите фокусное расстояние собирающей линзы F2.

Образец возможного решения (рисунок обязателен)

Ход лучей через систему линз изображен на рисунке:

Из рисунка ясно, что расстояние OS = —F1 = 20 см.

Расстояние от источника до собирающей линзы d = -F1 + (х2 – х1).

Расстояние от второй линзы до изображения  f равно (х3 – х2)

Формула тонкой собирающей  линзы:

                

Откуда фокусное расстояние второй линзы:

Ответ: F2= 20 см

C6. В дно водоема глубиной 3 м вертикально вбита свая, скрытая под водой. Высота сваи 2 м. Угол падения солнечных лучей на поверхность воды равен 30°. Определите длину тени сваи на дне водоема. Коэффициент преломления воды  n=4/3.

Образец возможного решения (рисунок обязателен):

Согласно рисунку длина тени L определяется высотой сваи h и углом γ между сваей и скользящим по ее вершине лучом света: L = h*tgγ.

Этот угол является и углом преломления солнечных лучей на поверхности воды. Согласно закону преломления:

C7. На поверхности воды плавает надувной плот шириной 4 м и длиной 6 м. Небо затянуто сплошным облачным покровом, полностью рассеивающим солнечный свет. Определите глубину тени под плотом. Глубиной погружения плота и рассеиванием света водой пренебречь. Показатель преломления воды относительно воздуха принять равным  4/3.

Образец возможного решения (рисунок обязателен).

Боковые грани очерчивают те лучи света, которые до преломления у краев плота распространялись вдоль поверхности воды. Согласно рисунку, глубину h тени можно определить по формуле

 где а –полуширина плота.

Значение tgγ найдем из закона преломления света:

где n — показатель преломления воды, а α=90°.

Имеем:

Ответ: 1,76 м

C8. Равнобедренный прямоугольный треугольник ABC площадью 50 см2 расположен перед тонкой собирающей линзой так, что его катет АС лежит на главной оптической оси линзы. Фокусное расстояние линзы 50 см. Вершина прямого угла С лежит ближе к центру линзы, чем вершина острого угла А. Расстояние от центра линзы до точки С равно удвоенному фокусному расстоянию линзы (см. рисунок). Постройте изображение треугольника и найдите площадь получившейся фигуры.

Образец возможного решения (рисунок обязателен)

Длина катетов

Длину х горизонтального катета А'С' изображения находим по формуле линзы:

Длина вертикального катета В'С' изображения равна а, т.к. для него d = f = 2F.

Площадь изображения:

C9. Условимся считать изображение на пленке фотоаппарата резким, если вместо идеального изображения в виде точки на пленке получается изображение пятна диаметром не более некоторого предельного значения. Поэтому, если объектив находится на фокусном расстоянии от пленки, то резкими считаются не только бесконечно удаленные предметы, но и все предметы, находящиеся дальше некоторого расстояния d. Оцените предельный размер пятна, если при фокусном расстоянии объектива 50 мм и диаметре входного отверстия 5 мм резкими оказались все предметы, находившиеся на расстояниях более 5 м от объектива. Сделайте рисунок, поясняющий образование пятна.

Образец возможного решения (рисунок обязателен)

Лучи, идущие от предмета на расстоянии d, собираются на расстоянии f, которое больше фокусного расстояния, и поэтому образуют на пленке пятно диаметром δ. Из подобия треугольников получаем соотношение:

Из формулы тонкой линзы

находим:  

 Получаем окончательно:

Ответ: δ = 0,05 мм.

C10. Два точечных когерентных источника света S1 и S2 расположены на расстоянии d друг от друга. На расстоянии L » d от источников помещен экран. Найти расстояние между соседними интерференционными полосами вблизи середины экрана, если источники излучают свет с длиной волны λ.

Образец возможного решения:

Пусть А — точка, в которой наблюдается интерференционный максимум, расположена на расстоянии h от середины экрана О. Разность хода световых волн,  излучаемых источниками, равна S2A – S1A.

 Из прямоугольного ΔS1AB:

По условию задачи d « L и  h « L, поэтому  (h - d/2)/L « 1. 

Известно, что при \х\ « 1

Поэтому

Из прямоугольного ΔS2AC:    S2A2 = L2 (l+ ((h+d/2)/L2)  или

Так как   (h+d/2)2/L<< l, то

Следовательно, разность хода

С  другой стороны, так как А — точка интерференционного максимума, то S2A – S1A = kλ, где k = 0, 1, 2. 

Следовательно, kλ = hd/L. Отсюда получаем, что интерференционные максимумы располагаются на экране симметрично относительно линии О1О на расстояниях  h = hk = kλL/d. Максимум, соответствующий целому числу k, называется максимумом порядка k. Максимум порядка (k + 1) будет находиться на расстоянии  hk+1 = (k + l)λL/d от середины экрана.

-   h,   =

Расстояние между максимумами   hk+1 – hk = (k + 1) λL/d - kλL/d = λL/d

Из последней формулы следует, что расстояние между максимумами тем больнее, чем больше отношение L/d.